MCAT

अब Quizwiz के साथ अपने होमवर्क और परीक्षाओं को एस करें!

Which of the following physical stimuli cannot be converted to electrical signals by the chemoreceptors of the human body? A. Light B. Taste C. Smell D. Blood Chemistry

A All of the answer choices can be converted to electrical signals by the human body that can then be transmitted to the central nervous system, but choices B, C, and D are converted by chemoreceptors, while choice A, light, is converted by electromagnetic receptors.

How many molecules of oxygen (O2) are required to complete the reactions of aerobic respiration beginning with four molecules of glucose? A. 24 B. 16 C. 4 D. 6

A. 24 Glucose is a six carbon sugar: C6H12O6

A neuron in an adult human is in which stage of the cell cycle? A. G0 B. G1 C. S D. G2

A. G0 Adult neurons are not preparing for, or in the process of cell division. Cells in the G0 phase cease to divide, describing these neurons perfectly.

The density of a human body can be calculated from its weight in air, Wair, and its weight while submersed in water, Ww. The density of a human body is proportional to: A. Wair/(Wair-Ww) B. (Wair-Ww)/Wair C. (Wair-Ww)/Ww D. Ww/(Wair-Ww)

A. Wair/(Wair-Ww) According to Archimedes' Principle, the ratio of the density of an object in air to the difference of submersed weight and weight in air.

What is the frequency of the emitted gamma photons? (Note: Use Planck's constant h= 6.6 x 10^(-34) Js and the elementary charge e= 1.6 x 10^(-19) A. 2.11 x 10 ^(35) Hz B. 3.38 x 10^(19) Hz C. 3.01 x 10^(-20) Hz D. 1.45 x 10^(-47) Hz

B. 3.38 x 10^(19) Hz The frequency f of a photon can be found from its energy using E=hf which gives f=E/h where h is Planck's constant.

What is the function of the RETICULAR FORMATION? A. Emotion and memory B. Arousal and alertness C. Hunger and thirst; emotion D. Smooth movement

B. Arousal and alertness

In reference to the photoelectric effect, which of the following will increase the kinetic energy of a photoelectron? A. Increasing the work function B. Increasing the frequency of the incident light C. Increasing the number of photons in the incident light D. Increasing the mass of photons in the incident light

B. Increasing the frequency of the incident light KE= hf-epsilon where h is Planck's constant, f is frequency, and epsilon is the work function of the metal. Note that photons are massless so D is eliminated.

What is the balanced half-reaction for the reduction of nicotinamide adenine dinucleotide (NAD+)? A. NAD+ + H+ --> NADH B. NAD+ + H+ + 2e- --> NADH C. NAD+ + 2e- --> NADH D. NADH --> NAD+ + H+ + 2e-

B. NAD+ + H+ + 2e- --> NADH When NAD+ is reduced, it accepts two electrons. When NAD+ is reduced, it accepts a hydrogen atom.

Gabapentin is a drug capable of blocking new synapse formation. Which type of memory would be LEAST affected in a patient treated with gabapentin? A. Working memory B. Sensory memory C. Long term memory D. Procedural memory

B. Sensory memory This question can be rephrased as: "Which type of memory is least dependent on new synapse formation?"

Which of the following is the correct electronic configuration for radioactive iodine, an isotope used in thyroid imaging? A. [Kr] 4d^10 5s^1 5p^6 B. [Kr] 4d^10 5s^2 5p^5 C. [Kr] 4d^10 5s^2 5p^4 D. [Kr] 5d^10 5s^2 5p^6

B. [Kr] 4d^10 5s^2 5p^5 Isotopes differ in number of neutrons not electrons...

What is the oxidation state of each nitrogen atom in hydrazine, N2H4? A. 0 B. -1 C. -2 D. -3

C. -2 If each H atom has an oxidation state of +1, the total of the oxidation states of the two N atoms must be -4 so that the molecule has a net oxidation state of zero. Therefore, each N atom has an oxidation state of -2

Which of the following increases with increasing atomic number within a family on the periodic table? A. Electronegativity B. Electron affinity C. Atomic radius D. Ionization energy

C. Atomic radius family is seen in vertical columns

Which term describes alterations in the composition of a gene pool due to migration of individuals between different populations? A. Assortive mating B. Directional selection C. Gene flow D. Genetic drift

C. Gene flow Genetic drift refers to changes in the frequency of alleles in a gene pool, simply due to chance. Assortive mating is when mates are not chosen randomly, but instead certain individuals are selected over others possibly because of phenotype or location. Directional selection occurs when one extreme of a trait is favored over another and the population starts to express the extreme.

A deficiency of oxaloacetate would most likely increase the catabolism of what potential energy source? A. Acetyl-CoA B. Ketogenic amino acids C. Glucogenic amino acids D. Acetoacetyl-CoA

C. Glucogenic amino acids Oxaloacetate (OAA) is an important intermediate in the citric acid cycle and in gluconeogenesis. OAA is principally formed from either malate or pyruvate. Ketogenic amino acids, as well as fatty acids, are principally metabolized to form acetyl-CoA. Glucogenic amino acids enter the citric acid cycle at multiple points, with the common theme of ultimately producing glucose. Many glucogenic amino acids are catabolized to pyruvate, or directly to form OAA.

Caspase enzymes mediate apoptosis by attacking which of the following? A. The cysteine residues in cytochrome c enzymes B. The cysteine residues in cytoplasmic proteins C. The asperatate residues in cytoplasmic proteins D. The cysteine residues in BCL-2 family proteins

C. The aspartate residues in cytoplasmic proteins Caspase enzymes are proteolytic enzymes found in the cytoplasm. Caspase enzymes are activated by cytochrome c. Caspase enzymes have cysteine-rich active sites that target aspartate residues in cytoplasmic proteins.

Which of the following organs CANNOT use fatty acids as an energy source? A. The liver B. Muscle C. The brain D. The kidney

C. The brain The brain cannot directly use fatty acids for energy, they must first be converted to ketone bodies.

What are the hormones secreted by the hypothalamus? A. Oxytocin and growth hormone B. Oxytocin and prolactin C. Prolactin and antidiuretic hormone D. Antidiuretic hormone and oxytocin

D. Antidiuretic hormone and oxytocin Prolactin and GH are both secreted by pituitary gland.

What powers the ATP synthase rotary motor? A. cAMP B. Coenzyme motive force C. GTP hydrolysis D. H+gradient

D. H+gradient

Which electron carrier would have the greatest negative impact on ATP production if its production was inhibited during oxidative phosphorylation? A. FADH2 B. Oxygen C. Water D. NADH

D. NADH Oxygen is an electron acceptor Water is not a reactant of oxidative phosphorylation NADH is the most important electron carrier, because it is more abundant than FADH2 ​​

Prior to infecting a bacterium, a bacteriophage must: A. reproduce, making copies of the phage chromosome. B. integrate its genomic into the bacterial chromosome C. penetrate the bacterial cell wall completely D. attach to a receptor on the bacterial cell membrane

D. attach to a receptor on the bacterial cell membrane

Which type of lethal, inheritable allele is most likely to persist in a population? A. Recessive B. Codominant C. Incompletely Dominant D. Dominant

A. Recessive A lethal allele can persist through carriers in the population. Carriers are generally heterozygous (1 disease allele; 1 healthy allele)

Which of the following is true concerning the induced fit model of enzyme catalysis? A. The active site can be influenced by molecules binding elsewhere on an enzyme B. The binding of enzyme and substrate is weakest in the transition state C. The induced fit must occur prior to the initial binding of enzyme and substrate in order for the reaction to proceed D. The initial binding of enzyme and substrate is the most tightly bound conformation

A. The active site can be influenced by molecules binding elsewhere on an enzyme Enzymes bind transition states more tightly than substrates. The order of a reaction must first undergo the initial binding before an induced fit occurs. Molecules can influence enzymes outside of the enzyme's active site. For example, in noncompetetive binding to an allosteric site can interfere with the binding of a substrate.

Researchers are seeking evidence to confirm a close evolutionary relationship between two extant species. Which of the following pieces of evidence would be LEAST likely to help the researchers demonstrate this relationship? A. The identification of several analogous structures between the species. B. The sequence of a ribosome-coding gene from each species. C. A fossil that shares traits with both species. D. The discovery of similar stages of embryonic development.

A. The identification of several analogous structures between the species.

Which of the following will be true regarding enzymes saturated with substrate? A. Increasing the substrate concentration will appreciably increase the reaction rate B. An enzyme with lower Km is more easily saturated than an enzyme with high Km C. Any excess substrate will shift the equilibrium towards the product end of the reaction D. At saturating levels of substrate, a competitive inhibitor will affect the reaction rate more than a non-competitive inhibitor

B. An enzyme with lower Km is more easily saturated than an enzyme with high Km In an enzyme catalyzed reaction, any spontaneous reaction of substrate to product is considered negligible. A non-competitive inhibitor will decrease the Vmax of the enzyme, since the enzyme-substrate inhibitor complex turns over to product at a slower rate than the enzyme-substrate complex. Km with units of molarity, is a general measure of the inverse affinity of the enzyme for a substrate. Therefore a low Km would indicate a high affinity, and therefore an enzyme with lower Km would be more easily saturated.

Fixed action patterns are responses to a particular type of stimuli that result from which of the following? A. Operant conditioning B. Cognitive responses C. Innate learning D. Classical conditioning

C. Innate learning A fixed action pattern is an automatic response, not learned behaviors. Fixed action patterns are considered "hard wired" or innate.

Participants in an attention study are given a dichotic listening task. Which of these changes to the sound transmitted to the unattended ear will NOT be noticed in a majority of the participants? A. A change from non-taboo content to taboo content. B. A change from spoken words to white noise. C. A change from a female to a male speaker. D. A change from English to Russian.

D. A change from English to Russian. Arousal determines our available capacity for attention. Narayan et al. discovered a sharp decline in the ability to discriminate between auditory stimuli when background noises were too numerous and complex.

Renin is not released in response to which of the following? A. Innervation by sympathetic nerve cells B. Detection of low glomerular filtrate electrolyte levels by macula densa cells C. Detection of low blood pressure by baroreceptor cells D. Detection of low plasma electrolyte levels by macula densa cells

D. Detection of low plasma electrolyte levels by macula densa cells Renin is released in response to signal that represent low mean arterial pressure. Sympathetic NS innervates juxtaglomerular apparatus and can signal in response to low blood pressure Macula densa cells detect low bp by sensing Na concentration in glomerular filtrate not in blood plasma.

Macula densa cells are in what part of the nephron? A. Glomerulus B. Bowman's capsule C. Proximal convoluted tubule D. Distal convoluted tubule

D. Distal convoluted tubule

Which of the following would most likely be found in the esophagus of a patient suffering from gastroesophageal reflux disease? A. Trypsin B. Chymotrypsin C. Carboxypeptidase D. Pepsin

D. Pepsin Pepsin is the most likely find, as it's secreted in the stomach by chief cells, while the other options are secreted into the small intestine.

As glomerular filtrate moves through the ascending loop of Henle, the filtrate becomes more dilute: why? A. ascending loop is impermeable to water and ions B. ascending loop is permeable to water and ions C. ascending loop is permeable to water but not ions D. ascending loop is permeable to ions but impermeable to water.

D. ascending loop is permeable to ions but impermeable to water. Glomerular filtrate contains water and small molecules. It can become more dilute by either increasing its concentration of water or decreasing its concentration of small molecules. descending loop is permeable to water, the ascending loop is impermeable.

Photuris lucicrescens is a bioluminescent coleoptera. In simple words, this just means the lightning bug, is a beetle that can glow in the dark. Is bioluminescence an endergonic or exergonic reaction, what is the ΔG of bioluminescence? A. Endergonic reaction with negative ΔG. B. Exergonic reaction with positive ΔG. C. Exergonic reaction with negative ΔG. D. Endergonic reaction with positive ΔG.

D.Endergonic reaction with positive ΔG Bioluminescent releases energy in the form of light, but it consumes a greater amount of energy (in the form of ATP) in the process! Hence this is an endergonic reaction, with positive ΔG.

Protecting an alcohol with tosyl chloride would likely have which of the following effects? A. Decreasing its London dispersion force. B. Decreasing its solubility in water. C. Increasing its acidity. D. Expose the molecule to a strong nucleophile.

B. Decreasing its solubility in water. The point of a protecting group is to protect the hydroxyl group and prevent it from reacting with a nucleophile. The hydroxyl will lose its ability to donate its hydrogen ion and thus we would expect a decrease in the acidity of the molecule. The solubility of a tosylate compound should decrease since the hydroxyl was smaller and could interact with water through hydrogen bonds.

Aerobic cellular respiration involves the flow of electrons from one chemical species to another: which of the following best describes this process? A.Electrons are transferred from glucose to oxygen B.Electrons are transferred from glucose to carbon dioxide C.Electrons are transferred from oxygen to water D.Electrons are transferred from oxygen to glucose

A. .Electrons are transferred from glucose to oxygen Glucose is the ultimate source of electrons for aerobic cellular respiration. Oxygen is the final electron acceptor in aerobic cellular respiration.

Placenta accreta is a medical condition in which the placenta attaches abnormally to the myometrium. What is the earliest embryonic stage that this could occur, which cells are responsible for this attachment? A. Early cleavage, embryoblast B. Blastulation, epiblast C. Blastulation, trophoblast D. Early cleavage, trophoblast

C. Blastulation, trophoblast Early cleavage is not directly linked with hypoblast formation. Placenta is mainly formed by trophoblast cells. Blastulation is the process that lead to the formation of trophoblasts, these cells will form part of the placenta.

Where is the first place in the brain where the olfaction and gustation systems integrate? A. Thalamus B. Nucleus of the solitary tract C. Orbitofrontal ortex D. Amygdala

C. Orbitofrontal ortex The nucleus of the solitary tract is the location where the nerves from the mouth and the tongue synapse on the medulla. The pathway for olfaction goes form the olfactory bulb to either the amygdala or the piriform cortex. From there the signal is transmitted to the orbitofrontal cortex. Taste sensory information does not pass through the thalamus before being integrated with signal from olfaction.

The reaction A + B ⇆ C is an exothermic reaction; if an equilibrium mixture of A, B, and C is heated slowly, what would happen to the concentrations of each species? A. All three concentrations increase B. A and B decrease, C increases C. All three concentrations decrease D. A and B increase, C decreases

D. A and B increase, C decreases Exothermic rxn produces heat as product If product of rxn is added to equilibrium mixture of elements of the reaction, equilibrium will shift towards reactants.

Tollen's reagent, Ag+, is an oxidation agent. What are the products of Tollen's reagent reaction with the following molecule? CH3-CH2-C(=O)H A. Propanoic acid and Ag+ B. Propanol and Ag+ C. Propanoic acid and Ag D. Propanol and Ag

C. Propanoic acid and Ag Oxidizing agents are substances that become reduced after oxidation of another molecule. Oxidation of an aldehyde will yield a carboxylic acid. The propanal is oxidized to a carboxylic acid and the silver ion is reduced to metallic silver.

Which of these criteria is necessary for a biological rhythm to be described as a circadian rhythm? A. The rhythm must be expressed according to social factors that determine the period of the cycle. B. The rhythm must reflect a light dark cycle. C. The rhythm must be internally generated. D. The rhythm must have a period of no more than 24 hours.

C. The rhythm must be internally generated. There are differences in the role of activity, food intake, light, and social factors on setting circadian rhythm. Although an approx 24 hr sleep--wake rhythm is necessary for a biological rhythm to be classified as circadian, many animals and other life have circadian rhythms that are slightly longer than 24 hrs. It has been shown that a person who is isolated in a bunker and deprived of light and dark cues will assume a free-running rhythm that will shift between 24 and 26 hrs. The rhythm must be internally generated.

An organic molecule has been isolated from the nucleus of a human cell. NMR studies reveal the core structure of the molecule to be 6-membered ring containing carbon and nitrogen. The ring also expresses an amine group and a keto group, and is attached to a five carbon sugar moiety. No phosphate groups are noted. What is the most likely identity of the molecule? A. Cytosine B. Guanosine C. Guanine D. Cytidine

D. Cytidine DNA largely made of Adenine, Guanine, Cytosine,and Thymine. When bases attached to a ribose moiety, the five carbon sugar is called nucleosides. Pyrimidine are six membered heterocyclic ring of carbon and nitrogen with various substitutions on it's four carbon atoms. Purines are made of prymidine plus imidazole ring made of two nitrogens and a carbon atom. A and G= purines C and T= pyrimidines

Recognition of self vs. non-self by the adaptive immune system in humans is accomplished in which of the following ways? A. Exposure of T cells to the body's own antigens in the thymus B. Exposure of B cells to the body's own antigens in the thymus C. Exposure of T cells to the body's own antigen in the bursa of Fabricius D. Exposure of B cells to the body's own antigens in the bursa of Fabricius

A. Exposure of T cells to the body's own antigens in the thymus The bursa of Fabricius is an organ found in birds; B-cells were originally found in this organ. The thymus is an organ in humans where T-cells mature.

A patient comes in with abdominal pain. Their lab results reveal normal levels of pancreatic enzymes, pepsinogen, and both stomach and gall bladder motility, but an acidic small intestine environment. Which hormone is not in balance? A. Leptin B. Secretin C. Gastrin D. Cholecystokinin

B. Secretin The digestive system is controlled by a series of hormones that trigger or inhibit each other to control digestion and absorption, mostly independently of other body systems. Gastrin is secreted in the stomach and triggers stomach motility and cholecystokinin release, among others. Secretin triggers the release of pancreatic bicarbonate solution, which would protect the small intestine from the acidity of the stomach contents. It is this hormone which is malfunctioning in the patient.

In human physiology endergonic and exergonic reaction are often coupled. How does the ATP/ADP cycle couples these reactions? A.Exergonic phosphorylation of ADP B. Endergonic phosphorylation of molecule by ATP. C. Exergonic hydrolysis of ATP D. Endergonic hydrolysis of ADP

C. Exergonic hydrolysis of ATP ADP hydrolysis is possible but it does not drive exergonic reactions ADP phosphorylation is an endergonic process. Exergonic ATP hydrolysis releases energy to drive endergonic processes.

Which of these people is NOT accruing a sleep debt? A. A 6-month-old baby who gets 11 hours of sleep a night. B. A 4-year-old who gets 9 hours of sleep a night. C. A 9-year-old boy who gets 8 hours of sleep a night. D. A 40 yo woman who gets 7 hours of sleep a night.

D. A 40 yo woman who gets 7 hours of sleep a night. A preschooler (3 to 5) should get at least 10 hours of sleep A school age child (6 to 13) should get at least 9 hours of sleep a night. An infant (4 mo to 11 mo) should get at least 12 hours.

Which of the following best describes the respective free energy changes for the one-step combustion of glucose in the presence of oxygen as compared to the multi-step oxidation of glucose during cellular respiration, assuming both reactions occur under identical conditions? A. The combustion has a higher change in free energy B. More information is needed to answer the question C. The combustion has a lower change in free energy D. Both reactions have the same change in free energy

D. Both reactions have the same change in free energy The multi-step oxidation of glucose during cellular respiration results in the same products as the one-step combustion of glucose and oxygen. Free energy change is "path-independent," meaning that it is a quantity that depends only on the initial and final state of a chemical system.

The pancreas arises from which embryonic germ layer? A. Ectoderm B. Mesendoderm C. Mesoderm D. Endoderm

D. Endoderm The skin and nervous system arose from the ectoderm. The pancreas arose from the endoderm, as did the rest of the GI system.

Which of the following is not a characteristic of social anomie? A. Weakened social norms B. Social uncertainty C. Alienation from social groups D. Strengthened social norms

D. Strengthened social norms Anomie: lack of the usual social or ethical standards in an individual or group. This means that there is a weakened sense of morality and criteria for behavior. To resolve social anomie, social norms must be strengthened and groups must redevelop sets of shared norms.

Which of the following best predicts the result of raising the pH of an equilibrium mixture for the reaction ATP + H2O ⇆ ADP + Pi + H+ A. [ADP] decreases B. [ATP] increases C. [water] increases D. [ADP] increases

D. [ADP] increases Raising pH of solution decreases concentration of H+ in the system. Decreasing the concentration of a product in an equilibrium mixture of reactants and products will result in the increased concentration of products and decreased concentration of reactants (Le Chatelier's Principle)

Which of the following is a correct statement explaining why toluene is an aromatic molecule? A. All electron contributing atoms to the aromatic structure are on the same plane. B. All atoms involved in the ring have sp-orbitals that overlap. C. Toluene is free from heteroatoms replacing one or more carbon atoms in the ring. D. The number of delocalized p-orbital electrons follows the rule 2n + 2, where n = 1, in the case of toluene.

A. All electron contributing atoms to the aromatic structure are on the same plane. Aromaticity follows Huckel's Rule, which states that the number of delocalized p-orbital electrons follows the rule 4n + 2 where n = 1 in the case of toluene. Heteroatoms can replace carbon atoms in a ring and can still be aromatic such as in the case of pyridine. All atoms involved in the ring have p-orbitals that overlap. This contributes to the molecule's stability. In order to have overlapping p-orbitals, it necessary for electron contributing atoms to be in the same orientation on the same plane.

On which of the following physical properties does the separation of the constituents of a mixture by column chromatography depend? A. Differential adsorption B. Indices of refraction C. Molecular weight D. Boiling points

A. Differential adsorption Column chromatography is based around the choice of the adsorbent, whether it be a polar or non-polar. Ion-exchange, affinity, and size-exclusion are some of the additional uses of column chromatography that takes advantage of solute's charge, ability to bind antibodies and to navigate through small porous beads. Gas chromatography can be used to separate a mixture of boiling points Size-exclusion chromatography can be used to separate a mixture by molecular weight, but size, of course, foremost.

Chemical reactions can be classified according to free energy changes. A chemical reaction has a ΔG of -686/kcal/mol. Is this an endergonic or exergonic reaction? How would the addition of catalyst change the ΔG this reaction? A. Exergonic, the catalyst would not reduce ΔG. B. Endergonic, the catalyst would reduce the ΔG. C. Exergonic, the catalyst would reduce the ΔG. D. Endergonic, the catalyst would increase the ΔG.

A. Exergonic, the catalyst would not reduce ΔG. A catalyst, or enzyme, lowers the activation energy but it does not reduce ΔG

Which of these papillae categories does not contain taste buds? A. Filiform papillae B. Foliate papillae C. Circumvallate papillae D. Fungiform papillae

A. Filiform papillae Fungiform papillae are mushroom-shaped located on the tip and sides of the tongue, which contain taste buds. Foliate papillae are folded structures at the back of the tongue on both sides, which contain taste buds. Circumvallate papillae are flat mound structures that are found at the back of the tongue and contain taste buds. Filiform papillae do not contain taste buds and exist all over the tongue. The center of the tongue contains only filiform papillae. This is why stimulation of the center of the tongue does not cause a taste sensation, while the back and perimeter produce a broad range of taste sensations.

What path do bile salts take? A. Liver, gall bladder, duodenum, jejunum, ileum, liver B. Liver, gall bladder, duodenum, jejunum, tissues C. Gall bladder, stomach, duodenum, juejunum, ileum, tissues D. Gall bladder, stomach, duodenum, tissues, liver

A. Liver, gall bladder, duodenum, jejunum, ileum, liver Bile salts help fat be emulsified and absorbed. Bile salts are recycled by the body. Bile travels from the liver to the gall bladder through the common hepatic and cystic ducts, then to the duodenum through the cystic and common bile ducts, passes through the jejunum, and is absorbed through the ileum and returned to the liver to begin the process again.

Which of the following is not one of the three main antigen-presenting cell types? A. Natural Killer Cells B. Dendritic Cells C. B lymphocytes D. Macrophages

A. Natural Killer Cells Macrophages and dendritic cells play an important role as antigen-presenting cell types. B-cells are mainly known as antibody producing cells, but they also can play a role as antigen-presenting cells. Natural killer cells, part of the innate immune system, secrete cytotoxic chemicals meant to destroy bacteria, cells infected by a virus, tumor cells, etc.

Which of the following statements accurately describes the difference between a sigma and a pi bond? A. Only sigma bonds use hybridized orbitals to overlap in forming the bond. B. Sigma bonds form from the end-to-end overlap of two hybridized s or p orbitals C. Pi bonds form from the side-to-side overlap of hybridized sp, sp2, or sp3 orbitals. D. The max number of sigma vs pi bonds between two atoms are different, but the energy level of the two bonds are the same.

A. Only sigma bonds use hybridized orbitals to overlap in forming the bond. In regards to total possible number of bonds between two atoms, the total possible number of bonds for a sigma bond is one and for a pi bond is two. Sigma bond is lower in energy than pi bond. sigma bond will form from an end-to-end overlap of an s, p or hybridized orbital. pi bond will only form from side to side overlap of two unhybridized p orbitals. since pi bonds only use unhybridized orbitals, only sigma bonds use hybridized orbitals to overlap in forming the bond.

Jim is walking down a quiet street. Suddenly, he hears a noise which captures his attention. As he begins attending to this noise, he turns his body toward the noise, to maximize the flow of sensory information. What term is used to describe Jim's actions? A. Overt orienting B. Endogenous attention C. Covert orienting D. Exogenous attention

A. Overt orienting Exogenous attention is driven by bottom-up or external events, i.e. pop-out. Endogenous attention is driven by top-down or internal events, i.e. the cocktail effect. Covert orienting is the act of bringing the spotlight of attention on an object or event without body or eye movement. During overt orienting, a person turns all or part of the body to alter or maximize the sensory impact of an event.

The drug disulfiram works by antagonizing acetaldehyde dehydrogenase by binding to copper ions. This prevents the liver from breaking acetaldehyde into acetic acid. Individuals with a gene variant that causes slower metabolization of acetaldehyde are more likely to experience adverse effects from a buildup of acetaldehyde. It has been shown that about half of the people in China and Japan have this gene. What effect could this have on the culture of alcohol abuse in China and Japan? A. The gene variant that causes slower metabolization of acetaldehyde would lower the likelihood of alcohol abuse due to the adverse effects of the buildup of acetaldehyde. B. The gene variant that causes slower metabolization of acetaldehyde would lower the likelihood of alcohol abuse due to the beneficial effects of the buildup of acetaldehyde. C. The gene variant that causes slower metabolization of acetaldehyde would raise the likelihood of alcohol abuse due to the adverse effects of the buildup of acetaldehyde. D. The gene variant that causes slower metabolization of acetaldehyde would raise the likelihood of alcohol abuse due to the beneficial effects of the buildup of acetaldehyde.

A. The gene variant that causes slower metabolization of acetaldehyde would lower the likelihood of alcohol abuse due to the adverse effects of the buildup of acetaldehyde. The buildup of acetaldehyde causes symptoms such as nausea, headache, flushing of the face, and internal organ damage. A person born with the gene variant for slower acetaldehyde metabolism would cause the individual to experience the effects associated with a hangover more quickly and with a smaller quantity of alcohol consumption.

Suppose that, in a mass spectrometer, charged isotopes enter the device with velocities along a direction that is neither perpendicular nor parallel to the magnetic field lines. Which of the following behaviors would result? A. The isotopes would travel in a corkscrew pattern B. The isotopes would travel in elliptical orbits C. The isotopes would commence uniform circular motion D. The isotopes would not be deflected

A. The isotopes would travel in a corkscrew pattern The Lorentz force law states that the magnitude of the force acting on the isotopes goes as qvBsin theta, where theta is the angle between the field lines and the velocity. The velocity of the particle can be written in terms of a component perpendicular to the field, vperp, and a component along the field line, vpara. The Lorentz force in the perpendicular direction is thus qv(perp)B, but it is zero in the parallel direction. The particle thus only undergoes circular motion in the plane perpendicular to the field line, and it travels inertially along the direction of the field lines. The charged particle thus traces out a corkscrew trajectory as it moves in the device.

Damage to which gland of the oral cavity would lead to an increase in the concentration of triglycerides in the esophagus? A. Von Ebner's B. Submandibular C. Sublingual D. Parotid

A. Von Ebner's Triglycerides are composed of three fatty acids and a glycerol molecule, and are fat polymers. Fat is digested by lipase. Von Ebner's gland secretes lingual lipase, and damage to this gland could lead to decreased fat digestion and increased triglyceride concentration in the esophagus.

Which of the following pairs of charged isotopes would be impossible to distinguish using a standard mass spectrometer? A. 12/21 Na+ and 12/12 Mg 2+ B. 12/11 Na+ and 24/20 Ca 2+ C. 12/11 Na+ and 14/11 Na+ D. 12/11 Na+ and 6/3 Li+

B. 12/11 Na+ and 24/20 Ca 2+ In the standard derivation of the mass spectrometer, the centripetal acceleration due to the Lorentz force results in an equation for the radius of deflection, r = mv/qB, where q and m are the charge and mass of the isotope, respectively. B is the deflecting field, which is the same for all isotopes. v is the velocity with which the particles enter the magnetic field region. The voltage that accelerates electrons to velocity vvv before entering the field region is set by energy conservation, qV = (1/2)mv^2, where V is a voltage set by the machine that remains constant for all ions. This means that the entering velocity v depends only on the ratio q/m for the isotopes. The pair of isotopes, 12/11 Na+ and 24/20 Ca 2+, are thus impossible to distinguish because they have the same charge-to-mass ratio and thus are deflected the same amount by the magnetic field. In an isotope: 12/11 Na+ 12 = atomic mass 11= atomic number

Which of the following carboxylic acids will yield the least amount of protons in an aqueous solution? A. butanoic acid B. 2-methylbutanoic acid C. 2-chlorobutanoic acid D. propanoic acid

B. 2-methylbutanoic acid The carboxylic acid that will yield the least amount of protons in solution will be the least acidic molecule. Chloro substituents are electron withdrawing which makes the molecule more acidic by stabilizing the conjugate base anion. A carboxylic acid with a longer carbon chain is less acidic due to the greater induction of electrons destabilizing the conjugate base anion.

Which of the following statements most accurately describes thin-layer chromatography? A. The stationary phase consists of a solid non-polar adsorbent, usually silica or alumina gel. B. A solvent with a greater elution power than petroleum ether is acetic acid. C. Rf is constant only if the temperature and solvent system is also unchanged. D. The mobile phase is the solvent, also called the eluent, and moves by gravity through the column.

B. A solvent with a greater elution power than petroleum ether is acetic acid. In thin-layer chromatography, there is a mobile phase or solvent which can vary in its eluting power. The more polar a solvent, the greater the eluting power. This eluent usually moves by capillary action up the plate, not by gravity. The stationary phase consists of a solid polar, not non-polar, adsorbent, usually silica or alumina gel. Rf values are only constant from one experiment to another if all the following conditions are also constant: solvent system, adsorbent, thickness of the adsorbent, amount of material spotted, and temperature. The order of eluting power from least to greatest: petroleum ether (hexane;pentane), cyclohexane, carbon tetrachloride, benzene, dicholoromethane, chloroform, ether (anhydrous), ethyl acetate (anhydrous), acetone (anhydrous), ethanol, methanol, water, pyridine, and finally organic acids.

Major histocompatibility complex (MHC) refers to a large group of genes that code for proteins that play an essential role in which of the following? A. Phagocytosis by neutrophils B. Antigen presentation to T lymphocytes C. Antigen presentation to B lymphocytes D. Phagocytosis by macrophages

B. Antigen presentation to T lymphocytes MHC genes code for cell surface proteins that play a critical role in antigen presentation. B lymphocytes produce antibodies and act as antigen presenting cells. Antigen presenting cells present their antigens to various types of T cells.

500 mL of water and 500 mL of dicholoromethane are added to a mixture containing benzoic acid, cresol, methoxyethane, and N-methylethanamine. Four solutions are available for extraction of the mixture: HCN, HCl, NaOH, and LiHCO3. After the initial wash, the top later from each extraction was retained and one of the preceding solutions was added. Which of the following statements most accurately described this procedure? A. Cresol must be extracted first with a strong mineral acid such as HCl B. Benzoic acid can be extracted and isolated with either LiHCO3 or NaOH. C. After washing with LiHCO3 and then NaOH, only N-methylethanamine and methoxyethane remain in the separatory funnel. D. Methoxyethane will only be extracted upon protonation into a carbocation.

B. Benzoic acid can be extracted and isolated with either LiHCO3 or NaOH. Cresol is like phenol, but with an extra methyl group at the para position. Due to its alcohol functional group, it's considered a weak acid and would only be extracted with the stronger base. With base extraction, consider the strength of the acid before picking the base. Benzoic acid can be extracted and isolated with either a weak or strong base, but if in a mixture with an alcohol, it should be extracted first with a weak base like LiHCO3 and the alcohol extracted a strong base like Na2S.

Which of the following compounds is the most acidic? A. butane B. butanal C. 2-butanone D. 1-butanone

B. Butanal Butane is an alkane that is relatively unreactive. Ketones and aldehydes have acidic hydrogens in the alpha-carbon position. Butanal is the most acidic because it is an aldehyde. The more acidic substance will be the one whose conjugate base is more stable. Since the conjugate base carries a negative charge, electron-withdrawing substituents enhance stability by delocalizing that charge. Conversely, alkyl groups are electron-donating relative to -H, so the ketones have a conjugate base less stabilized than that of aldehydes.

Which of the following has the highest boiling point? A. butanal B. butanol C. propanal D. propanol

B. Butanol Higher boiling points are the result of strong intermolecular forces holding molecules together. Alcohols have higher boiling points than aldehydes due to hydrogen bonding donor and acceptor capabilities. As chain length gets longer, the molecular weight increases, and the London dispersion force increases. This increases the overall intermolecular force, which raises boiling point. Butanol is the alcohol that has the longest chain. It should have the highest boiling point of the molecules listed.

ATP cycle refers to a set of reactions that promotes reversible changes of ATP into ADP via endergonic and exergonic reactions. According to Le Chatellier's principle, what would you expect to occur if ADP levels increased? A. Decreased levels of ATP B. Decreased levels of inorganic phosphate C. No change in delta G D. Increased rate of the hydrolysis reaction

B. Decreased levels of inorganic phosphate If ADP levels increased, the endergonic phosphorylation of ADP into ATP would be favored, therefore decreasing the levels of inorganic phosphate.

An attention researcher interested in cross-modal interference in perception designs a 2X2 within-subjects factorial experiment. The participant is required to detect whether a visual stimulus, given just above the threshold of perception, is present or not present, with or without sound interference. When debriefing the participants, one states "I was more attentive for the visual stimulus because I knew I was being watched." When the data was examined, that participant showed many false positives, when there was sound interference. What is the likely threat to validity posed by the participant quoted? A. Due to the role demands perceived by the participant, he or she was more likely to act according to the behavior the participant believes is expected by the experiment. This effects the cause-effect relationship or the external validity of the study. B. Due to the Hawthorne effect, the participant changed his or her behavior in response to being observed. This effects the cause-effect relationship or the internal validity of the study. C. Due to good-subject tendency, the participant was more likely to act according to the behavior the participant believes is expected by the experimenter. This effects the cause-effect relationship or the external validity of the study. D. Due to the criterion demands perceived by the participant, he or she was more likely to act according to the behavior the participant believes is expected by the experiment. This effects the cause-effect relationship or the internal validity of the study.

B. Due to the Hawthorne effect, the participant changed his or her behavior in response to being observed. This effects the cause-effect relationship or the internal validity of the study. Internal validity describes the extent that a study is able to show a cause-effect relationship between the variables tested in the study. External validity describes the extent that the results of a study can be generalized or repeated in multiple settings. The good-subject tendency refers to the tendency of participants to act according to what they think the experimenter wants. A participant's role demands refers to the expectations of the participant regarding what an experiment necessitates him or her to do. The Hawthorne effect occurs when an individual participant changes his or her behavior, specifically due the awareness of being observed. Due to the Hawthorne effect, the participant changed his or her behavior in response to being observed. This effects the cause-effect relationship or the internal validity of the study.

Which of the following reactions are ways to synthesize carboxylic acids? I. Oxidation of an aldehyde II. Hydrolysis of an anhydride III. Hydrolysis of a ketone Please choose from one of the following options. A. I B. I & II C. II & III D. I & III

B. I & II Oxidizing an aldehyde would add an additional oxygen atom to the functional group and successfully convert the aldehyde into a carboxylic acid. I is correct. When an anhydride undergoes hydrolysis, one of the carbonyl leaves and a hydrogen ion attaches to the bridge oxygen. This process forms two carboxylic acids. II is correct. Since both of the groups flanking the carbonyl carbon in a ketone are alkyl groups, there is no readily available leaving group on the ketone molecule after the H2O addition to the carbonyl carbon. III is incorrect.

Which of the following are true statements regarding enolate formation? I. Overall the more substituted a double bond on an enolate, the less stable the molecule is. II. Low temperature reactions favor a more rapidly formed enolate III. Thermodynamic enolates are favored in reactions with strong, sterically hindered bases. A. I + III B. II only C. II + III D. None of the above

B. II only Kinetic enolates are more rapidly formed, but are less stable. Thermodynamic enolates are slower to form, but are more stable Saytseff's (or Zaitsev's) rule states that the more substituted alkene will form in preference to the less substituted molecule. So, generally, the more substituted a double bond is, the more stable the alkene will be. A strong, sterically hindered base will favor the less hindered portion of a molecule, which is more likely to result in a kinetic enolate Lower temperatures favor the formation of the kinetic enolate; therefore statement II is correct.

ACE inhibitors lower blood pressure: how? A. Inhibit renin, disrupts the production of angiotensin 1 from angiontensinogen B. Inhibit angiotensin converting enzyme, which disrupts the production of angiotensin 1 and angiotensin 2 C. Inhibit angiotensin converting enzyme, which disrupts the production of angiotensin 1 from angiotensin 2 D. Inhibit renin, which disrupts the production angiotensin 2 from angiotensinogen

B. Inhibit angiotensin converting enzyme, which disrupts the production of angiotensin 1 and angiotensin 2

Which of these routes of entry would have the highest addiction potential? A. Oral B. Intramuscular C. Insufflation D. Transdermal

B. Intramuscular The faster a drug reaches the brain, the higher the addiction potential. Oral administration of a drug is the slowest route of entry into the body. Transdermal administration occurs slowly, since the drugs have to be absorbed through the skin before the effects can be felt. Insufflation, inhaling drugs through the nose, is highly addictive but less addictive than drugs that are injected. Intramuscular injection is the fastest route of entry. Most abused drugs are injected intravenously, however.

Given that nicotinamide adenine dinucleotide (NAD), in its reduced form as NADH, is a key electron-donating molecule in many metabolic reactions, which of the following is a good prediction for the standard reduction potential (E^deg) of the reduction half-reaction of NAD? A. Large and positive B. Less than zero C. Zero D. Between zero and 1

B. Less than zero The standard reduction potential is a measure of the tendency of a chemical species to be reduced. Electron accepting molecules like NAD have a strong tendency to be reduced under physiological conditions. The result is the electron-carrying molecule NADH A good prediction for the standard reduction potential of the reduction half-reaction of NAD is "less than zero."

What is the primary difference between mores and folkways? A. Mores regulate nonmaterial culture, whereas folkways regulate material culture B. Mores are based in ethics, whereas folkways are based in social approval C. Mores regulate material culture, whereas folkways regulate nonmaterial culture D. Deviance from mores is always punishable by law, whereas deviance from folkways is no

B. Mores are based in ethics, whereas folkways are based in social approval Mores and folkways are both based on norms and are related to nonmaterial culture. Deviance from mores is sometimes punishable by law, but not always; it depends on the significance of the norm that was violated. Mores are based on values and ethics, whereas folkways are based on social approval.

ATP is generated in the cell by extracting energy from the foods we eat. Different food sources, like carbohydrates and dietary lipids, are broken down in different ways to achieve this goal, via numerous different biochemical pathways which are localized to discrete compartments of the cell. Where within the cell is the majority of ATP produced from the complete metabolism of carbohydrates and lipids, respectively? A. Most of the ATP from the metabolism of carbohydrates is produced in the mitochondria, while most of the ATP from the metabolism of lipids is produced in the cytosol. B. Most of the ATP from the metabolism of carbohydrates is produced in the mitochondria, and most of the ATP from the metabolism of lipids is produced in the mitochondria. C. Most of the ATP from the metabolism of carbohydrates is produced in the cytosol, while most of the ATP from the metabolism of lipids is produced in the mitochondria. D. Most of the ATP from the metabolism of carbohydrates is produced in the cytosol, and most of the ATP from the metabolism of lipids is produced in the cytosol.

B. Most of the ATP from the metabolism of carbohydrates is produced in the mitochondria, and most of the ATP from the metabolism of lipids is produced in the mitochondria. Glycolysis occurs in cytoplasm but only generates two ATP molecules. Lipids, as fatty acids, are metabolized in the mitochondria via beta oxidation to form acetyl-CoA The products of glycolysis and beta oxidation both enter the TCA cycle in the matrix of the mitochondria, where NADH and FADH2 are generated. Mitochondrial NADH and FADH2 enter the electron transport chain and produce ATP within the mitochondria.

The electron transport chain is an important chemical reaction of cellular respiration. What would occur if oxygen was not present in the electron transport chain (anaerobic conditions)? A. The anaerobic reaction would be endergonic with magnitude of ΔG smaller than in the presence of oxygen. B. The anaerobic reaction would be exergonic with magnitude of ΔG smaller than in the presence of oxygen. C. The anaerobic reaction would be exergonic with magnitude of ΔG larger than in the presence of oxygen. D. The anaerobic reaction would be endergonic with magnitude of ΔG larger than in the presence of oxygen.

B. The anaerobic reaction would be exergonic with magnitude of ΔG smaller than in the presence of oxygen. Electron transport chain is characterized by decreasing free energy levels on each step. Oxygen is the final electron acceptor in electron transport chain, its absence would reduce the overall free energy released, thus this exergonic reaction would have a smaller magnitude ΔG in the absence of oxygen.

Which of these statements regarding alcohol withdrawal symptoms is inaccurate? A. Insomnia, anxiety and other symptoms can last for three to six month, after the acute withdrawal stage has been completed. B. Withdrawal symptoms show little improvement until a week after cessation of alcohol consumption, at which point it is at its peak. C. Symptoms can be relieved by the administration of benzodiazepines. D. Typically, withdrawal symptoms begin within 4 to 12 hours after cession of alcohol consumption.

B. Withdrawal symptoms show little improvement until a week after cessation of alcohol consumption, at which point it is at its peak.

Josephine is an American college student who spends her sophomore year studying abroad in France. Her appreciation of French culture is heavily influenced by her experiences with French cuisine. She feels that the French believe it is important for families to take pride in the quality of their food and how it is prepared. She believes that the culture benefits from families eating and enjoying their food together. She ultimately decides French culture is superior to American culture. What term describes Josephine's beliefs? A. Ethnocentrism B. Xenocentrism C. Cultural relativism D. Cultural imperialism

B. Xenocentrism Cultural imperialism is the deliberate imposition of one's own cultural values on another culture. Cultural relativism is the practice of assessing a culture by its own standards rather than viewing it through the lens of one's own culture. Ethnocentrism is judging one's own culture as superior to another culture. Xenocentrism is judging another culture as superior to one's own culture.

Which of the following cells is not capable of entering replicative senescence? A. Skin cells B. Cardiac muscle cells C. Pluripotent stem cells D. Brush border cells

B. cardiac muscle cells Replicative senescence is a state that mitotically active cells enter into when they have reached their Hayflick limit (when their telomeres have become very short). Cardiac muscle cells are not mitotically active (they don't divide).

There are many hypotheses regarding the biological etiology of schizophrenia. One, called the "dopamine hypothesis", asserts that schizophrenia is caused by an abnormality in neurotransmission of dopamine. Which of these statements describes possible limitation to this hypothesis? A. Drugs that increase dopamine levels in the brain make schizophrenia symptoms worse. B. Anti-psychotic drugs take two to three weeks to begin to show efficacy. C. People with schizophrenia have on average twice as many D2 receptors as neurotypical individuals. D. Neuroleptic drugs are used to treat schizophrenia.

B.Anti-psychotic drugs take two to three weeks to begin to show efficacy. The use of neuroleptic drugs, like chlorpromazine, reduce dopamine transmission in the brain lessening some symptoms of schizophrenia. This supports the "dopamine hypothesis". The use of high levels of drugs such as L-Dopa which increase dopamine levels causes schizophrenic-like symptoms. A correlation between an individual having more or less than a normal amount of D2 receptors and schizophrenia supports the "dopamine hypothesis" Anti-psychotic drugs take two to three weeks to begin to show efficacy.

If there is 10 kg of a radioactive isotope with decay rate 0.1 1/s, how much of the isotope will be left in 30 s? A. 9.0 kg B. 5.0 kg C. 0.5 kg D. 0 kg

C. 0.5 kg Several answer choices can be eliminated by considering the properties of the exponential function. The inverse of the decay rate is 10 s, which means that the amount should decrease by several factors of e in 30 s First, write the exponential decay equation N(t) = No*e^-(lamba*t) No= 10 kg lamba= 0.1 1/s t= 30 s N= 10 x e^-(0.1x30) = 10 x e ^ (-3) = 0.5 kg

Which of the following will be a decay product when 23/12 Mg undergoes beta minus decay? A. 24/12 Mg B. 22/12 Mg C. 23/13 Al D. 24/14 Si

C. 23/13 Al Beta minus decay results in the release of an electron and converts a neutron into a proton. Atomic number and identity of element increases by one during beta minus decay.

Which of these theories describes semantic processing as happening after the sensory information is filtered and is therefore unable to account for the 'Cocktail Party Effect'? A. Deutsch and Deutsch's late selection theory B. Johnston and Heinz's multi-mode theory C. Broadbent Early Selection theory D. Treisman's Attenuation theory

C. Broadbent Early Selection theory Deutsch and Deutsch's late selection theory moved the selective filter from before perception to after the perceptual process. Treisman's Attenuation theory replaced the selective filter with an attenuator, which selectively allows the attended message to be processed to a greater extent than the unattended message. Johnson and Heinz proposed that the location of the information attenuator (sometimes described as a bottleneck) was able to be varied by the listener depending on the demand necessitated by a particular attention task. Broadbent's early selection theory is the only theory listed that is unable to account for the 'Cocktail party effect'.

Which of these criteria is NOT necessary for an action to be described as automatic according to Posner and Snyder? A. Other mental activities are not affected by the action. B. The action does not draw conscious awareness. C. Conscious effort is required to initiate action. D. The action occurs without conscious intention.

C. Conscious effort is required to initiate action. Posner and Snyder described an action as automatic if the action did not affect other mental activities. An action is automatic if it occurs without being initiated consciously. Conscious effort is required to initiate action.

Which of these accurately describes the mesolimbic pathway, which is associated with reward and motivation? A. Dopamine, produced in the substantia nigra, is transmitted from neuron soma to axons projecting into the caudate nucleus and the putamen of the neostriatum. B. Dopamine, produced in the ventral tegmental area (VTA), is transmitted from the VTA to the pre-frontal cortex. C. Dopamine, produced in the ventral tegmental area (VTA), is transmitted from the VTA to the nucleus accumbens, the amygdala, and the hippocampus. D. Dopamine, produced in the arcuate nucleus, is transmitted from the hypothalamus to the anterior pituitary gland.

C. Dopamine, produced in the ventral tegmental area (VTA), is transmitted from the VTA to the nucleus accumbens, the amygdala, and the hippocampus. Dopamine, produced in the arcuate nucleus, is transmitted from the hypothalamus to the pituitary gland via the tuberoinfundibular pathway. The dopamine released regulates the secretion prolactin by inhibiting its release in the anterior pituitary. Dopamine, produced in the substantia nigra, is transmitted from neuron soma to axons projecting into the caudate nucleus and the putamen of the neostriatum via the nigrostriatal pathway. This pathway is associated with motor planning and purposeful movement. Dopamine, produced in the ventral tegmental area (VTA), is transmitted from the VTA to the pre-frontal cortex via the mesocortical pathway. This pathway is associated with cognition, affect, and negative symptoms of schizophrenia. Dopamine, produced in the ventral tegmental area (VTA), is transmitted from the VTA to the nucleus accumbens, the amygdala, and the hippocampus. The mesolimbic pathway is associated with reward, motivation, and many of the positive symptoms of schizophrenia.

Which of these subfunctions of attention, modulated by dopamine release, is most affected by diseases such as schizophrenia? A. Vigilance B. Orienting C. Executive attention D. Alerting

C. Executive attention Vigilance (like active search, selective attention, and divided attention) is a type of attention. These types of attention are often described as main functions rather than subfunctions of attention. Alerting attention is affected by regular aging but deficits are not often associated with schizophrenia. Neurotransmitter modulation of this attentional network is associated with norepinephrine produced in the locus ceruleus. Orienting attention involves the capacity to change the focus of attention from one stimulus to another stimulus. This network is predominantly modulated by acetylcholine produced in the basal forebrain. Executive attention is involved in goal-directed behavior, monitoring conflicts between internal processes, and anticipating the effects of behavior. Dopamine from the ventral tegmental area is associated with executing attention.

Which of these is NOT a theory of how odor is coded in the brain? A. Vibrational theory of olfaction B. Labeled-line theory of olfaction C. Gate control theory of olfaction D. Steric theory of olfaction

C. Gate control theory of olfaction The labeled-line theory of olfaction describes a scenario where each receptor would respond to specific stimuli and is directly linked to the brain. The vibrational theory of olfaction asserts that the vibrational frequency of a molecule gives that molecule its specific odor profile. Steric theory of olfaction, or shape theory, asserts that odors fit into receptors similar to a lock-and-key.

A clinical trial is designated to test the efficacy of mindfulness-based stress reduction (MBSR). The participants (diagnosed with agoraphobia based on the DSM-5 criteria) were divided into groups after being matched for age, socioeconomic status, and level of anxiety. Groups received interventions as follows: Group 1: received MBSR training for 6 weeks Group 2: was given a drug intervention (group members were instructed to take "a newly approved and released anxiety drug", actually a sugar pill) for acute anxiety. Group 3: received cognitive behavioral therapy for 6 weeks. Group 4: received a therapy created for the experiment by the researchers. Which group was the Active comparator arm? A. Group 1 B. Group 2 C. Group 3 D. Group 4

C. Group 3 An arm refers to a group of participants in a randomized controlled trial, who are allocated a particular treatment. Most controlled trials have at least two arms, an experimental intervention and at least one control arm. Group 2 would be described as the placebo comparator since only the placebo (a sugar pill) was administered as therapy. Group 1 is the experimental arm, or the group receiving the therapy to be tested (MBSR). Group 4 is the sham comparator. A sham therapy is a mock therapy is a mock therapy, or a treatment created for the experiment, without drugs being administered. Group 3 was the active comparator arm. Cognitive behavioral therapy is widely used to treat patients with agoraphobia.

Narcolepsy is an alertness disorder associated with excessive sleepiness, sleep attacks, and cataplexy. They study of narcolepsy in humans has been informed by the study of Dachsunds, Doberman Pinchers, and Labrador Retrievers. In these dogs, possession of two recessive genes that control a neurotransmitter receptor, which affects the control of sleep and arousal, and results in direct transitions from an alert state to REM sleep. What NT is being described? A. Dopamine B. Serotonin C. Hypocritin/orexin D. GABA

C. Hypocritin/orexin The NT in question is believed to work as an excitatory NT. The NT, GABA, works as the primary inhibitory NT. Dopamine has many functions but is most often associated with reward, learning, and attention. Serotonin also has many functions but is most often associated with mood, appetite, social behavior, and memory. The primary role of hypocritin also called orexin in the CNS is to control sleep and arousal.

When a nucleus undergoes ordinary fission into two daughter nuclei, what happens to the binding energy of the parent nucleus? A. It is fully transferred to the binding energy of the daughter nuclei. B. It is transferred into the excited energy levels of the daughter nuclei C. It partly becomes the kinetic energy of the daughter nuclei D. It is released as a high-energy photon

C. It partly becomes the kinetic energy of the daughter nuclei In ordinary fission, gamma ray photons are not emitted by a decaying nucleus. The reason that fission is energetically favorable is because the sum of the binding energies of the daughter nuclei is less than that of the parent nuclei. During ordinary fission, the electronic energy levels do not strongly play a role in the decay process. In ordinary fission, the binding energy of the parent nucleus is transformed into the kinetic energy of the daughter nuclei.

Steatorrhea is the presence of increased fat in feces. Which organ is least likely to be the cause of a patient's steatorrhea? A. Pancreas B. Small intestine C. Stomach D. Liver

C. Stomach Increased concentration in the feces means that a substance is not being absorbed out of the GI tract. Due to its chemical properties, fat can be hard to absorb, and so must be emulsified. Fat is chemically digested by lipase, which is produced by the pancreas. The stomach plays the least role in fat digestion and absorption, and would be the least likely to cause steatorrhea.

Many factors can affect one's ability to pay attention. Which of these factors would cause the most negative impact on the ability of a driver to react to adverse road conditions, such as a patch of black ice? A. The individual is in an enhanced state of arousal. B. The skill of the driver is high. C. The maneuver required to avoid the obstacle is difficult. D. The driver has practiced evasive maneuvers before.

C. The maneuver required to avoid the obstacle is difficult. Previous practice on any skill will improve performance. Although too high of an arousal state can be detrimental to attention, enhanced arousal can enhance attention. A person who is skilled at a task will require less attention to complete that task. A difficult or novel task will require more attention than a practiced or skilled task.

Which of the following carboxylic acids or carboxylic acid derivatives is the most reactive to a nucleophilic attack at the carbonyl carbon? A. Propionyl chloride B. Ethanoic acid C. Ethanoyl chloride D. Propanoic acid

C. ethanoyl chloride carboxylic acids and carboxylic acid derivatives can undergo a nucleophilic attack at its electron deficient carbonyl carbon. Substituents that withdraw electrons away from this carbonyl carbon will make it more reactive to a nucleophilic attack. Likewise substituents that donate electrons to the carbonyl carbon will make it less reactive to a nucleophilic attack. Longer alkyl substituents will more readily donate electrons than shorter alkyl substituents. This means that carbonyl groups with longer alkyl substituent are less electrophilic and therefore less reactive. Substituents that withdraw electrons away from the carbonyl carbon such as a halide will make the compound more reactive. Since chlorine atoms are more electronegative than oxygen atoms, ethanoyl chloride and propionyl chloride are more reactive than their carboxylic acid counterparts.

Which of the following reactions will undergo an SN2 reaction? A. 2-bromo-2-methylbutane with lithium diisopropylamide B. 2-chloro-3-methylhexane with HCl C. t-butyl bromide with water D. 1-bromobutane with KOH

D. 1-bromobutane with KOH SN2 reactions are preferred in reactants with no steric hindrance A strong nucleophile along with a primary electrophile is indicative of SN2. The reactants 1-bromobutane with KOH are likely candidates to undergo SN2

Suppose that natural nitrogen is found in 70% abundance as the isotope 14/7 N and the rest of the time as 12/7 N. In this scenario, which of the following would give the atomic mass of this element? A. 13.0 amu B. 14.0 amu C. 12.6 amu D. 13.4 amu

D. 13.4 amu .70 x 14 = 9.8 .30 x 12 = 3.6 9.8 + 3.6 = 13.40 amu

Under which of the following conditions would one expect ATP hydrolysis to be non-spontaneous? A. A temperature slightly below 37 deg C B. A very basic environment (high concentration of proton acceptors) C. No enzymes available to lower activation energy D. ATP concentrations very far below equilibrium concentration, and ADP concentrations very far above equilibrium concentration

D. ATP concentrations very far below equilibrium concentration, and ADP concentrations very far above equilibrium concentration A non-spontaneous reaction has a fractional equilibrium constant. A fractional equilibrium constant (one that is less than one but greater than zero) arises when the product of the concentrations of the reactants, raised to the powers of their balanced stoichiometric coefficients, is greater than the concentrations of the products, raised to the powers of their balanced stoichiometric coefficients.

Juxta glomerular cells are part of which structure in the nephron? A. Efferent arterioles B. Proximal convoluted tubule C. Glomerulus D. Afferent arterioles

D. Afferent arterioles Juxta glomerular cells release renin into blood stream. Juxta glomerular cells respond to short range prostaglandin secretion from macula densa cells, which line the distal proximal tubule.

Problem Which of the following decay processes results in the largest change in mass of a nucleus? A. Neutron emission B. Beta decay C. Gamma decay D. Alpha decay

D. Alpha decay Recall what types of particles are released during each process: Alpha decay releases helium nuclei, beta decay releases electrons, gamma decay releases photons, and neutron decay releases neutrons. An alpha particle or helium nucleus consists of 2 neutrons and 2 protons, for a total mass of 4 atomic mass units. Alpha decay results in the largest mass change for a decaying nucleus

Which of the following is not a mechanism for induction of cell differentiation? A. Paracrine signaling B. Formation of gap junctions C. Direct contact D. Autocrine signaling

D. Autocrine Induction occurs when entities external to a cell initiate the process of differentiation. Autocrine signaling refers to cell signaling that occurs when secreted cytokines from a cell interact with receptors on the secreting cell.

To study whether an olfactory receptor neuron is activated by pyradine a researcher employs an imaging technique that utilizes a chemical indicator that glows green when exposed to ultraviolet light. If a decrease in the intensity of glow observed corresponds to an increase in ionic concentration in the olfactory receptor neuron in response to an odorant, which ion would this inidcator chelate? A. Cl- B. K+ C. Na+ D. Ca 2+

D. Ca2+ Extremely versatile since positively charged ion. A rise in this ion, postsynaptically, in dendritic spines is essential for activity-dependent plasticity. This ion is an important second messenger in neuron. Abnormal amounts of signaling in this ion has been implicated in disease states like Huntington's, Alzheimer's, and schizophrenia.

Which of the following has a rate of urinary excretion (units of solute per unit time) that is almost always identical to its rate of glomerular filtration in a healthy adult? A. Glucose B. Na C. Cl D. Creatinine

D. Creatinine The rate of urinary excretion should equal the rate of glomerular filtration for molecules that the body does not want to reabsorb under any circumstance (waste products). Creatinine is a waste product, eliminated by the kidney. Sodium, chloride, and glucose are all molecules that the body may want to eliminate or reabsorb, depending on the situation.

An azeotrope is a mixture of liquids whose proportions cannot be changed by simple distillation because, when it is boiled, the vapor has the same proportions of constituents as the unboiled mixture. From an azeotropic mixture of 95% ethanol and 5% water, which of the following methods would NOT allow for further purification of ethanol at the azeotropic point? A. Addition of calcium oxide to the mixture and then filtration B. Dissolution of salt like potassium acetate into mixture and the distillation C. Addition of an additional agent like cyclohexane to the mixture to form a ternary azeotrope and then distillation. D. Distillation conducted in all-nitrogen gas (N2) environment

D. Distillation conducted in all-nitrogen gas (N2) environment Azeotropes are mixtures that cannot be separated by simple distillation techniques. Generally additional separation techniques must be applied. Adding another agent to create a ternary azeotrope coupled with further distillation would result in greater purification of ethanol. Adding calcium oxide in water would create insoluble calcium hydroxide. Calcium oxide is essentially a dessicant, and what is left is pure ethanol. Dissolving a salt will increase the boiling point or decrease the volatility of the solvent, namely water. Further purification of ethanol can be achieved. Nitrogen is extremely inert, and distillation conducted in an all-nitrogen gas environment would not allow for greater purification of the mixture.

At which point in development do we start to observe a significant increase of the size of embryo? A. During organogenesis B. During fertilization C. During early cleavage D. During blastulation

D. During blastulation Organogenesis is not the first observable growth of the embryo. The sperm is so small in relation to the egg that fertilization does significantly increase the size of the embryo. During cleavage cells get smaller after each division, thus during blastulation is when we begin to observe the embryo growing in size.

Hallucinogen Persisting Perception Disorder is characterized by the re-experiencing of altered perceptions after the cessation of hallucinogen use. These symptoms continue either continuously or episodically for weeks after hallucinogen intoxication and cause significant distress or impairment in daily activities. Which of these drugs would NOT cause this perceptual disorder upon cession of use? A. Mescaline B. Marijuana C. LSD D. Haloperidol

D. Haloperidol LSD is the prototypical hallucinogen and most often associated with this disorder. Mescaline is a psychedelic that acts on norepinephrine and is a possible cause of this disorder. Marijuana is often classified as a psychedelic and is described as a possible cause of this disorder. Haloperidol is an antipsychotic medication used to treat schizophrenia.

S-Adenosyl methionine (SAM) is a molecule utilized in various metabolic pathways to transfer methyl groups from SAM to an acceptor. What is the proper designation of the complex of SAM and its enzyme together? A. Prosthetic group B. Coenzyme C. Cofactor D. Holoenzyme

D. Holoenzyme A cofactor is non-protein chemical compound bound to an enzyme to assist in catalysis. A coenzyme is a specific type of cofactor that carries chemical groups between enzymes, like methyl groups A holoenzyme describes the complex of an enzyme that has its cofactor bound and is considered 'complete'.

Increased stomach pH is the consequence of which of the following? A. Increased G cell activity B. Damage to the mucous cells C. Overexpression by chief cells D. Inhibition of parietal cells

D. Inhibition of parietal cells pH increases as solution becomes less acidic and more basic. Mucous helps protect the stomach from acid damage, but does not change the pH. Parietal cells secrete hydrochloric acid, which increases the [H+] of the stomach. Inhibiting these cells would increase the pH.

Macula densa cells sense concentrations of which of the following? A. Human growth hormone B. ADH C. ANP D. Na2+

D. Na2+ Macula densa cells (which form the lining of the distal convoluted tubule) are part of the juxtaglomerular apparatus, which regulates blood pressure. A component of blood pressure is blood volume: higher blood volume means higher blood pressure. A decrease in Na ion in the distal convoluted tubule implies low blood pressure.

Potassium cyanide is a poison which combines with cytochrome a3 to prevent binding of oxygen to the enzyme without altering the Km of the reaction with respect to reduced cytochrome c. Which type of inhibition does this represent? A. Uncompetitive inhibition B. Competitive inhibition C. Reversible inhibition D. Noncompetitive inhibition

D. Noncompetitive inhibition In competitive inhibition, the Km of the reaction changes, while the Vmax will stay constant with increased amounts of inhibition. In uncompetitive inhibition, both the Km and Vmax will decrease with increased amounts of inhibition. In noncompetitive inhibition, the Km of the reaction stays constant while the Vmax will decrease with increased amounts of inhibition. Therefore inhibition that does not alter Km must not be noncompetitive inhibition.

Following the passing of a controversial law by a local leader, a crowd of individuals gathers and begins to act violently. The crowd destroys property, engages in theft, and sets fires in local business places. The crowd is fueled by a deep feeling that an injustice has occurred, but their behavior is unfocused, violent, and random. What collective behavior does this describe? A. Bystander effect B. Mob C. Mass hysteria D. Riot

D. Riot Mass hysteria refers to behavior that occurs when groups react emotionally or irrationally to real or perceived threats. It is characterized by panic and spread of information (or misinformation) by the media. A mob is a group of individuals who are emotional and violent, but target specific individuals or categories of individuals. A riot is a violent form of crowd behavior that results from feelings of injustice or feeling that needs have been ignored. Riots typically result in property damage and other significant crimes.

Which answer describes the correct pathway that a signal takes from the olfactory mucosa to the frontal lobe of the brain? A. The pathway for olfaction goes from the olfactory bulb to the thalamus and the piriform cortex. Form there the signal is transmitted to the insula. B. The pathway for olfaction goes from the olfactory bulb to the amygdala and the piriform cortex. From there the signal is transmitted to the insula. C. The pathway for olfaction goes from the olfactory bulb to the thalamus and the piriform cortex. From there the signal is transmitted to the orbitofrontal cortex. D. The pathway for olfaction goes from the olfactory bulb to the amygdala and the piriform cortex. From there the signal is transmitted to the orbitofrontal cortex.

D. The pathway for olfaction goes from the olfactory bulb to the amygdala and the piriform cortex. From there the signal is transmitted to the orbitofrontal cortex. Vision, taste, olfaction, and touch are all first integrated in the orbitofrontal cortex. Olfaction, unlike the other sense, does not synapse in the thalamus before higher processing.

Dr. Soreth is a behavioral researcher who uses pigeons in her research; she teaches the birds to peck a green button and receive a treat. She wants to try out a new schedule of reinforcement for the pigeons. She wants the birds to peck as much as possible, and to have their pecking behavior occur at a steady rate with very little pausing between responses. Which schedule of reinforcement would be best for her to choose? A. Fixed interval B. Variable interval C. Fixed ratio D. Variable ratio

D. Variable ratio A fixed ratio schedule is one in which reinforcement is given after a specific number of responses are completed. A fixed interval schedule is one in which reinforcement is given after a specific amount of time has passed. A variable interval schedule is one in which reinforcement is given after random amounts of time have passed. A variable ratio schedule is one in which reinforcement is given after random numbers of responses are completed. It produces the highest, steadiest rate of responding with short pauses after delivery of the reinforcers.

Consider the whole gestational period. When does fertilization normally occur? A. Week 1 of gestation B. At the very beginning of gestation C. Prior to gestation occur D. Week 2 of gestation

D. Week 2 of gestation Gestation begins with the uterus getting ready for implantation. Fertilization occurs after the beginning of gestation.

Which of the following strategies describes how a person would extinguish a response using principles of operant conditioning? A. Keep subject from being exposed to the stimulus for an extended time B. Present the unconditioned stimulus without the conditioned stimulus C. Present the conditioned stimulus without the unconditioned stimulus D. Withhold all reinforcement after responses to the stimulus

D. Withhold all reinforcement after responses to the stimulus Extinction refers to the gradual weakening of a conditioned response. Pairing of stimuli is a feature of classical conditioning. If a person learned a behavior via operant conditioning, they would likely receive rewards after each behavioral response. If the rewards were no longer given, the behavior would decrease and eventually stop.

Which of the following represents the numerator for the expression used to determine the equilibrium constant for the hydrolysis of ATP to yield AMP and 2 Pi? A.[AMP]×2[Pi] B. [ATP] x [H20] C. [ AMP] + [Pi]^2 D. [AMP] x [Pi]^2

D. [AMP] x [Pi]^2 The equilibrium constant is defined as the ratio of the concentrations of the products of a reaction to the concentrations of the reactants, each raised to the power of their stoichiometric coefficients in the balanced equation for the reaction. The products of the reaction in question are one equivalent of AMP and two equivalents of Pi

A tetrapeptide Tuffsin has the following amino acid sequence: Thr-Lys-Pro-Arg. Tuftsin binds to specific receptors on the surface of macrophages stimulating their migration and phagocytic activity. Proteins can be separated electrophoretically on the basis of net charge by isoelectric focusing in a pH gradient. At pH=6, what is the overall charge of the tetrapeptide and the direction of travel in the gel? A. overall charge of +3 that travels to the anode. B. overall charge of -1 that travels to the anode. C. overall charge of +1 that travels to the cathode D. overall charge of +2 that travels to the cathode

D. overall charge of +2 that travels to the cathode The tetrapeptide has two basic amino acids: lysine and arginine. The other two, proline and threonine, do not have R groups to consider.

The lungs, spine, and muscles are derived from 3 different germ layers. What would be the correct order of formation of the embryological cells from which these tissues are derived from? A. spine then muscles then lungs B. muscles then lungs then spine C. muscles then spine then lungs D. spine then lungs then muscles

D. spine then lungs then muscles first ectoderm then endoderm then mesoderm lungs--> endoderm spine--> ectoderm muscles --> mesoderm

The products of the liver and pancreas participate in what type of digestion? A. Mechanical and chemical, respectively B. Both mechanical C. Both chemical D. Chemical and mechanical, respectively

A. Mechanical and chemical, respectively Mechanical digestion separates molecules of a substance, without changing its chemical properties, and is also known as physical digestion. Chemical digestion breaks intramolecular bonds and changes the "identity" of a substance. Bile from the liver is involved in emulsification of fats, a mechanical process, while pancreatic enzymes chemically cleave apart macromolecules.

Which of the following describes a general feature of proteins that makes them well suited to be enzymes? A. Proteins have large variability in structure, allowing for lock-and-key specificity with different substrates. B. Proteins are defined by their amphipathic nature, which allows them to assemble into membranes that isolate substrates from the watery environment of the cell. C. Proteins cannot be denatured even in extreme environmental conditions. D. Proteins have high regularity in structure, allowing for lock-and-key specificity with different substrates.

A. Proteins have large variability in structure, allowing for lock-and-key specificity with different substrates.

A former college athlete who currently works in advertising demonstrates a preference for hiring athletic people. This could be construed as: A. bias B. prejudice C. stereotyping D. nepotism

A. bias

When action potential induces muscle contraction, which of the following occurs? A. Ca is released into cytosol; Ca binds to tropomyosin B. Ca is released into the cytosol; Ca binds to troponin complex C. Ca is pumped into the SR; Ca binds to tropomyosin D. Ca is pumped into the SR; Ca binds to troponin complex

B. Ca is released into the cytosol; Ca binds to troponin complex An AP resulting in muscle contraction causes t-tubule depolarization, followed by SR release of Ca in order for Ca to bind to the troponin complex, thereby allowing tropomyosin to expose the myosin binding sites on actin.

Which of the following statements is (are) true concerning separations? I. Any two compounds with sufficiently different boiling points can be separated completely by distillation II. Crystallization is an efficient method of compound purification for most compounds III. Distillation is most effective when it is carried slowly A. I only B. III only C. I and III only D. II and III only

B. III only II is false because crystallization is generally not an efficient method of purification. I is false because distillation is generally effective, but it cannot completely separate two compounds.

Which of the following is true concerning the nucleolus? A. It is bound by a phospholipid membrane B. It disappears during prophase C. It is the site of translation of ribosomal RNA D. It is found in most bacteria

B. It disappears during prophase

Most steroid hormones regulate enzymatic activity at the level of: A. replication B. transcription C. translation D. the reaction

B. transcription Steroids act at the level of transcription by regulating the amount of mRNA transcribed. Because steroids diffuse into the cell and head for the nucleus, choice C can be eliminated -translation happens in the cytoplasm, not the nucleus. Although some steroid hormones have an effect on cellular growth, they don't all control replication. In a cell, reactions are typically regulated by controlling relative concentrations of the enzyme and/or inhibitors (i.e. through regulation of transcription or translation).

A child playing in the sink empties a short, wide glass of water into a tall, thin glass of equal volume. Questioning revealed that the child believed the tall glass contained more water. This indicates that she has not yet reached which of Piaget's stages? A. Sensorimotor B. Preoperational C. Concrete operational D. Logical operational

C is correct. The situation described illustrates a failure to understand conservation, the notion that a quantity remains unchanged despite changes of its shape or container. Understanding of conservation is one of the hallmarks of the concrete operational phase of development, making choice C the correct answer.

Lactose intolerance is the inability to digest lactose, due to insufficient amounts of the enzyme lactase. Where is lactase usually found? A. Brush border of the jejunum B. Lumen of the small intestine C. Brush border of duodenum D. Lumen of the stomach

C. Brush border of duodenum Carbohydrate digestion takes place in the oral cavity and small intestine. The duodenum is the primary site of digestion, while the jejunum is the primary site of absorption.

Which of these would describe a person utilizing crystallized intelligence, a concept of intelligence proposed by Cattell and believed to peak in late adulthood? A. Through practicing tasks that increase working memory, John improves speed that he is able to identify patterns in number sequences. B. Cynthia practices a video game that gives her logic puzzles that must be solved by a specific set of rules. C. Daniel plays crosswords every morning to learn new facts and improve his acquired knowledge base. D. Beth plays cognitive games that require her to identify objects and organize them spatially by relevant features.

C. Daniel plays crosswords every morning to learn new facts and improve his acquired knowledge base. Cattell defined fluid intelligence as: in novel situations, the ability to recognize and reason relationships between objects or ideas independent of previous experience. Cattell defined crystallized intelligence as the ability to retrieve and acquire knowledge. Fluid intelligence helps one see patterns, organize and identify feature and spatial relationships to solve complex problems. Crystallized intelligence is based on fact, experience, prior learning and accumulates as one ages. Daniel plays crosswords every morning to learn new facts and improve his acquired knowledge base.

Central tolerance refers to which of the following processes? A. Destruction of self-antigens that are specific for lymphocytes B. Destruction of lymphocytes that are not specific for self-antigens C. Destruction of lymphocytes that are specific for self-antigens D. Production of self-antigens that are specific for lymphocytes

C. Destruction of lymphocytes that are specific for self-antigens Central tolerance is the process whereby the body destroys immune cells that are reactive with antigens present on the bodies own cells. "Self-antigens" are the antigens derived from the bodies own cells.

The synthesis of which of the following would be LEAST affected by a disease which selectively destroys the smooth ER? A. Testosterone B. Phospholipids C. Insulin D. Cholesterol

C. Insulin The smooth ER is responsible for many functions, including the production of various lipid products. Also responsible for the production and degradation of steroid hormones such as testosterone. Key steps in synthesis of cholesterol occur within the smooth ER. Insulin is a peptide hormone, which is a product of the translation of mRNA, which occurs in the cytoplasm. The synthesis of insulin would be least affected by a disease which destroys the smooth ER, but would be affected if the rough ER were affected.

All of the following are composed of microtubules EXCEPT: A. the tail of a sperm cell B. the spindle apparatus C. It is the site of translation of ribosomal RNA D. It is formed in most bacteria

C. It is the site of translation of ribosomal RNA

Suppose that a penguin is fed a fish whenever it waves its fin at an experimenter. Over time, the penguin increases the frequency of fin waving. This situation is best described as which of the following? A. Classical conditioning B. Punishment C. Reinforcement D. Observational learning

C. Reinforcement

Anxious about a nagging illness, a patient feels ignored by a doctor who is struggling to catch up with patient examinations on a very busy day. The doctor misinterprets the discomfort and agitation of the patient as hostility. Which sociological paradigm can best explain this scenario? A. Functionalism B. Conflict Theory C. Symbolic Interactionism D. Social Constructionism

C. Symbolic Interactionism The scenario concerns the inter-subjective negotiation of symbols or meanings, which is indicative of symbolic interactionist theory.

Which of the following is considered necessary as part of standard state conditions? A. 1 Molar for all solutions, including water B. pH 7 for all solutions C. ambient pressure of 100,000 Pascals D. ambient temperature of 273 K or 310 K

C. ambient pressure of 100,000 Pascals Some of us may be more familiar with seeing 1 atmosphere, but IUPAC does recommend a standard pressure of 10^5 Pascals for standard state conditions. Since 10^5 is 0.99 atmospheres, they are essentially the same.

The most reactive of the four derivatives would be the: A. Amide B. Ester C. Anhydride D. Acid Chloride

D. Acid Chloride Cl- is the best leaving group of the list, thus acid chlorides are predicted to be the most reactive.

Positron Emission Spectroscopy (PET) imaging involves injecting a patient with a positron (anti-electron) emitting isotope. The emission of positrons occurs during: A. Alpha decay B.Alpha absorption C. Gamma absorption D. Beta decay

D. Beta decay Beta decay is the decay in which a beta particle (electron or positron) is emitted from a heavy-atom nucleus.

Which of the following structures is (are) found in prokaryotes? I. A cell wall containing peptidoglycan II. A plasma membrane lacking cholesterol III. Ribosomes A. I only B. II only C. I and II only D. I, II, and III

D. I, II, and III

Which hormonal supplementation therapy is the least likely to assist a female that has difficulties ovulating? ALuteinizing hormone B. Follicle stimulating hormone C. Estrogen D. Progesterone

D. Progesterone Estrogen levels must rise before ovulation Follicle stimulating hormone levels must rise before ovulation Luteinizing hormone levels must rise before ovulation. Progesterone levels rise after ovulation already occurred.

Relative to sigma bonds, pi bonds exhibit which of the following characteristics? I. Higher energy II. Increased stability III. Greater strength A. I only B. I and II only C. II and II only D. I, II, and III

A Compared to e- that form sigma bonds, those that form pi bonds inhabit higher energy orbitals. Therefore, I is correct. This higher energy is associated with decreased stability and lesser strength. Although a pi bond is weaker than a sigma bond, a double bond is still stronger than a single bond as it contains both a sigma and pi bond, rather than just the sigma bond found in a single bond.

What is the approximate total yield of ATP that is produced via the Krebs cycle and subsequent oxidative phosphorylation of a single molecule of acetyl CoA? A. 10 B. 32 C. 20 D. 16

A. 10 The energy needed to synthesize one molecule of ATP requires four protons. Under ideal conditions, ten protons are shuttled into the intermembrane space per NADH, and 6 per FADH2. A single molecule of acetyl CoA yields one ATP, three NADH, and one FADH2. 3((10 protons/NADH)*(1 ATP/4 protons)) + ((6 protons/FADH2)*(1 ATP/4 protons)) + 1 ATP = 10 ATP

The principal quantum number is a measure of which of the following? A. Approximate radial size of an electron cloud B. Approximate shape of an electron cloud C. Number of valance electrons that orbit a nucleus D. Number of protons and neutrons found in the nucleus of an atom

A. Approximate radial size of an electron cloud n is most closely associated with the potential energy of the electron. Since potential energy is proportional to the square of the distance of two oppositely charged particles by Coulomb's Law, it is also true that n is associated with the radial "size" of the electron cloud.

A blacksmith apprentice notices that all other blacksmith apprentices wear their aprons untied, so she decides to leave her own apron untied. Which of the following phenomena does this situation best demonstrate? A. Conformity B. Obedience C. Compliance D. Assimilation

A. Conformity

What is the function of the CEREBELLUM? A. Coordinated movement B. Complex, perceptual, cognitive, and behavioral processes C. Sensorimotor reflexes D. Sensory relay station

A. Coordinated Movement

After using the Sanger (chain-termination) method of DNA sequencing, a researcher utilizes gel electrophoresis and obtains a single dark band. Which of the following sources of error is most likely? A. Dideoxynucleotides were omitted from the reaction mixture B. The DNA was denatured into single strands using heat C. Deoxynucleotides were present in excess D. The primer annealed to the template strand in numerous places.

A. Dideoxynucleotides were omitted from the reaction mixture In the absence of dideoxynucleotides, DNA polymerase will replicate the entire template and thus produce complete copies--all of the same length.

Which of the following correctly describes the relationship between an empirical formula and a molecular formula? A. Each subscript is equal than or greater to its counterpart in the empirical formula. B. There are an infinite number of possible empirical formulas for a given molecular formula. C. The two formulae are always identical D. At least one subscript of the empirical formula always equals one.

A. Each subscript is equal than or greater to its counterpart in the empirical formula. The empirical formula is the simplest integer number of molecules that captures the molar ratio present in a sample of a compound. This does not mean that the formula reduces until one of the subscripts is one. For example, C3D4H2 is a valid empirical formula. The molecular formula can reduce to the empirical formula, but not vice versa. For the molecular formula, each subscript is equal or greater to its counterpart in the empirical formula.

What is the function of the LIMBIC SYSTEM? A. Emotion and memory B. Arousal and alertness C. Vital function (breathing, digestion) D. Hunger and thirst; emotion

A. Emotion and Memory Limbic system comprises a group of interconnected structures looping around central portion of brain and is primarily associated with emotion and memory. Its primary components include septal nuclei (responsible for feelings of pleasure), amygdala, and hippocampus (learning and memory processes)

Which statement concerning the glycolytic and gluconeogenic pathways is correct? A. Gluconeogenesis is anabolic and glycolysis is catabolic B. Both are catabolic C. Gluconeogenesis is catabolic and glycolysis is anabolic D. Both are anabolic

A. Gluconeogenesis is anabolic and glycolysis is catabolic Glycolysis is the breakdown of glucose; gluconeogenesis is the synthesis of glucose. Catabolism can be thought of as the breakdown of larger molecules into smaller ones, while anabolism can be thought of as the opposite.

Captive mammals and birds that are forced to dive display an overall decrease in metabolism. As a result of such dives, the concentration of which of the following is likely to be elevated? A. Lactate dehydrogenase B. Pyruvate dehydrogenase C. Alcohol dehydrogenase D. Isocitrate dehydrogenase

A. Lactate dehydrogenase Animals with reduced metabolism and reduced access to oxygen, anaerobic metabolic mechanisms will tend to dominate. In animals, anaerobic metabolism (fermentation) produces lactic acid, which is toxic. Therefore, such animals would be expected to have higher concentrations of enzymes that process lactic acid. B and D both participate in aerobic cellular respiration, while C breaks down ethanol (the product of fermentation in yeast).

What is the smallest meaningful unit of a word? A. Morphemes B. Syntax C. Phonemes D. Semantics

A. Morphemes Syntax describes how words are arranged to create grammatically correct sentences. Semantics are the broad meanings of each word, phrase, sentence, or text. Morphemes are the smallest significant unit of meaning in a word.

We would expect to find smooth muscle in all of the following EXCEPT: A. Myocardial tissue B. Pulmonary circulatory tissue C. Esophagus D. Small intestine

A. Myocardial tissue A general rule of thumb is that smooth muscle is often found in hollow organs, like stomach and intestine. This leaves us with pulmonary circulation. Recall that smooth muscle also has a role in regulating how dilated blood vessels are. Cardiac tissue is made up of cardiac muscle, not smooth muscle.

Which of the following most likely represents the correct order of ion size from greatest to smallest? A. O2-, F-, Na+, Mg2+ B. Mg2+, Na+, F-, O2- C. Na+, Mg2+, O2-, F- D. Mg2+, Na+, O2-, F-

A. O2-, F-, Na+, Mg2+ This is an isoelectric series, so the number of electrons on each ion is the same. In this series of ions, the nuclear charge increases with increasing atomic number and draws the electrons inward with greater force. The ion with fewest protons produces the weakest attractive force on the electrons and has the largest size. Oxygen has the smallest atomic number, and thus the fewest protons; therefore, it will have the greatest ion size.

Which of the following statements best describes how parathyroid hormone (PTH) affects the serum concentration of calcium, [Ca2+]? A. PTH stimulates osteoclasts, increasing serum [Ca2+] B. PTH stimulates osteoblasts, decreasing serum [Ca2+] C. PTH stimulates osteoblasts, increasing serum [Ca2+] D. PTH stimulates osteoclasts, decreasing serum [Ca2+]

A. PTH stimulates osteoclasts, increasing serum [Ca2+] Osteoblasts synthesize hydroxyapatite, the mineral component found in bones. Hydroxyapatite synthesized by osteoblasts contains calcium, so stimulating osteoblasts removes calcium from the bloodstream. You can eliminate one of the answers using this information. Osteoclasts secrete a substance called tartrate resistant acid phosphatase, which breaks down bone tissue and thereby add calcium to the bloodstream. This information should allow you to eliminate another answer. One of the main effects of PTH is bone resorption.

One method of isolating polypeptides and proteins from aqueous extracts is freeze drying. The aqueous solution of the polypeptide or protein is frozen. What procedure can be used to remove the water from the frozen sample? A. Sublimation under reduced pressure B. Distillation using steam C. Extraction with organic solvent D. Addition of magnesium sulfate

A. Sublimation under reduced pressure Only sublimation of water under reduced pressure will keep the mixture cold and will maintain the protein in its native state.

Which of the following statements regarding transcription is FALSE? A. The mRNA produced is complementary to the coding strand B. Activators and repressors bind to DNA and affect the activity of RNA polymerase C. The nucleotide sequence of the promoter region affects how often a region of RNA is transcribed D. During transcription, RNA polymerase moves along the DNA strand in the 3' to 5' direction

A. The mRNA produced is complementary to the coding strand Both the mRNA produced through transcription and the coding strand are complementary to the template strand, making choice A false and the correct answer. Choices B, C, and Dare all true. Activators and repressors bind to the DNA close to the promoter region and either activate or repress the transcription activities of RNA polymerase. The nucleotide sequence of the promoter region affects the likelihood and frequency of transcription - sequences which more exactly estimate the consensus sequence are more likely bind RNA polymerase and are more likely to be transcribed. During the elongation phase of transcription, the RNA polymerase moves along reading the antisense DNA strand in the 3' -> 5' direction and synthesizes the new mRNA strand in the 5' - >3' direction.

Consider the synthesis of methyl benzoate from benzoic acid and methanol, catalyzed by an acid chloride. How would this synthesis be affected if a carboxylic acid were used instead of the acid chloride? A. The reaction can occur but will require an acid catalyst B. The reaction can occur because carboxylic acids are more reactive than acid chloride C. This reaction cannot occur because the carboxylic acid cannot undergo substitution D. The reaction cannot occur because the carboxylic acid will react immediately with the water in the solvent

A. The reaction can occur but will require an acid catalyst Acid chlorides are more reactive than carboxylic acids due to chloride atom's electron withdrawing nature, making the carbonyl carbon more electrophilic. Carboxylic acid can undergo substitution. The carbonyl carbon is still relatively electrophilic. The carboxylic acid will react more readily with methanol than with water. Alcohols are more basic than water. The reaction will still yield methyl benzoate. However, carboxylic acids are less reactive than acid chlorides so an acid catalyst is needed.

Like cellulose, chitin is a polysaccharide that cannot be digested by animals. Chitin differs from cellulose in that it possesses an acetyl-amino group at the second carbon. What molecule is a reactant in the breaking of the beta-1,4 glycoside linkages of cellulose and chitin? A. Water B. Oxygen C. alpha-1,4 glucosidase D. beta-1,4 glucosidase

A. Water

In an asthma attack, a patient suffers from difficulty breathing due to constricted air passages. The major causative agent is a mixture of leukotrienes called slow-reacting substance of anaphylaxis. During an asthma attack, slow reacting substance of anaphylaxis most likely causes: A. smooth muscle spasms of the bronchioles B. cartilaginous constriction of the trachea C. edema of the alveoli D. skeletal muscle spasms in the thorax

A. smooth muscle spasms of the bronchioles Constricted air passages is the key clue. The bronchioles are surrounded by smooth muscle and small enough to constrict. Cartilage does not constrict, while muscle does. The skeletal muscle in the thorax does not constrict the air passages. The alveoli are not part of the air passages.

The capillary network comprises the greatest cross-sectional area of blood vessels in the body with high resistance to blood flow. In a healthy individual, the highest blood pressure would most likely be found in: A. the aorta B. the venae cavae C. the systemic capillaries D. the pulmonary capillaries

A. the aorta The not-so-subtle physics reference is a trick. Bernoulli's equation, which would indicate a greater pressure at the greater cross-sectional area, doesn't work here. The blood pressure in a human is greatest in the aorta and drops until the blood gets back to the heart. The longer the blood has travelled since leaving the heart, the lower the pressure at that point.

Which of the following best describes a serial information processing model of cognition? A. A child simultaneously hears a dinner bell and recognizes that it is dinner time, decides to walk home, and then walks home. B. A child hears a dinner bell, recognizes that the bell signifies that dinner is to be served, and then walks home C. A child hears a dinner bell and then walks home D. A child thinks of cereal and then walks home

B is correct. The serial information processing model of cognition posits that the brain first receives a stimulus, then processes the stimulus, and then selects an output function. This is best described in choice B, where the child hears the dinner bell (stimulus), recognizes its significance (processing), and then walks home (output).

A group of researchers performed an experiment to determine the difference threshold for noise frequency in mice that had been fear conditioned using the experimental noise. After analyzing the results, they calculated a Weber fraction of 1/8, making the fear conditioned mice twice as sensitive to frequency changes as the control mice. If the experiment began with a device playing the noise eight times every second (8Hz), what frequency change would be required for the control mice to be able to detect the change? A. 1 Hz B. 2 Hz C. 4 Hz D. 7 Hz

B. 2 Hz the fear conditioned mice had a Weber fraction of 1/ 8, meaning that the stimulus had to change by at least one eighth of its original value for the change to be detected. Since the question asks about the control mice, it is also important to note that they are only half as sensitive to changes as the conditioned mice, as stated in the question stem. In other words, the control mice will need twice as large a stimulus change in order for the change to be perceived, and their Weber fraction should be twice as large: %. If the starting frequency is eight times per second (8Hz), the control mice should need a change of 25% *8Hz, or 2Hz, in order for perception to occur.

If you know that an electron is inside a 25 cm wide X-ray radiography machine what is the error on the most precise measurement that you could make of its momentum? Planck's constant is 6.6 X 10^{-34} m^2*kg/s. A. 4 X 10^{-14} kg x m/s B. 2 X 10^{-34} kg x m/s C. 2 X 10^{-14} kg x m/s D. 4 X 10^{-34} kg x m/s

B. 2 X 10^{-34} kg x m/s This is an application of Heisenberg's uncertainty principle. The uncertainty of the position is determined by knowing that the electron is somewhere inside the 25 cm wide machine, so x=0.25 m. The minimum uncertainty is met when the inequality in the Heisenberg uncertainty inequality becomes an equation: xp > h/4pi. Solving the equation for p, we have p= h/4pi * x = 2 x 10^-34 kg*m/s, the most precise measurement for the electron's momentum when it is inside the radiography machine. .

When an acyl halide reacts with a primary alcohol, which of the following will form? A. An ester B. An ether C. A secondary alcohol D. A carboxylic acid

B. An ester The carbon of the acyl halide acts as the electrophile; it is having electron density pulled away by both the carbonyl oxygen and the halogen. Alcohol, with its lone pairs of electrons on oxygen, can then attack, pushing up electrons onto the carbonyl oxygen. Those electrons reform a double bond kicking off the halide, which is a good leaving group. Lastly, the oxygen from the alcohol group loses its proton and positive charge and we are left with an ester.

Game theory would NOT be well suited to which of the following situations? A. Deciding the optimal bidding strategy at an auction B. Choosing the ideal car purchase given a matrix of advantages and disadvantages for each C. Determining foreign policy decisions D. Selecting the best play to run in a football game

B. Choosing the ideal car purchase given a matrix of advantages and disadvantages for each

Which of the following accurately describes a major difference between the theories of development proposed by Freud and Erikson? A. Freud posits distinct stages of development, while Erikson's theory involves continuous development B. Freud focuses on early life, while Erikson examines entire lifespan C. Freud emphasizes only internal psychological factors, while Erikson posits that only social influences are significant D. Freud's theory is based on psychosocial development, while Erikson's theory is based on psychosexual development

B. Freud focuses on early life, while Erikson examines entire lifespan Freud and Erikson both developed theories that included distinct stages rather than continuous development, eliminating choice A. Choice C is reasonable except for the use of "only"; it's true that Freud emphasized internal factors while Erikson's theory focused more on social factors, but neither of them completely discounted personal or social factors. Choice D flips the two theories. Choice B correctly points out that Freud's theory was mainly concerned with early life, while Erikson's theory extended across the entirety of an individual's lifetime

When hormone-sensitive lipase is active during the fasting state, which of the following processes will not be active in a muscle cell? A. Glycolysis B. Gluconeogenesis C. The citric acid cycle D. The electron transport chain

B. Gluconeogenesis Gluconeogenesis occurs in the liver, not in muscle. The activation of hormone-sensitive lipase indicates that glucose produced by the liver will be preferentially used by the brain and red blood cells, but muscle has its own store of glycogen which it can use. This glycogen will be broken down into glucose, which can then enter glycolysis. Both the pyruvate produced by glycolysis and the acetyl-CoA from fatty acids can enter aerobic respiration.

A second degree burn can result in a loss of sensation, but do not necessarily affect motor control of arrector pili muscles. Which of the following are damaged in a second degree burn? A. Ruffini's endings B. Hair follicle receptors C. Meissner's corpuscles D. Pacinian's corpuscles

B. Hair follicle receptors Pacinian's corpuscles respond to deep touch. Arrector pili muscles control hair follicle movement. Meissner's corpuscles respond to light touch and are found at the border of the epidermis and dermis - they will be damaged in any second degree burn.

If the cytoskeletal model of cell movement is true, which of the following would one not expect to find in a motile macrophages as compared to stationary macrophages, assuming all else to be equal? A. Increased levels of mRNA B. Increased levels of DNA polymerase activity C. Increased levels of ribosome activity D. Increased levels of gene transcription

B. Increased levels of DNA polymerase activity. The cytoskeletal model of cell movement holds that cells move by synthesizing actin chains. Actin is a protein. One would not expect to find increased levels of DNA polymerase activity in motile vs stationary macrophages, since DNA polymerase is only active during cell division (don't confuse DNA polymerase with RNA polymerase).

Very-long chain fatty acids (VLCFAs) are unique among lipids in that they must be degraded within peroxisomes. Disorders such as adrenoleukodystrophy prevent this peroxisomal degradation, and result in an accumulation of VLCFAs throughout the body. Similarly, long-chain fatty acids (LCFAs) are known to accumulate in the setting of a deficiency of the enzyme carnitine acyltransfersase (CAT). If CAT were restored to normal function, where within the cell would LCFAs be metabolized? A. Nucleus B. Mitochondria C. Peroxisome D. Lysosome

B. Mitochondria No fatty acid degradation occurs in nucleus. Lysosomes are more of a 'trash can' for the cell, not a metabolic organelle. Carnitine is required for shuttling LCFAs into the organelle where they are oxidized to produce ATP and acetyl CoA.

Which of the following functions is LEAST regulated by a structure within the brain? A. Involuntary breathing B. Reflexive motor responses C. The vestibular sense D. Blood pressure

B. Reflexive motor responses Motor reflexes are caused by interactions between the peripheral NS and the spinal cord.

The phenomenon known as "goosebumps" is often combined with shivering to reduce heat loss. What integumentary layer is most directly related to this reflex? A. Papillary dermis B. Reticular dermis C. Hypodermis D. Epidermis

B. Reticular dermis Arrector pili muscles attach to hair follicles to control individual hair movement. The hypodermis is an insulating layer, mostly comprised of fat deposits. Partial thickness second degree burns may leave the goosebump mechanism intact. The reticular dermis is the location of arrector pili muscles, anchored in the dense connective tissue that characterizes this layer.

What is the function of the BASAL GANGLIA? A. Vital function (breathing, digestion) B. Smooth Movement C. Arousal and alertness D. Coordinated Movement

B. Smooth Movement Found in the middle of brain: the basal ganglia coordinate muscle movement as they receive information from the cortex and relay info to the brain and spinal cord. Helps make movement smooth and posture steady. Parkinson's Disease is associated with destruction of portions of basal ganglia. Basal ganglia could be related to Schizophrenia and OCD.

Which of the following is NOT innervated by the autonomic nervous system? A. Branching muscle cell B. Striated muscle cell C. Myocardial cell D. Intestinal muscle cell

B. Striated muscle cell Branching and myocardial cells are one and the same, and are innervated by the autonomic nervous system. Smooth muscle in the intestine is also innervated by the autonomic nervous system. Striated cells, aka skeletal muscle, are under voluntary control and therefore innervated by the somatic nervous system.

What does it mean to say that ATP hydrolysis is a spontaneous chemical reaction? A. That the reaction does not require an input of energy to proceed B. That the products of the reaction have lower free energy than the reactants C. That the products of the reaction have higher free energy than the reactants D. That the reaction requires an input of energy to proceed

B. That the products of the reaction have lower free energy than the reactants All reactions require an input of energy to proceed (thus overcoming the activation energy). The speed of a reaction is a kinetic concept, not a thermodynamic concept. Spontaneous reactions are defined as those whose products have a lower free energy than the reactants.

At high altitude, water vapor pressure in the lungs remains the same and carbon dioxide pressure falls slightly. Oxygen pressure falls. The body of a person remaining at high altitudes for days, weeks, and even years will acclimatize. All of the following changes assist the body in coping with low oxygen EXCEPT: A. increased red blood cells B. decreased vascularity of the tissues C. increased pulmonary ventilation D. increased diffusion capacity of the lungs

B. decreased vascularity of the tissues The person would need increased vascularity to deliver more blood to the tissues because the blood would carry less oxygen. Red blood cells carry oxygen, and an increase would be helpful in a low oxygen environment. Increased pulmonary ventilation would continuously introduce fresh oxygen into the alveoli and circulation. Finally, increased diffusing capacity increases the efficiency of gas exchange, helping to compensate for the lower oxygen content at high altitude.

Social-cognitive theory describes: A. changes in social behavior driven by thought processes B. changes in attitudes driven by observations of the actions of others C. behavior change driven by social learning in addition to internal and external drives D. behavior change driven by social learning in the absence of internal and external drives

C is correct. Social-cognitive theory involves behavior change that is not only influenced by internal and external drives, but also informed by observation of the actions of others. Choice C provides the most complete explanation of social-cognitive theory. Choice Dis incorrect because it indicates that internal and external drives are not involved. Choice A leaves out the important aspect of learning through observation. Finally, choice C incorrectly posits that attitudes are the main focus of social-cognitive theory. Note that social-cognitive theory also emphasizes the control of behavior in the absence of reward and punishment, though this is not reflected in any of the answer choices.

All of the following act as second messengers for hormones EXCEPT: A) cyclic AMP. B) calmodulin. C) acetylcholine. D) cyclic GMP.

C) acetylcholine. Acetylcholine acts through a second messenger system, and is not a second messenger itself. The other choices, cAMP, cGMP, and calmodulin are all examples of second messengers.

The liver synthesizes factors that act cooperatively with platelets to facilitate which physiological process? A. Cholesterol synthesis B. Glucose metabolism C. Blood clotting D. Fat digestion

C. Blood clotting Platelets form a plug at the site where a blood vessel has been damaged. Blood clotting factors that have been synthesized in the liver in an inactive form then participate in a cascade that leads to a blood clot.

The Lucas test distinguishes between the presence of primary, secondary, and tertiary alcohols based upon reactivity with a hydrogen halide. The corresponding alkyl chlorides are insoluble in Lucas reagent and turn the solution cloudy at the same rate that they react with the reagent. The alcohols, A, B, and C, are solvated separately in Lucas reagent made of hydrochloric acid and zinc chloride. If the alcohols are primary, secondary, and tertiary respectively, what is the order of their rates of reaction from fastest to slowest? A. A,B,C B. B,A,C C. C,B,A D. B,C,A

C. C,B,A The reaction of an alcohol with a hydrogen halide to form an alkyl chloride is a substitution reaction. Regardless of whether or not the mechanism is SN1 or SN2, the hydroxyl group will not leave until it has been protonated. For SN1, the formation of the cation, although rate determining, is very fast after protonation. It is faster, in fact, than SN2, which depends on the chloride ion colliding with an already protonated molecule from the correct side. Molecules with tertiary leaving groups are the fastest to react in SN1, followed by secondaries. Primary leaving groups react only through SN2. The fastest reaction will thus involve the tertiary alcohol and the slowest will involve the primary; every choice but C can be eliminated. This is a very difficult question- organic chemistry questions this difficult are relatively rare.

Which structures are responsible for maintaining levels of estrogen earlier and later in pregnancy? A. Placenta earlier and ovaries later B. Uterus earlier and ovaries later C. Corpus luteum earlier and placenta later D. Placenta earlier and umbilical cord later

C. Corpus luteum earlier and placenta later Uterus does not secrete estrogen Umbilical cord does not secrete estrogen The ovaries hold the corpus luteum that secrete estrogen earlier in the pregnancy and the placenta is responsible for estrogen production later in the pregnancy

Cellular respiration involves the flow of electrons from one chemical species to another: which of the following best describes this process? A. Electrons from oxygen are ultimately transferred to glucose B. Electrons from oxygen are ultimately transferred to water C. Electrons from glucose are ultimately transferred to oxygen D. Electrons from glucose are ultimately transferred to carbon dioxide

C. Electrons from glucose are ultimately transferred to oxygen Glucose and oxygen are the reactants in the net chemical reaction for cellular respiration. During the process of cellular respiration, glucose is oxidized to carbon dioxide whereas oxygen is reduced to water. This last point is usually captured by the statement "oxygen is the final electron acceptor in cellular respiration." Electrons from glucose are ultimately transferred to oxygen.

Which of the following represent possible sources of error when estimating Km and Vmax from a Lineweaver-Burk plot? I. The inability to obtain negative values of 1/[S] often requires extrapolation of the linear fit over a long stretch lacking data points in order to determine the x-intercept. II. Errors in the data obtained a low substrate concentration have a disproportionate impact on the determination of the best linear fit. III. Visual estimation of Km and Vmax is more challenging from a linear plot as compared to the hyperbolic plot of V vs [S]. A. I only B. II only C. I and II only D. I, II, and III

C. I and II only

Which of the following are products when an alcohol is added to a carboxylic acid in the presence of a strong acid? I. water II. ester III. aldehyde A. I only B. II only C. I and II only D. I and III only

C. I and II only This is an esterification reaction. As the name implies, one of the products will be an ester, so II is true. This eliminates choices A and D. Esterification is an example of a dehydration synthesis reaction, which means that a new bond is formed by removing a water molecule. For this reason, water is a product, and I is also true. Esterification does not generate an aldehyde, making choice B incorrect and C correct.

Patients with Huntington's disease have difficulties recognizing when others are feeling disgust. Damage to what brain region in Huntington's disease likely results in this severe deficit, due to its important role in the recognition of the facial expression associated with disgust? A. Basal ganglia B. Hippocampus C. Insula D. Amygdala

C. Insula Damage to the basal ganglia causes problems recognizing angry facial expressions. The amygdala is most associated with the recognition of the facial expressions associated with fear. The hippocampus is necessary for fear conditioning. The insula is the brain structure most associated with disgust. The anterior insula receives signals from the senses of olfaction and gustation, while the posterior insula receives signals from audition and somatosensation.

Reductive detoxification of reactive oxygen intermediates is critical to the survival of aerobic organisms. Which of the following is most directly involved in this process in humans? A. NADH produced by pentose phosphate pathway B. NADH produced by gluconeogenesis C. NADPH produced by the pentose phosphate pathway D. NADPH produced by gluconeogenesis

C. NADPH produced by the pentose phosphate pathway NADPH is not generated by gluconeogenesis and NADH is not generated by pentose phosphate pathway.

All of the following accurately describe differences between RNA and DNA EXCEPT: A. In general, DNA is a double-stranded and RNA is single-stranded. B. In a eukaryotic cell, DNA is found in the nucleus; RNA can be found in both the cytosol and nucleus. C. On one of the carbons of the pentose sugar, there is a hydroxyl group in DNA but only a hydrogen in RNA. D. RNA contains uracil where DNA countains thymine.

C. On one of the carbons of the pentose sugar, there is a hydroxyl group in DNA but only a hydrogen in RNA. DNA = Deoxy-ribo... RNA= ribo... RNA contains hydroxyl DNA only has hydrogen

Based on electronic structure, which of the following ions would be expected to be pulled toward a magnet? A. S 2- B. Mg 2+ C. Rb 2+ D. F -

C. Rb 2+ Paramagnetic materials are pulled toward magnet fields. Paramagnetic materials have at least one unpaired electron. All of the electrons in a noble gas configuration are paired. Rb2+'s electron configuration is [Ar]4s2 3d10 4p5, so one of the 4p electrons is unpaired.

In a Hb dissociation curve, what kind of shift most likely corresponds to a patient suffering from acidemia (low blood pH)? A. No shift B. Left shift C. Right shift

C. Right shift An increase in [H+] decreases Hb affinity for oxygen; that is, the binding of hydrogen ions to a molecules of oxyhemoglobin enhances the release of oxygen. A patient suffering from acidosis has a decreased blood pH, which means H ions concentration is higher than normal so Hb will release oxygen more readily than it does under normal conditions. This means that at a given partial pressure of oxygen in the blood, the percent saturation of Hb with oxygen in a patient with acidosis will be lower than it would be at physiological pH.

Which of the following would most likely occur in the presence of a carbonic anhydrase inhibitor? A. The blood pH would increase B. The carbamino hemoglobin concentration inside erythrocytes would decrease C. The rate of gas exchange in the lungs would decrease D. The oxyhemoglobin concentration inside erythrocytes would increase

C. The rate of gas exchange in the lungs would decrease The rate of gas exchange in the lungs would decrease, as carbonic anhydrase catalyzes the reaction involving carbonic acid and carbon dioxide. The carbon dioxide in the blood would not be expelled as quickly with lowered carbonic anhydrase activity. Catalysts increase the rate of a reaction. If the catalyst is inhibited, the rate decreases. Since the reaction moves in one direction in the lungs, and the opposite direction in the tissues, choice A is ambiguous. Unless one believes that a carbonic anhydrase inhibitor will affect transcription or degradation of hemoglobin, choices B and D are equivalent. If one were true, the other should also be true, so both can be ruled out.

What dietary source of energy contains the most energy per gram (kcal/g)? A. Disaccharides B. Cellulose C. Triglycerides D. Proteins

C. Triglycerides ATP is the biochemical currency of energy, and is produced via the Krebs cycle/ oxidative phosphorylation. Acetyl CoA is the starting reactant for the Krebs cycle. In oversimplified terms, in order to produce one molecule of acetyl CoA, two carbons must be available from a food source. Triglycerides (triacylglycerols) are long chains of hydrocarbons, and many molecules of acetyl CoA can be produced from them. Furthermore, relative to proteins and carbohydrates, they have a low ratio of high molecular weight atoms (oxygen, nitrogen, etc.) to hydrocarbons. This makes them 'energy dense'.

What is the function of the MEDULLA OBLONGATA? A. Coordinated movement B. Smooth movement C. Vital function (breathing, digestion) D. Sensorimotor reflexes

C. Vital function (breathing, digestion) Part of the lower brain structure; it is responsible for regulating vital functions such as breathing, heart rate, and blood pressure.

Heart and liver cells can produce more ATP fore each molecule of glucose than other cells in the body. This is most likely due to: A. a more efficient ATP synthase on the outer mitochondrial membrane B. an additional turn of the Krebs cycle for each glucose molecule C. a more efficient mechanism for moving NADH produced in glycolysis into the mitochondrial matrix D. production of additional NADH by the citric acid cycle

C. a more efficient mechanism for moving NADH produced in glycolysis into the mitochondrial matrix This answer can most easily be found by process of elimination. Choice A is incorrect because ATP synthase is on the inner mitochondrial membrane. Choice B and 0 are poor answers because they mention the specific processes, Glycolysis and Krebs cycle, with which you should be familiar. A change in these processes would indicate a completely different process. Choice C, on the other hand, refers to membrane transport in a more general way allowing for the possibility that a specific mechanism of transport may differ in heart and liver cells.

Which of the following reactions will undergo an SN1 reaction? A. 1-iodo-3-methyl pentane with KOH B. 1-bromopropane with ethanol C. t-butyl bromide with ethanol D. 2-bromo-4-methylpentane with LiBr

C. t-butyl bromide with ethanol SN1 reactions typically have weak nucleophiles. Large, branched electrophiles with great leaving groups lead to SN1 reactions. Tertiary alkyl halides are large, branched electrophiles with great leaving groups. In addition, ethanol is a weak nucleophile. The reactants t-butyl bromide with ethanol are likely candidates to undergo SN1

The electrons in the pi bond of an alkene have: A. 33% p character and are at a lower energy level than the electron pair in the sigma bond B. 50% p character and are at a higher energy level than the electron pair in the sigma-bond C. 100% p character and are at a lower energy level than the electron pair in the sigma bond D. 100% p character and are at a higher energy level than the electron pair in the sigma bond

D An alkene is one or more carbon-carbon double bond that contains one sigma and one pi bond. According to the valance bond theory, the sigma bond is formed by the overlap of sp2 hybrid orbitals whereas the pi bond is formed by the overlap of unhybridized p orbitals. The electrons in the pi bond have 100% p character. Th electrons occupying the pi bond are said to occupy a 2p orbital whereas those in the sigma bond are said to occupy an sp2 hybrid orbital with energy higher than that of a 2s orbital but lower than that of a 2p orbital. Therefore, the electrons in the pi bond are considered to be at a higher energy level than those in the sigma bond.

The heart, bone and skeletal muscle most likely arise from which of the following primary germ layers? A) The ectoderm B) The endoderm C) The gastrula D) The mesoderm

D) The mesoderm Generally, the inner lining of the respiratory and digestive tracts, and associated organs, come from the endoderm. The skin, hair, nails, eyes and central nervous system come from ectoderm. Everything else comes from the mesoderm. The gastrula is not a germ layer.

Suppose 12x 10^23 atoms of sodium metal react stoichiometrically with chlorine gas. How many grams of sodium chloride will result if the molar mass of sodium chloride is 60 g/mol? A. 60 g B. 10 g C. 100 g D. 120 g

D. 120 g 2 Na + Cl2 --> 2NaCl Divide 12 x 10^23 by avogadro's number to yield 2 mol of Na entering reaction From balanced reaction, 2 mol of Na will yield 2 mol of NaCl From the molar mass, 2 mol of NaCl corresponds to 120 grams.

216Po undergoes two alpha decays and two beta decays to form A. 208Tl B. 224Ra C. 212Pb D. 208Pb

D. 208Pb 216Po has mass of 216 and atomic number of 84. alpha decay= loss of two protons and two neutrons beta decay= converts neutron into proton, electron, and neutrino For mass, each alpha decay result in loss of four mass units whereas beta decay does not change mass. therefore the isotope has a mass number of 208

What is an aponeurosis? A. A segment of muscle that is not attached to bone B. An area of smooth muscle in the abdomen C. A group of neurons that innervate striated muscle cells D. A large, flat tendon that anchors muscle

D. A large, flat tendon that anchors muscle An aponeurosis is not muscle, but does relate to muscular attachments. Despite the name, an aponeurosis does not pertain to the nervous system. An aponeurosis is a large, flat tendon that joins certain muscle groups, like those in the abdomen, the lower back, the palms and soles, and the muscles around the scalp.

Which of the following mitochondrial genome characteristics differs most from the characteristics of the nuclear genome? A. Mitochondrial DNA is a double-helix B. Some mitochondrial genes code for tRNA C. Specific mutations to mitochondrial DNA can be lethal to the organism D. Almost every base in mitochondrial DNA codes for a product

D. Almost every base in mitochondrial DNA codes for a product

A different patient with a brain injury is having difficulty naming objects and retrieving words. He often looks at objects and cannot retrieve the word that corresponds with the object.. What is he most likely experiencing? A. Global aphasia B. Wernicke's aphasia C. Broca's aphasia D. Anomic aphasia

D. Anomic aphasia Global aphasia is a combination of impaired comprehension and production of speech. Wernicke's aphasia is characterized by difficulty understanding spoken words and sentences, as well as difficulty producing sentences that make sense. Anomic aphasia is characterized by problems difficulties in naming objects or in retrieving words.

Which of the following best describes the relationship between consciousness and attention? A. Consciousness and attention are separate processes that act independently B. Attention ensures that all sensory information is available for conscious perception C. Consciousness controls which information becomes available for analysis through attention D. Attention controls the flow of information that becomes available for conscious perception

D. Attention controls the flow of information that becomes available for conscious perception Consciousness can be described simply as the sum of all information that we are aware of, at the exclusion of all unconscious information. For example, a person reading a book might be conscious of the visual information conveying the words on each page, but not conscious of the sensory information coming from the left foot that has not been moved for an hour. The fact that not all information is available to consciousness at any given time makes choice B incorrect, and necessitates some mechanism for controlling what information reaches the level of conscious perception. Attention is the "gatekeeper" that performs this function. For this reason, choice D is correct. Consciousness and attention are intimately related, meaning choice A can be eliminated. Although choice C describes a relationship between the consciousness and attention, the relationship is reversed and thus incorrect.

Which of the following is a major organic component of bone? A. Elastin B. Keratin C. Hydroxyapatite D. Collagen

D. Collagen Elastin is an elastic protein that can be found in certain kinds of connective tissue that must be flexible but retain shape, such as the skin. However, it is not highly prevalent in bone. Keratin is the major structural protein of skin, hair, and nails. Keratin is also not highly prevalent in bone. Hydroxyapatite is a major component of bone, but it is not organic. Hydroxyapatite is an inorganic mineral with the formula, Ca10(PO4)6(OH)2 Collagen is the protein that makes up the major organic component of bone. It gives bone tensile strength and some amount of flexibility.

Compounds I and II, two oxides of the same transition metal, are placed into two vessels of aqueous media. Compound I produces an acidic solution, and Compound II produces a basic solution. It can be concluded that the metal-oxide bonding in: A. Compounds I and II are nonpolar B. Compound I is more polar than in bonding in Compound II C. Compound I is more ionic, and the bonding in Compound II is more covalent D. Compound I is more covalent, and the bonding in Compound II is more ionic.

D. Compound I is more covalent, and the bonding in Compound II is more ionic. Acidic compounds are those in which the MO-H bond is the ionic one; basic compounds are those in which the M-OH bond is ionic. Since compound I is acidic and compound II is basic, the metal-oxide bond in compound I is not ionic (because the O-H bond is), but the metal-oxide bond in compound II must be ionic.

Based upon closed-shell and half-closed shell considerations, which of the following metals is the LEAST reactive? A. Li (s) B. K (s) C. Mg (s) D. Cu (s)

D. Cu (s) Neither Li nor K has a closed valance shell, eliminating choices A and B. Mg has a closed valance s subshell and copper has a closed valance d subshell, giving copper greater stabilization, making it much less reactive.

Which of the following is NOT a feature of the stratum granulosum? A. There are 3-5 layers of cells in this layer B. Cells in this layer contain keratohyaline granules C. The cells of this layer secrete a lipid, waterproof layer D. Cytokeratin is produced in this layer

D. Cytokeratin is produced in this layer The stratum granulosum is the third deepest layer of the epidermis. Cytokeratin goes on to form tonofibrils. The stratum granulosum does not produce cytokeratin, this is done in the previous layer - the stratum spinosum.

Researchers hope to design an assay to measure the progression of apoptosis in live cells in real time. Which of the following four methods is the most promising avenue to pursue in order to develop this assay? A. Detecting increases in mitochondrial concentrations of cytochrome c B. Measuring decreases in mitochondrial volume C. Measuring increases in cytoplasmic volume D. Detecting increases in cytoplasmic concentrations of cytochrome c

D. Detecting increases in cytoplasmic concentrations of cytochrome c Apoptosis occurs via mechanisms that increase the permeability of mitochondria. Cytochrome c is an enzyme that's normally sequestered inside mitochondria. Detecting increases in cytoplasmic concentrations of cytochrome c is the most promising avenue, of the four listed, for developing an assay to measure the progression of apoptosis.

During pregnancy hormones are responsible for maintaining pregnancy. What kind of hormonal imbalances can lead to miscarriages during pregnancy? A. Decreasing levels of oxytocin B. Stimulation of progesterone secretory tissues C. Increasing levels of estrogen D. Early degeneration of corpus luteum

D. Early degeneration of corpus luteum Oxytocin is responsible for labor contractions and kept at low levels during pregnancy Levels of estrogen are kept high during pregnancy Progesterone secretion has to be kept high during pregnancy. Progesterone is initially secreted by corpus luteum, so early degeneration of corpus luteum may lead to misscarriage.

Metabolism of carbohydrate and fat spares protein tissue. All of the following are true of fats EXCEPT: A. Fats may be used in cell structure B. Fats may be used as hormones C. Fats are a more efficient form of energy storage than proteins D. Fats are a less efficient form of energy storage than carbohydrates

D. Fats are a less efficient form of energy storage than carbohydrates

A child points to a tree and says he sees a bird. The mother turns around toward the tree but does not see any bird. She assumes the child is lying. What tendency is the mother most closely exhibiting? A. Cognitive dissonance B. Belief perseverance C. Confirmation bias D. Fundamental attribution error

D. Fundamental attribution error The mother was unable to see the bird. This could have a number of explanations--the bird flew away by the time she saw it, the bird was camouflaged by the tree, the mother's eyesight was poor, the child was hallucinating or was unable to communicate properly what he was actually seeing. By assuming the child was lying, the mother overestimated the effect of the child's personality and underestimated the effect of the situation itself.

Parathyroid hormone is an important hormone in the control of blood calcium ion levels. Parathyroid hormone directly impacts: I. bone density II. renal calcium reabsorption III. blood calcium concentration A. I only B. I and II only C. I and III only D. I ,II, and III

D. I,II, and III Parathyroid hormone is all about raising blood calcium levels. It stimulates osteoclast (bone resorption) activity, decreasing bone density. It also works in the kidney to slow calcium lost in urine. It controls blood calcium levels via these two mechanisms. Because parathyroid hormone has to do with blood calcium levels choices A and B can be eliminated because they do not contain option III. Then, option I is in both remaining answers, so it does not need to be carefully considered. Afterwards, determine if parathyroid hormone impacts renal calcium reabsorption, which it does.

Introns that are self splicing are classified as either group I or group II introns. Which of the following is a common trait of group I and II introns that makes them different from introns that are not designated into a group number? A. Their removal is catalyzed by a complex called a spliceosome B. A lariat structure intermediate is formed during splicing C. A uracil-rich snRNA is involved during the splicing reaction D. No protein enzymes are involved in the splicing mechanism

D. No protein enzymes are involved in the splicing mechanism Group I and II introns are unique in that they are self-splicing, or do not involve protein enzymes in their splicing reactions. Choices A and C actually apply to non-number designated introns, and choice B occurs for both kinds of introns.

What is spermiogenesis? A. Selection of spermatogonia to become sperm. B. The process of meiosis in males. C. A process happening in the spermatic cord. D. Shedding of unnecessary cytoplasm.

D. Shedding of unnecessary cytoplasm. Meiosis in males is called spermatogenesis Spermiogenesis occurs in epididymis Spermiogenesis is not selection of spermatogonia During spermiogenesis, maturation of the sperm, unnecessary cytoplasm is shed off

What type of memory loss is most common during the initial stage of Alzheimer's disease? A. Semantic memory loss B. Episodic memory loss C. Procedural memory loss D. Short term memory loss

D. Short term memory loss In early stages of Alzheimer's disease, most patients with Alzheimer's can still remember details about their childhood. In the early stages of Alzheimer's disease, most patients can still remember how to use objects. In the early stages of Alzheimer's disease, most patients can still remember general information that they've learned throughout their life. In the early stages of Alzheimer's disease, most patients have difficulty remembering the name of a person they just met.

Which of the following claims would most likely be made by a proponent of the behaviorist theory of personality? A. By selecting certain behaviors and life experiences, individuals can shape their own personality development B. Personality development occurs through a continual interaction between genetics and behaviors C. Mental life, social influences, and learning experiences combine to influence personality development D. Similar to how people learn certain behaviors based on environmental consequences, personality development occurs as people become more likely to carry out certain behaviors based on input from the environment

D. Similar to how people learn certain behaviors based on environmental consequences, personality development occurs as people become more likely to carry out certain behaviors based on input from the environment Behaviorist theory states that personality is made up of behaviors, which are shaped by learning experiences. Someone who adheres to the behaviorist theory of personality would express a more narrow view on personality development. It also pays little attention to mental life.

Eukaryotic proteins may be toxic to bacteria when produced in large amounts. As such, scientists find it useful to insert eukaryotic proteins into bacterial DNA at a site where the protein may be induced. Scientists use the lac operon to control eukaryotic protein expression. Provision of allolactose induces expression of the eukaryotic protein. What occurs in the absence of allolactose? A. The repressor binds RNA polymerase, preventing it from transcribing the eukaryotic genes. B. The repressor binds the structural gene, preventing RNA polymerase from transcribing the eukaryotic genes. C. The repressor binds the promoter, preventing RNA polymerase from transcribing the eukaryotic genes. D. The repressor binds the operator, preventing RNA polymerase from transcribing the eukaryotic genes.

D. The repressor binds the operator, preventing RNA polymerase from transcribing the eukaryotic genes. The regulator gene codes fro the repressor molecule. The promoter gene is the site of binding for the RNA polymerase. The operator gene is the binding site for the repressor. When the repressor is bound, the RNA polymerase cannot transcribe past the operator and the structural gene cannot be transcribed. The structural gene encodes for the desired protein. When the inducer is present, the repressor and inducer bind each other. The operator is free and the RNA polymerase can transcribe the structural gene. When no inducer is present, the repressor binds the operator and prevents RNA polymerase from transcribing the eukaryotic gene.

What is the role of tropomyosin in muscle contraction? A. To release calcium from the sarcoplasmic reticulum B. To release troponin from tropomyosin, allowing myosin to bind to the actin filament C. To aid in myosin 'cocking' on the actin filament D. To prevent myosin from continuing to slide up the actin filament

D. To prevent myosin from continuing to slide up the actin filament Tropomyosin does not directly aid in calcium release or myosin 'cocking'. Tropomyosin does not enhance binding of actin and myosin - in fact, it does quite the opposite. Tropomyosin binds to actin and prevents myosin from sliding up the actin filament until tropomyosin experiences a conformational change.

A 50/50 mixture of (+)-dopa and (-)-dopa would be best described as: A. optically active B. diastereomers C. a meso compound D. a racemic mixture

D. a racemic mixture 50/50 mix of enantiomers forms a racemic mixture

Insulin shock occurs when a patient with diabetes self- administers too much insulin. Typical symptoms are extreme nervousness, trembling, sweating, and ultimately loss of consciousness. The physiological effects of insulin shock most likely include: A. a pronounced increase in gluconeogenesis by the liver. B. a rise in blood fatty acid levels, leading to atherosclerosis . C. a dramatic rise in blood pressure. D. dangerously low blood glucose levels.

D. dangerously low blood glucose levels. The only important thing to recognize from the question is that high insulin levels exist. Then go to the basics; insulin decreases blood glucose. Telling the liver to make more glucose through gluconeogenesis will raise the blood glucose and is the job of glucagon. Although glucagon will definitely be released in response to the dangerously low blood glucose levels, it is not the direct result of high insulin.

What are the quantum numbers describing the highest angular momentum number, spin-up electron in a neutral, unexcited chlorine atom's highest-energy orbital? A. n=3, l=1, ml=+2, ms=-½ B.. n=3, l=3, ml=+3, ms=+½ C. n=3, l=2, ml=+2, ms=+½ D. n=3, l=1, ml=+1, ms= +½

D. n=3, l=1, ml=+1, ms= +½ Chlorine's electron configuration is [Ne]3s2 3p5 The highest energy electrons are in the last orbitals to fill. The last orbitals to fill in this case are the 3p orbitals, so the principle quantum number, n, is 3 from the 3 in "3p." The p-orbital electrons (l=1) have ml=-1, 0, or 1 (of which 1 is the highest angular momentum number). Spin "up" for an electron means ms=+½.

Which of the following is true of all steroids? A. The target cells of any steroid include every cell in the body B. Steroids bind to receptor proteins on the membrane of their target cells C. Steroids are synthesized on the rough ER D. steroids are lipid soluble

D. steroids are lipid soluble Steroids are lipid soluble. Different steroids may have different target cells. For instance, estrogens are very selective while testosterone affects every, or nearly every, cell in the body. Steroids act at the transcription level in the nucleus, and are synthesized by the smooth endoplasmic reticulum.

If an acetylcholine antagonist were administered generally into a person, all of the following would be affected EXCEPT: A. The neuroeffector synapse in the sympathetic nervous system B. The neuroeffector synapse in the parasympathetic nervous system C. The neuromuscular junction in the somatic nervous system D. the ganglionic synapse in the sympathetic nervous system

A Every type of synapse in the peripheral nervous system uses acetylcholine as its neurotransmitter except the neuroeffector synapse in the sympathetic nervous system. As the "effector", it can be reasoned that this synapse is the end organ synapse. An effector is an organ or a muscle, something that responds to neural innervation by making something happen in the body.

Which of the following changes to the neuron at rest would be LEAST likely to prevent the formation of the normal resting membrane potential? A. Disruption of the electrochemical gradient of chlorine B. Disruption of the electrochemical gradient of sodium C. Disruption of the electrochemical gradient of potassium D. A change to the membrane resulting in impermeability to potassium

A electrochemical gradient of K+ is important for resting membrane potential. Leakage of Na+ down its concentration gradient also contributes to the resting membrane potential, making it slightly less negative than equilibrium potential of K+ The electrochemical gradient of Cl- makes only a minor contribution to the resting membrane potential compared to those of K+ and Na+

Which of the following does NOT accurately describe a way in which traditional IQ tests are inconsistent with the theory of multiple intelligences? A. Traditional IQ tests do not asses a range of abilities B. Traditional IQ tests are used to diagnose global learning disabilities C. Performance on traditional IQ tests predicts performance on a variety of intelligence tests. D. Traditional IQ tests correlate with academic performance

A is correct. Although tests of multiple intelligences emphasize the variety of modalities they assess, this does not mean that traditional IQ tests only test one ability. This is evidenced by the fact that scores are often composed of a verbal and a performance metric, as well as the fact that IQ scores correlate with performance in a variety of domains, including math and verbal skills. The use of IQ tests to diagnose global learning disabilities (however flawed a practice this may be) is unique to IQ tests as compared to tests of multiple intelligences, eliminating choice B. Performance on traditional IQ tests does predict performance on a variety of intelligence tests. This implies the presence of a g, or a general intelligence, a notion that is antithetical to the theory of multiple intelligences; thus C is incorrect. While it is true that traditional IQ tests correlate with academic performance, there is no indication that tests of multiple intelligences share this correlation, making choice D incorrect.

Which of the following LEAST demonstrates a way in which language affects cognition? A. A new language is developed that only contains gender-neutral pronouns. B. Speakers of a language with a highly developed vocabulary for blue colors have enhanced recall of the particular shade of various blue colored objects C. A vocabulary for a new field of ethics makes ethical constructs easier to understand D. The Australian language Guugu Yimithirr lacks relative directions (e.g. left, ahead); speakers of this language express all spatial relationships in terms of cardinal directions (e.g. North, West)

A is correct. Choice A is the only answer choice that does not contain any allusion to a process related to cognition. While it is likely that a language containing only gender-neutral pronouns might affect patterns of thought, such an effect is not implied in this choice. By contrast, all the other choices point to effects on cognition. Choice B describes how enhanced vocabulary within a particular domain of a language facilitates memory. There is an implied facilitation of cognition in this scenario: different shades of blue are better remembered because they are thought of as different colors to that language speaker (it is much easier to remember that one object is cerulean and another is azure rather than that one is a medium blue-gray and the other is a deep sky blue}, making choice B incorrect. Choice C describes a scenario where language makes thought easier, demonstrating an effect of language on cognition; choice C is incorrect. The lack of relative directions in Guugu Yimithirr affects cognition by making it impossible to think in terms of relative directions and only allowing thought in terms of cardinal directions, making choice D incorrect.

A camper recognizes that she is in the path of a deadly snake, realizes that there is no possibility of getting to a hospital in time if she is bitten, and begins to panic. This scenario best describes: A. cognitive appraisal B. fight-or-flight response C. a cataclysmic event D. elaboration likelihood model

A is correct. Cognitive appraisal describes the modulation of a fear response as a result of thinking about the stressor. More specifically, a situation is evaluated for the presence of a potential threat. A secondary appraisal assesses one's personal ability to cope with the threat, with a greater stress response triggered if the threat cannot be dealt with. While the fight-or-flight response is likely involved in this situation, the sequence of events presented more closely aligns with a cognitive appraisal, making choice B incorrect. More mention of a physiological stress response would have been necessary to favor choice B over choice A as a correct answer. A cataclysmic event is a momentous, violent event and does not inherently refer to individuals' emotional responses, making choice C incorrect. The elaboration likelihood model describes how people can be persuaded and is not relevant to the scenario described in the question stem; choice D can be eliminated.

Avogadro's law states that one mole of an ideal gas takes up around 22 liters at standard temperature and pressure. Assuming all reagents can be treated as ideal gases, how many grams of carbon dioxide are produced in the complete reaction of 44 liters of butane (C4H10) with oxygen to produce carbon dioxide (at STP)? A. 10 g B. 44 g C. 4400 g D. 350 g

A. 10 g 2 C4H10 + 13 O2 --> 8 CO2 + 10 H20 From Avogadro's law, 44 liters of butane is 2 mol, which would produce 8 mol of carbon dioxide. The molar mass of carbon dioxide is 12+2x16 = 44 g/mol suggesting that 8x44=350 grams of carbon dioxide is released during the reaction.

What is the net number of ATP molecules synthesized by an obligate anaerobe per molecule of glucose? A. 2 ATP B. 6 ATP C. 8 ATP D. 36 ATP

A. 2 ATP An obligate anaerobe is an organism that must live WITHOUT oxygen in order to survive. Obligate anaerobes produce ATP via fermentation, which includes both glycolysis and the reactions necessary to regenerate the NAD+ necessary for glycolysis to continue. Fermentation leads to a net production of 2 ATP; this ATP is generated during glycolysis. Therefore, an obligate anaerobe will produce 2 ATP per molecule of glucose.

What is the work done by the patient during a 3-minute exercise on the bicycle? (Note: load is 30 W) A. 5400 J B. 90 J C. 6 J D. 0.16 J

A. 5400 J Power = Work / time Solve for Work = Power x time W= (30W) x (3 min x 60 sec/1min) = 5400 J

One type of anaerobic respiration converts glucose, C6H12O6, to ethanol C2H5OH and carbon dioxide. If the molecular weight of glucose is 180 g/mol and the molar mass of ethanol is 50 g/mol, how many grams of carbon dioxide are produced when 1 mol of glucose is digested via respiration? A. 80 grams B. 40 grams C. 100 grams D. 0 grams

A. 80 grams Balance reaction C6H12O6 --> 2 C2H5OH + 2 CO2 1 mol glucose = 180 grams. Total weight of products must equal to this weight due to conservation of mass. 2 mols of C2H5OH are produced, weighing 100 grams. 180-100=80 grams

Which of the following will most easily react with an amine to form an amide? A. Acyl chloride B. Ester C. Carboxylic acid D. Acid anhydride

A. Acyl chloride Amines typically act as weak bases or nucleophiles. Since an amide is a carboxylic acid derivative formed through nucleophilic substitution, the amine is likely going to attack as a nucleophile. Amides are the least reactive of the carboxylic acid derivatives, so any of the answers could participate in an amide-forming substitution. Since the question stem asks for the reagent that would most easily react, the correct answer is the most reactive carboxylic acid derivative. Esters, carboxylic acids, and acid anhydrides are all less reactive than acyl chlorides, so choice A must be correct. The reactivity of these derivatives is based on the strength of their leaving groups; the better the leaving group, the more reactive the molecule.

A protein hormone is least likely to act by which of the following mechanisms? A. Binding to a receptor in the nucleus B. Opening a membrane bound channel C. Activating a membrane bound enzyme D. Activating a G protein

A. Binding to a receptor in the nucleus

Which of the following will most likely occur during heavy exercise? A. Blood pH will decrease in the active tissues B. Less oxygen will be delivered to the tissues due to increased cardiac contractions that result in increased blood velocity C. Capillaries surrounding contracting skeletal muscles will constrict to allow increased freedom of movement D. The respiratory system will deliver less nitrogen to the blood

A. Blood pH will decrease in the active tissues Cellular respiration produces carbon dioxide, which, in turn, lowers blood pH. Increased cardiac contractions and blood velocity would likely increase oxygen delivery to muscle tissue. During heavy exercise, capillaries dilate in order to deliver more oxygen to the active tissues. Nitrogen is irrelevant to respiration.

Which aspect of equality is considered a requirement in a meritocracy? A. Equality of opportunity B. Equality of talent C. Equality of skill D. Equality of outcome

A. Equality of opportunity A meritocracy is when societal rewards, status, and positions are awarded to individuals based on their own ability and work (that is, merit). In order for a meritocracy to operate, everyone within the society would need the same opportunity to succeed, so that rewards are actually based (primarily) on merit.

A patient with renal failure has nephrons which lack the ability to actively secrete or reabsorb any substances. Which of the following actions will the patient's kidney most likely still be able to perform? A. Excretion of salt in urine B. Production of hypertonic urine C. Regulation of blood osmolarity D. Conservation of glucose

A. Excretion of salt in urine Recall that the fluid in Bowman's capsule is isotonic to plasma. Without reabsorption or secretion, isotonic urine can still be produced. In other words, this patient's nephrons filter the blood at the glomerulus, but that initial filtrate then becomes urine without any further modification. This process still allow excess salt to be removed from the body, because the salt diffuses through the glomerulus, which is still functioning in this patient. B is wrong because the production of hypertonic urine requires the concentration gradient between the medulla and cortex of the kidney. This gradient is generated by active secretion and reabsorption.

The index of refraction of the vitreous humor is greater than the index of refraction of the aqueous humor, which is greater than the index of refraction of air. How does the speed of light in each of these media compare? A. Fastest in air, slower in aqueous humor, slowest in vitreous humor B. Fastest in vitreous humor, slower in aqueous humor, slowest in air. C. Fastest in aqueous humor, slower in air, slowest in vitreous humor. D. Depends on the index of refraction of the lenses that separate the media

A. Fastest in air, slower in aqueous humor, slowest in vitreous humor Recall that the material with the lowest index of refraction will enable light to travel through it at the fastest speed. Therefore, the correct answer choice must list the media in order to increasing indices of refraction, which will correspond to decreasing order of speed of light in the media.

Which glycolytic reaction cannot be directly reversed in gluconeogenesis? A. Glucose → glucose-6-phosphate B. Glyceraldehyde 3-phosphate → 1,3-bisphosphoglycerate C. 1,3-bisphosphoglycerate → 3-phosphoglycerate D. Glucose-6-phosphate → fructose-6-phosphate

A. Glucose → glucose-6-phosphate Three distinct glycolytic reactions are notable for having a large, negative free energy change. These three reactions are therefore spontaneous and irreversible. The three most favorable steps of glycolysis are: glucose→ G-6-P, fructose-6-phosphate→ fructose 1,6-bisphosphate, and phosphoenolpyruvate→ pyruvate. Glucose → glucose-6-phosphate cannot be directly reversed in gluconeogenesis.

A researcher is interested in fear and explicit memory. He recruits a sample of undergraduate students, divides them into three groups, and shows each group of participants a different video. Group A watches a neutral video that reliably induces boredom, group B watches a video that reliably induces mild fear, and group C watches a video that reliably induces overwhelming fear. The participants are tested a week later to determine how much of the video they are able to remember. Given the design of the study and what you know about emotion and memory, which group(s) probably remembers the most about the video? A. Group B B. Group C C. Groups A and C D. Group A

A. Group B Explicit memory is a type of long term memory that focuses on recalling previous experiences and information. The relationship between long term memory and fear follows a Yerkes-Dodson curve. This means that extreme emotional responses usually impact memory negatively. Boredom is usually associated with inattention, which also impacts memory negatively. Moderate emotions, like mild fear, are associated with optimal memory recall. Group B probably remembered the most from the video.

Prior to assimilation, ethnic groups migrating from a less industrialized culture to a more industrialized culture generally compare to their new culture in which of the following ways? A. Higher fertility, higher mortality B. Higher fertility, lower mortality C. Lower fertility, higher mortality D. Lower fertility, lower mortality

A. Higher fertility, higher mortality Ethnic groups emigrating to a more industrialized culture typically have larger families, but shorter lifespans initially.

A scientist uses thin-layer chromatography on silica plates to separate a mixture of phenylalanine and tyrosine. Does phenylalanine have a higher or lower Rf than tyrosine? A. Higher, because tyrosine is more polar than phenylalanine. B. Higher, because phenylalanine is more polar than tyrosine. C. Lower, because tyrosine is less polar than phenylalanine. D. Lower, because phenylalanine is less polar than tyrosine.

A. Higher, because tyrosine is more polar than phenylalanine. Silica plates are very polar, thus the stationary phase is polar. Molecules with an affinity for polar phases will have a lower Rf. Since tyrosine is more polar (due to OH group) it will have a greater preference for the stationary phase than phenylalanine, and phenylalanine will have a higher Rf.

Which of the following correctly indicates increasing levels of complexity in the organization of genetic information around protein complexes? A. Histones --> Nucleosomes --> Chromatin --> Chromosomes B. Nucleosomes --> Histones --> Chromatin --> Chromosomes C. Histones --> Nucleosomes --> Chromosomes --> Chromatin D. Nucleosomes --> Histones --> Chromosomes --> Chromatin

A. Histones --> Nucleosomes --> Chromatin --> Chromosomes The DNA is wrapped around histone proteins. Eight wrapped histones in conjunction form a nucleosomes. The nucleosomes wind into complexes to form chromatin. When cellular division takes place, chromatin is further condensed into chromosomes that can be split between the two daughter cells.

A hot object is placed next to a cold object so that they are touching. Which of the following statements is true? I. Heat will transfer from the hot object to the cold object because the hot object has a higher temperature. II. The two objects are in thermal equilibrium III. Internal energy will transfer from the hot object to the cold object because the hot object has greater internal energy. Please choose from one of the following options. A. I B. II C. I & III D. III

A. I Two objects are only in thermal equilibrium if they are physically touching each other (or have a third intermediary object touching both of them) and experience no heat flow between them. This is not the case in this situation as heat will always transfer from hot objects to cold objects. Temperature is related to speed of molecules. During collisions between molecules, the speeds tend to average out, increasing the speed of the slower molecules, so the kinetic energy and the related temperature of the molecules tends to even out over time, causing the heat to always transfer from hot to cold as long as there is a conducting path. Heat flows from hot to cold but internal energy does not necessarily get transferred from high internal energy to lower internal energy. Internal energy also encompasses a system's stored potential energy that does not immediately alter the speed of molecules and is not easily transferred through a conducting path.

A conservationist would like to test the acidity of a sample of rainwater by titrating it with 0.05M NaOH. What additional information is needed to calculate the initial number of moles of H2CO3 in the sample? I. Volume of NaOH used to reach the end point II. Ka of H2CO3 III. Initial volume of rainwater in the sample A. I only B. I and II only C. I and III only D. I, II, and III

A. I only Goal: find number of moles of H2CO3 not concentration. Therefore, I is correct. The Ka of weak acid doesn't matter since we are titrating with a strong base. Once OH- is introduced in to solution, it will remove the H+ no matter how strong or weak the acid

Von Gierke disease is an autosomal recessive disorder characterized by a deficiency in the enzyme Glucose-6-phosphate. Which of the following would most likely be observed in an individual with Von Gierke disease? A. Increased glycogen in liver B. Decreased post-meal glycolysis C. Decreased glucose tolerance D. Increased gluconeogenesis

A. Increased glycogen in the liver G-6-Phosphatase hydrolyzes g-6-p into a phosphate molecule plus glucose, which can then be used by cells to generate energy. Without this enzyme, gluconeogenesis cannot be completed. Glycolysis would not be affected, and glucose tolerance would be expected to be normal. Glycogenolysis produces g-6-p from glycogen stores in the body during times of fasting, or when blood glucose is low. However, without g-6-phosphatase, glucose cannot be made from g-6-p. Therefore, the breakdown of glycogen is impaired in these individuals.

One hour after a meal, which hormone would be expected to be elevated? A. Insulin B. Glucagon C. Epinephrine D. Cortisol

A. Insulin One hour after a meal, the primary source of energy is the intestines. In the fed state, blood glucose levels are high. Hormones respond to environmental conditions. Insulin levels would rise in response to high blood glucose levels. Glucagon and epinephrine promote glycogenolysis and cortisol promotes gluconeogenesis. They are all catabolic hormones. Their levels should be low while blood glucose is high.

Which of the following best expresses the algebraic relationship between molar solubility, x, of Ca3(PO4)2 and its solubility product constant Ksp? A. Ksp=108x^5 B. Ksp=72x^5 C. Ksp=32x^5 D. Ksp=81x^5

A. Ksp=108x^5 Ca3(PO4)2 --> 3 Ca^2+ + 2 PO4^ 3- Ksp = [Ca2+]^3 [PO4 ^3-]^2 = (3x)^3 (2x)^2 = (27x^3)(4x^2) = 108x^5

Carboxylic acids typically undergo all of the following reactions EXCEPT: A. Nucleophilic addition B. Nucleophilic substitution C. Decarboxylation D. Esterification

A. Nucleophilic addition This is a straightforward content question. Nucleophilic substitution is one of the most common and important reaction types for carboxylic acids, so choice B can be eliminated. As the name implies, decarboxylation involves the removal of a carboxylic acid group, making choice C incorrect. Esterification is an ester-forming reaction that occurs between carboxylic acids and alcohols, so choice D can also be eliminated. Nucleophilic addition reactions are favored by ketones and aldehydes, but do not occur with carboxylic acids.

A scientist monitors the nucleotide sequence of the third chromosome as a cell undergoes normal meiosis. What is the earliest point in meiosis at which the scientist can deduce with certainty the nucleotide sequence of the third chromosome of each gamete? A. Prophase I B. Metaphase I C. Prophase II D. Telophase II

A. Prophase I In normal meiosis, the only change in the nucleotide sequence of the third chromosome will occur during crossing over. Crossing over occurs in prophase I. Even though the cells remain diploid with sister chromatids at this point in meiosis, by the end of prophase I, crossing over has finished and the genetic sequence of each future gamete should be established with certainty, making choice A the best answer.

Feature detection is best described as which of the following? A. Sequential processing of information from parallel pathways B. Parallel processing of information from sequential pathways C. Sequential processing of information from sequential pathways D. Parallel processing of information from parallel pathways

A. Sequential processing of information from parallel pathways Parallel processing occurs when different types of ganglion cells in the retina respond simultaneously to different aspects of the visual scene. These cells send projections to two different pathways, one representing "form" and the other "motion." Since both of these pathways respond simultaneously to all visual information, this processing is designated "parallel." Once these projections reach the visual cortex, feature detection occurs in both pathways. Feature detection is referred to as "sequential" processing because increasingly complex features are analyzed in a serial fashion rather than sequentially. (For example, vertical and horizontal lines, shapes, and complex features like shadows might be detected in that order.) For this reason, choices Band D can be eliminated. Since feature detection occurs in both of the parallel pathways, choice C is also incorrect.

What is the role of acetylcholine in calcium release? A. Stimulates cellular contraction by opening voltage-gated ion channels B. Stimulates postsynaptic acetylcholine receptors to release intracellular calcium C. Binds to sarcoplasmic reticulum, causing calcium release D. Binds to the sarcolemma, causing calcium release

A. Stimulates cellular contraction by opening voltage-gated ion channels Remember that acetylcholine functions at the neuromuscular junction, not within the muscle cell - so it would not bind to structural elements like sarcoplasmic reticulum or sarcolemma. Acetylcholine receptors do not release calcium. Acetylcholine indirectly causes calcium release by opening voltage-gated ion channels.

People with albinism have a defect in tyrosinase, an oxidase that helps to control skin pigment production. In what epidermal layer is tyrosinase active? A. Stratum basale B. Stratum spinosum C. Stratum granulosum D. Stratum lucidum

A. Stratum basale Tyrosinase controls melanin production by melanocytes. Melanocytes are found in the deepest layer of the epidermis. Tyrosinase in the stratum basale, which contains the melanocytes that produce melanin - skin pigment that is absent in those with albinism.

Which of the following statements about the genome is inaccurate? A. The genome can be changed by environmental influence through epigenetic alterations B. The genome can be read in variable ways to produce different products C. The genome can be composed of DNA or RNA D. In multi-cellular organisms, different types of cells can read the genome differently

A. The genome can be changed by environmental influence through epigenetic alterations Epigenetic alterations change the way the genome is read by the cellular machinery, but they do not change the genome itself, making choice A false and the right answer.Choice C is true and incorrect because, while most organisms have DNA genomes, some viruses do contain genomes made from RNA. For choices B and D, it is true that the genome can be read differently under different circumstances. Notice that choices Band Dare essentially saying the same thing- that it is possible for the genome to be read differently. The repetitiveness of these two choices prevents either from being the correct answer.

The human brain is often described by the analogy of the brain as a computer. Which of these choices describes a possible problem with the information-processing model? A. The information-processing model assumes serial processing; however, the human brain has the capacity for parallel processing. B. The information-processing model assumes a limited capacity for attention; however, the human brain has unlimited attention capacity. C. The information-processing model assumes top-down processing; however, the human brain has the capacity for both stimulus and expectation driven processing. D. The information-processing model assumes unlimited storage capacity; however, the human brain has a limited capacity for storage.

A. The information-processing model assumes serial processing; however, the human brain has the capacity for parallel processing. The information-processing model is a bottom-up or stimulus driven model. The information-processing model assumes limited storage capacity. The human brain is believed to have a limited capacity for attention. The information-processing model assumes serial processing; however, the human brain has the capacity for parallel processing.

Which of the following is true concerning the slope of a lineweaver-burk plot in the presence of a competitive vs a noncompetitive inhibitor? A. The slope changes in the presence of both a competitive and noncompetitive inhibitor B. The slope changes in the presence of a competitive inhibitor only. C. The slope changes in the presence of a noncompetitive inhibitor only. D. The slope does not change in the presence of either type of inhibitor.

A. The slope changes in the presence of both a competitive and noncompetitive inhibitor.

When heating a solution, a scientist detects a temperature increase in the solution during a period of time. Which of the following statements accurately characterizes the solution during this period? A. The velocity of molecules in the solution is increasing. B. The solution's temperature increase is proportional to its ΔHvaporization. C. The solution is at boiling point. D. The solution is undergoing a phase change.

A. The velocity of molecules in the solution is increasing. When a substance absorbs heat, it can either use this to increase its temperature or undergo a phase change. It cannot do both. A solution at boiling point will use its continued applied heat energy to complete its phase change. The heat of vaporization (ΔHvaporization) is only used to determine the amount of heat energy used during the vaporization phase change. It does not indicate any information about the temperature change for a given heat energy input. As heat increases a solution's temperature, the velocity of the solution's molecules will increase.

Which of the following is accurate regarding Type I and Type II muscle fibers? A. Type I fibers store much of their energy as triglycerides, whereas Type II fibers store energy as ATP or creatine phosphate B. Type I fibers are white, whereas Type II fibers are red C. Type I fibers employ anaerobic metabolism, whereas Type II fibers utilize aerobic metabolism D. Type I fibers are easily fatigued and are therefore intended for short bursts of activity

A. Type I fibers store much of their energy as triglycerides, whereas Type II fibers store energy as ATP or creatine phosphate The important thing to remember for this question is that Type I fibers are those we use for long-term muscle activity, like standing in line, whereas Type II fibers are used for fast temporary actions. Therefore, the Type I fibers cannot be easily fatigued. They also utilize aerobic, not anaerobic, metabolism. Type I fibers store their energy as triglycerides (fat), rather than ATP or creatine phosphate, both of which are quickly and easily utilized.

Which of the following events does NOT play a role in the life cycle of a typical retrovirus? A. Viral DNA is injected into the host cell B. Viral DNA is integrated into the host genome C. The gene for reverse transcriptase is transcribe and the mRNA is translated inside the host cell D. Viral DNA incorporated into the host genome may be replicated along with the host DNA

A. Viral DNA is injected into the host cell

Which of the following is the electronic structure of Fe2+? A. [Ar]4s1 3d5 B. [Ar]4s2 3d6 C. [Ar]4s1 3d7 D. [Ar]4s2 3d4

A. [Ar]4s1 3d5 The electronic structure of neutral iron is [Ar]4s2 3d6. Fe2+ has two fewer electrons, so you might expect that the correct configuration is obtained by removing two electrons from the highest energy orbital, 3d, yielding [Ar]4s2 3d4. However, Fe2+ would have a lower - more favorable - energy if the 3d shell is half-filled with 5 electrons, so it will take on the [Ar]4s1 3d5 configuration (like chromium).

During the female menstrual cycle, increasing levels of estrogen cause: A. a positive feedback response, stimulating LH secretion by the anterior pituitary. B. a positive feedback response, stimulating FSH secretion by the anterior pituitary. C. a negative feedback response, stimulating a sloughing off of the uterine lining. D. a negative feedback response, stimulating decreased progesterone secretion by the anterior pituitary.

A. a positive feedback response, stimulating LH secretion by the anterior pituitary. Increased secretion of estrogen sets off the luteal surge, which involves increased secretion of LH and leads to ovulation. Choice B is close, but estrogen stimulates a surge in LH, not FSH. Choice C describes what takes place when there are decreasing levels of estrogen. Progesterone is not secreted by the anterior pituitary: it is secreted by the placenta or the corpus luteum.

In some specialized cells, glucose is transported against its concentration gradient via an integral protein using the energy of the sodium ion electrochemical gradient. If no ATP is used for this transport, it is most likely: A. active transport B. facilitated transport C. passive transport D. osmosis

A. active transport Anytime a compound moves against its electronchemical gradient across a membrane, it's active transport. The sodium electrochemical gradient was established by the expenditure of ATP, making this secondary active transport. The rest go down the concentration gradient

When UDP-glucose reacts with the nonreducing end of a lengthening glycogen polymer, which type of bond is most commonly formed? A. alpha-1,4-Glycosidic bond B. alpha-1,6-Glycosidic bond C. beta-1,4-Glycosidic bond D. beta-1,6-Glycosidic bond

A. alpha-1,4-Glycosidic bond The bond that links most glucosyl moieties in glycogen is the alpha-1,4-glycosidic bond.

Which of the following best exemplifies a primary circular reaction during the sensorimotor stage of development? A. an infant repeatedly vocalizing the same syllable B. an infant repeatedly banging their fist against a wall C. an infant understanding that a toy which is taken out of their view continue to exist D. an infant considering a person to "disappear" during a game when they hide their face

A. an infant repeatedly vocalizing the same syllable circular reactions are named for their repetitive nature and primary are those involving a repetitive behavior which the child finds soothing and secondary are those which involve a repetitive behavior that involves and affects their environment.

Most viruses that infect animals: A. enter the host cell via endocytosis B. do no require a receptor protein to recognize the host cell. C. leave their capsid outside the host cell D. can reproduce independently of a host cell

A. enter the host cell via endocytosis

Based on your understanding of intermolecular forces, which of the following pairs of organic solvents would be the least miscible? A. glycerol and propanone B. propanone and benzene C. propanone and 1-butanol D. 1-pentanol and toulene

A. glycerol and propanone Propanone and benzene are miscible. Although propanone is polar, most of the IMF derived from van der Waals force, so dipole-dipole attractive forces do not prevent miscibility. Only H bonding would prevent miscibility. Propanone and 1-butanol are miscible since 1-butanol can be a hydrogen bond donor and propanone can be a hydrogen bond acceptor. 1-butanol can also H-bond with itself, but there is no issue with miscibility here. 1-butanol and toulene are miscible because the highly polar hydroxyl group does not entirely dominate the interactions. The alkyl hydrocarbon allows for van der Waals forces between toulene and 1-butanol. Glycerol, aka propan-1,2,3-triol, and propanone are immiscible bc the glycerol-glycerol interactions due to its triple H-bonding sites will outweigh any potential glycerol-propanone interactions.

Vasopressin, a hormone involved in water balance, is produced in the: A. hypothalamus B. posterior pituitary C. anterior pituitary D. kidney

A. hypothalamus This is an important distinction to be made. The hormones of the posterior pituitary are synthesized in the bodies of neurons in the hypothalamus, and transported down the axons of these nerves to the posterior pituitary. Remember the hormones produced in the anterior pituitary are FLAT PG. The hormones produced in the hypothalamus are regulating hormones used to control the release of other hormones from the pituitary. The kidney is the target organ of vasopressin, not where it is produced.

A man lives alone and runs an online business, experiencing little contact with others. He makes enough money to buy food and pay rent. He suffers from mild depression and has few creative outlets. Presently, according to Maslow's hierarchy of needs, the man is probably experiencing motivation to meet the need for: A. intimacy B. self-esteem C. safety D. self-actualization

A. intimacy Maslow's hierarchy of needs dictates that higher needs do not produce drives until lower needs are met. This man has met the lowest two needs: physiological and safety, but since he lives alone and has little interaction with others, he has not met the next need on the hierarchy, intimacy.

Suppose that a researcher is interested in studying personality from the psychoanalytic perspective. She would most likely ask participants to: A. participate in role-playing activities that involve choosing between impulse gratification and societally appropriate behaviors B. complete questionnaires about how they would behave in morally complex situations C. describe their personal experiences with the conflict between following their own desires and following the "rules" for how to behave in public D. answer questions about the principles for social behavior that they learned from observing caregivers

A. participate in role-playing activities that involve choosing between impulse gratification and societally appropriate behaviors The psychoanalytic approach: psychological processes involved in personality are not consciously perceived. The correct answer will allow the researcher to study psychological phenomena of which the participants are not aware

Somatic symptom and related disorders are best defined as: A. psychiatric conditions defined by the psychological response to bodily symptoms B. mental illnesses that produce physical ailments C. disorders treated by clinicians that do not specialize in psychiatric care D. mental disorders presenting in situations that pose physical challenges

A. psychiatric conditions defined by the psychological response to bodily symptoms Both physical and psychological symptoms are required for a diagnosis of a somatic symptom disorder; the psychological symptoms consist of the response to the physical ("somatic") symptoms

Which of the following is a carbohydrate polymer that is stored in plants and digestible by animals? A. starch B. glycogen C. cellulose D. glucose

A. starch

How much s character is in the hybridized orbital on either nitrogen atom in diatomic nitrogen (N2)? A. 33.3% B. 50% C. 66.6% D. 100%

B Diatomic nitrogen comprises two nitrogen atoms that are triple bonded to one another. Given nitrogen's five valance electrons, this is the only configuration that will allow each atoms to obtain full octet of valance electrons. In addition to retaining a lone pair of electrons, each atom participates in one sigma bond, which consists of two overlapping sp hybridized orbitals and two pi bonds, each of which consists of two overlapping unhybridized p orbitals. Thus the hybridized orbitals of the nitrogen atoms are sp hybridized and exhibit 50% s character and 50% p character. sp2 hybridized orbitals exhibit 33.3% s characters.

Benzene exhibits resonance. The carbon-carbon bonds of benzene are: A. shorter and stronger than the double bond of an alkene B. longer and weaker than the double bond of an alkene C. longer and stronger than the carbon-carbon bond of an alkane D. longer and weaker than the carbon-carbon of an alkane

B Bonds that are stronger are shorter. The bonds are stabilized by resonance and are shorter and stronger than carbon-carbon alkane bonds but longer and weaker than carbon-carbon alkene bonds.

Which of the following is found in vertebrates but NOT in invertebrates? A. A dorsal, hollow nerve chord B. Myelination to increase the speed of nervous impulse transmission along the axon C. Axons through which the nervous impulse is conducted. D. An Na+/K+ pump is the neuronal membrane

B Invertebrates do not have myelinated axons to accelerate nervous impulse transmission. Instead, they rely upon increased size. Vertebrata is a subphylum of Chordata which is characterized by a dorsal nerve chord at some point in their development.

Which of the following is NOT true of the olfactory system? A. The olfactory system makes use of chemoreceptors that bind odorants B. Olfactory information does not reach the amygdala and hippocampus until after it undergoes processing in the cortex. C. Unlike other sensory systems, the olfactory system does not create a "map" of olfactory information in the environment D. Olfactory information does not pass through the thalamus before reaching cortical areas for higher level processing

B One of the significant features of the olfactory system is that information about olfactory stimuli has a direct route to the amygdala and hippocampus, rather than first being processed in the cortex. Choice B is not a true statement. The other answer choices accurately describe features of the olfactory system. Note that in choice D, olfactory sensory information bypasses the thalamus in its path to the cortex. This aspect of the olfactory system is unique among the sensory systems

During an action potential, the neuronal membrane: A. depolarizes, then hyperpolarizes, then repolarizes B. depolarizes, then repolarizes, then hyperpolarizes C. hyperpolarizes, then depolarizes, then repolarizes D. hyperpolarizes, then repolarizes, then depolarizes.

B Depolarization of membrane is due to the flow of positive sodium ions into the cell in the first step of the action potential; hyperpolarization is the brief drop to unusually low membrane potential at the end of the action potential, occurring after repolarization, the step that returns the membrane to a negative potential

Compared to patients i the other two clusters, those in cluster 2 were younger and had obtained higher educational levels. This difference is best described as a difference in: A. Social Class B. Demographics C. Social Institutions D. Material Culture

B Demographics Although education may relate to social class, difference in social class cannot be assumed on the basis of educational differences alone. Demographics are social categories that include age and educational levels.

Which of the following descriptions best aligns with the Cannon-Bard theory of emotion? A. Someone with a fear of spiders, feels her heart racing, then recognizes that she is feeling fearful. B. A viewer of a horror film simultaneously experiences a pounding heartbeat and a feeling of fear C. A pedestrian walking home at night notices that she has a rapid heart rate. She interprets this emotional response as excitement if she is looking forward to returning home, or as fear if she is being followed home by an unknown person. D. A student with a fear of public speaking thinks about a public speaking event, becomes anxious, and then experiences physical discomfort

B is correct. The Cannon-Bard theory of emotion states that emotional and physiological responses to a situation are experienced simultaneously, as described in choice B. Choice A describes the James-Lange theory, which says that a physiological response is interpreted as an emotional experience; thus choice A is incorrect. Choice C demonstrates the Schacter-Singer theory, which emphasizes the interpretation of a physiological response based on situational cues, and thus is incorrect. Choice D does not accurately represent any of the three theories of emotion described in the lecture, and lacks the simultaneous emotional and physiological response to a stimulus described by the Cannon- Bard theory, making it incorrect as well.

Four grams of glucose circulate in the blood of an average person. How many grams of water would be produced if all the glucose in the blood underwent cellular respiration? The equation for cellular respiration is: C6H12O6 + 6 O2 --> 6 CO2 + 6 H2O A. 1.3 grams B. 2.4 grams C. 13 grams D. 24 grams

B. 2.4 grams convert grams of glucose to moles of glucose (4 g) (180 g/mol) = 0.022 moles balanced equation indicates that for every mole of glucose consumed, six moles of water are produced. therefore, about 0.13 moles of water will be produced for every 0.02 moles of glucose. the molecular weight of water is 18 g/mole so approximately 2.4 g of water would be present. (0.13 moles)(18 g/mol) = 2.4

Problem How many total molecules of ATP are synthesized from ADP via glycolysis of a single molecule of glucose? A. 38 B. 4 C. 2 D. 36

B. 4 Glycolysis is only the first step of a larger process known as cellular respiration. Glycolysis consumes ATP as well as producing it, though the question asks how many molecules are generated, not the "net" number generated. Two molecules of ATP are consumed in the "investment phase" and four molecules of ATP are generated in the "payoff phase". Four total molecules of ATP are generated via glycolysis of a single molecule of glucose.

When choosing a buffer to use for an experiment conducted at pH 5.3, it would be best to choose one with a pKa of: A. 2.14 B. 4.75 C. 6.5 D. 7.0

B. 4.75 Good experimental design protocols state that a good buffer has a pKa within 1 pH unit of the desired experimental conditions.

Which of the following is the primary neurotransmitter used by the parasympathetic nervous system? A. Epinephrine B. Acetylcholine C. Norepinephrine D. Dopamine

B. Acetylcholine ACh is released by all preganglionic neurons of the autonomic system, by all the postganglionic neurons of the parasympathetic nervous system, and by those postganglionic neurons of the sympathetic nervous system that innervate the sweat glands.

Citric acid is a weak triprotic acid. Because it has multiple acidic protons, a solution of citric acid can: A. Not act as an effective buffer at a pH near its pKa2 value. B. Acts as a buffer over several pH ranges C. only act as an effective buffer at a pH near its pKa2 value. D. Not act as an effective buffer at a pH near any of its pKa values.

B. Acts as a buffer over several pH ranges A buffer resists a change in pH. This relationship can be described using the HH equation where pH varies least around the pKa of a buffer when there is an equal proportion of weak acid and conjugate base. Because citric acid is triprotic, it has 3 pKa values and 3 different weak acid states (H3A, H2A-, and HA2-). This gives it the ability to act as a buffer across three different pH ranges, which in the case of citric acid overlap one another slightly.

If a scientist was interested in studying brain abnormalities in individuals with unusually high capacities for working memory, which of the following regions of the brain would she be mostly likely to study? A. Sympathetic ganglia B. Auditory cortex C. Hypothalamus D. Visual cortex

B. Auditory cortex This question requires an understanding of how working memory is encoded. Whereas sensory memory involves information from all sensory modalities, working memory is usually encoded through an auditory representation. Think, for example, of trying to remember a phone number in the time between hearing it and writing it down. For most people this task involves repeating the number sequence, either out loud or mentally, generating a phonetic representation. Choice B is the only answer that lists a brain region directly involved with auditory processing, so it is correct. The visual cortex is more likely to play a role in sensory or long term memory, so choice D can be eliminated. Although the hypothalamus is a brain region, it is associated with circadian rhythms and the endocrine system rather than memory encoding; choice Cis incorrect. Sympathetic ganglia, which are part of the peripheral autonomic nervous system, are not brain structures, and thus cannot be correct.

In a 1930s study on the relationship between attitudes and behavior, a researcher traveled with a Chinese couple to 251 hotels and restaurants. At the time of the study, it was thought that there was heavy prejudice towards Asians, and there were no laws against racial discrimination. After their travels, the researcher sent letters to all of the businesses and asked whether they would accept Chinese guests in their establishments. During their travels, only 1 establishment had refused to serve them. However, in the 128 letter responses received, 92% of the establishments said that they would not want to accept Chinese guests. According to the information offered in this study, which of the following is false? A. Cognitive and affective components of attitudes does not necessarily match behavior. B. Behavior influences attitudes through changes in cognition. C. This study demonstrated the gap between attitude and behaviors. D. Perceptions and attitudes toward race does not necessarily manifest in behavior.

B. Behavior influences attitudes through changes in cognition. This study shows the gap between attitude (not liking Chinese people) and behavior (serving Chinese people). This study shows that cognitive and affective (negative feelings towards Chinese people) components of attitudes does not necessarily lead to same types of behavior (negative).

Which of the following best describes the respective free energy changes for the one-step combustion of glucose in the presence of oxygen as compared to the multi-step oxidation of glucose during cellular respiration, assuming both reactions occur under identical conditions? A. More information is needed to answer the question B. Both reactions have the same change in free energy C. The combustion has a higher change in free energy D. The combustion has a lower change in free energy

B. Both reactions have the same change in free energy A fundamental concept in thermodynamics is that a thermodynamic quantity such as free energy is a state function. This means that free energy is determined by the state of the system, and not the path that the system took in order to arrive at the state.

A liquid situated in a gaseous medium reaches its boiling point when: A. its temperature equals that of the surrounding medium B. its vapor pressure equals that of the surrounding medium C. its density equals that of the surrounding medium D. its heat capacity equals that of the surrounding medium

B. its vapor pressure equals that of the surrounding medium liquid boils when vapor pressure is equal to the ambient pressure

Schizophrenia is a mental disorder that manifests itself through complex behavioral and emotional abnormalities. Although no single cause has ever been identified for schizophrenia, heredity is generally thought to be involved, especially since the disease is almost twice as common in males. Which of the following most likely explains the link between schizophrenia and genetic inheritance? A. The products of several Y-linked alleles interact with each other and lead to the attributes of schizophrenia B. Childhood experience may lead to schizophrenia in individuals with inherited X-linked recessive allele for a neurotransmitter transporter. C. Homozygous expression of a recessive X-linked synaptic scaffolding protein allele causes schizophrenia D. Homozygous expression of a dominant X-linked synaptic scaffolding protein allele causes schizophrenia

B. Childhood experience may lead to schizophrenia in individuals with inherited X-linked recessive allele for a neurotransmitter transporter. The question stem states that schizophrenia is more prevalent in males than females. This indicates that the genetic component of the disease is likely sex-linked. On the MCAT, a question about a phenotype that is more prevalent in males will likely turn out to be "X-linked." Y-linked diseases would not affect females at all, since they do not possess Y chromosomes. Since the question stem implies that women are susceptible to schizophrenia, choice A can be eliminated. One of the most important psychobiological concepts for the MCAT is that the interaction between genes and the environment influences behavior. Purely genetic explanations are rarely adequate for complex behaviors such as those associated with schizophrenia. Since both choices C and Dare restricted to genetic influences, they can both be eliminated. Choice B describes a relationship between an environmental factor and a genetic predisposition, and is thus the most likely explanation.

When gaseous ammonia is passed over solid copper (II) oxide at high temperatures, nitrogen gas is formed. 2NH3 + 3CuO --> N2 + 3Cu + 3H2O What is the limiting reagent when 34 grams of ammonia form 26 grams of nitrogen in a reaction that runs to completion? A. NH3 B. CuO C. N2 D. Cu

B. CuO Ammonia has molar mass of 17g; therefore, 34g of ammonia is equal to 2 moles. ammonia and nitrogen are consumed and produced in a 2 to 1 ratio so if ammonia were the limiting reagent, 28g of nitrogen would be produced. the molar mass of molecular nitrogen is 28g. since only 26g are produced, some other regent must be limiting the yield. the only other reactant is CuO

James wants to begin a training regimen. He gets up each morning for a few months, runs 2 miles, and then has a nutritious breakfast before work. One morning after months of this routine, he decides he wants to sleep in the next morning. Unfortunately for James, he wakes up at the same time he does every morning and feels like he should take a run. Although James is upset that he can't go back to sleep, he has created a healthy habit. What type of memory is habit learning and where in the brain is it located? A. He is learning a habit, a type of semantic memory, which is located in the neocortex. B. He is learning a habit, a type of procedural memory, which is located in the basal ganglia. C. He is learning a habit, a type of explicit memory, which is located in the medial temporal lobe. D. He is learning a habit, a type of episodic memory, which is located in the hippocampus.

B. He is learning a habit, a type of procedural memory, which is located in the basal ganglia. Habit learning occurs in a specific type of implicit memory. Explicit memory can be divided into two categories, episodic and semantic. Procedural memory is type of implicit memory. He is learning a habit, a type of procedural memory, which is located in the basal ganglia.

What is the function of the HYPOTHALAMUS? A. Sensory relay station B. Hunger and thirst; emotion C. Emotion and memory D. Arousal and alertness

B. Hunger and thirst; emotion It is subdivided into lateral hypothalamus, ventromedial hypothalamus, and anterior hypothalamus; serves homeostatic functions, key player in emotional experiences during high arousal states, aggressive behavior, and sexual behavior. Four F (Function): Feeding, Fighting, Flighting, (sexual) Functioning. Destruction of LH: Lacks Hunger Destruction of VMH: Very Much Hungry Destruction of AH: Asexual

Which of the following statements is (are) true concerning tight junctions? I. They connect adjacent cell II. They may form a barrier to extracellular fluids III. They have the greatest strength of all cellular adhesions A. I only B. I and II only C. II and III only D. I, II, and III

B. I and II only desmosomes are anchored to the cytoskeleton and are stronger than tight junctions

Which of the following forms of reproductive isolation are pre-zygotic? I. Hybrid Inviability II. Gametic Isolation III. Zygote Mortality IV. Temporal Isolation A. I, II, and IV B. II and IV C. I and III D. IV only

B. II and IV Pre -zygotic reproductive isolation refers to barriers that prevent two organisms from mating and occur before a zygote is successfully formed by the combination of a sperm and an egg. Hybrid inviability occurs when a hybrid is born, but cannot live to a reproductive age. Gametic isolation refers to the inability of two gametes (i.e. a sperm and egg) to join. Zygote mortality refers to the inability of a zygote to survive embryological development. Temporal isolation is when organisms are separated by a factor of time, such as day and night or summer and winter, and therefore cannot reproduce.

A sample of benzene undergoes a phase change from gas to liquid. The enthalpy change associated with this process is: A. negative, because IMF are broken in the process B. negative, because IMF are formed in the process C. positive, because IMF are broken in the process D. positive, because IMF are formed in the process

B. negative, because IMF are formed in the process During condensation (gas to liquid), IMF are being formed to hold molecules together in cohesive liquid.

A researcher is interested in learning more about linguistic similarities and differences between cultures. He shows swatches of blue and green paper (in a variety of different shades) to groups of individuals from Egypt, Russia, and the United States, and asks them what word they use for each color. He finds that individuals from the United States tend to broadly name the colors with two words, "blue" and "green." Individuals from Egypt have one word for all blue, green, and blue-green colors, "wadjet." Individuals from Russia have different words for each shade of blue. What does his study best support? A. Lexical access B. Linguistic relativity C. Linguistic universals D. Transformational grammar

B. Linguistic relativity Lexical access refers to identifying a word and connecting it to its meaning, which has been stored in long-term memory. Transformational grammar refers to the different ways that words can be arranged to convey the same information. Linguistic universals theory states that there are characteristics that remain consistent across all languages of different cultures. Linguistic relativity theory states that there are significant linguistic differences between cultures.

What important reducing agent is an important product of the pentose phosphate pathway? A. NADP+ B. NADPH C. NAD+ D. NADH

B. NADPH A reducing agent is an electron donor. A hydrogen atom contains one proton and one electron. Loss of the electron only would change the charge of a molecule from neutral to positive. NADH/ NAD+ are not associated with the pentose phosphate pathway. NADPH is an important reducing agent produced by the pentose phosphate pathway.

Which structures contain connective tissue, blood vessels, and nerves? A. Papillary dermis only B. Papillary and reticular dermis C. Epidermis, papillary dermis, and reticular dermis D. Reticular dermis only

B. Papillary and reticular dermis A continuous basement membrane separates the epidermis from the dermis. Capillaries in the papillary dermis are responsible for perfusing the epidermis - oxygen and nutrients diffuse across the basement membrane. Both the papillary and reticular dermises are comprised of connective tissue, and carry nerves and blood vessels to the epithelium.

How are electrons extracted from the citric acid cycle for use in the electron transport chain? A. Reduction of ATP and GTP B. Reduction of NAD+ and FAD C. Oxidation of ATP and GTP D. Oxidation of NAD+ and FAD

B. Reduction of NAD+ and FAD A molecule which undergoes the addition of electrons or decrease in oxidation state is said to be 'reduced'.

What is the function of the THALAMUS? A. Smooth movement B. Sensory relay station C. Sensorimotor reflexes D. Arousal and alertness

B. Sensory relay station Thalamus is a structure within the forebrain that serves as an important relay station for incoming sensory information, including all senses except for smell. After receiving incoming sensory impulses, thalamus sorts and transmits them to appropriate areas of cerebral cortex.

Suppose that multiple newly created amino acids interact to build a small protein molecule. The primary structure of that protein is formed when: A. The R-group of one amino acid forms intermolecular attractions with the R-group of another. B. The amino-terminus of one amino acid attacks the carboxy-terminus of another. C. H-bonds with the carboxy terminus of another D. The carboxy-terminus of one amino acid attacks the amino terminus of another.

B. The amino-terminus of one amino acid attacks the carboxy-terminus of another. The lone pair of electrons on one amino-terminus nucleophillically attack the carbonyl carbon of another amino acid's carboxy-terminus.

Which of the following accurately describes the difference between alpha-D-glucose and beta-D-glucose? A. alpha-D-glucose and beta-D-glucose are enantiomers and thus rotate polarized light in opposite directions. B. The hemiacetal carbons of alpha-D-glucose and beta-D-glucose have opposite absolute configurations. C. One is a reducing sugar and the other is not. D. alpha-D-glucose takes the pyranose ring confirmation while beta-D-glucose takes the furanose form.

B. The hemiacetal carbons of alpha-D-glucose and beta-D-glucose have opposite absolute configurations. alpha-D-glucose and beta-D-glucose are a pair of two anomers, which differ only in the absolute configuration at their anomeric carbon. The anomeric carbon is the carbon at the center of a hemiacetal group whose configuration will determine whether the -OH group is axial or equatorial in cyclic glucose.

A block of mass m is sitting motionless on a plane with an incline of θ, a coefficient of static friction of μs, and a coefficient of kinetic friction μk. Which of the following will occur once the plane is elevated on one side, increasing θ? A. The maximum force caused by static friction will increase. B. The maximum force caused by static friction will decrease. C. The coefficient of static friction μs will decrease. D. The coefficient of kinetic friction μk will decrease.

B. The maximum force caused by static friction will decrease. The coefficients of friction, both kinetic and static, are not affected by the slope angle of the surface and stays constant throughout the elevation. The coefficient of friction is determined experimentally and is usually only affected by the types of materials used in the interface between the surfaces of the object and plane. The maximum force of static friction is proportional to the normal force. The normal force is decreased as the plane is elevated because the component of gravitational force directed into the plane (Fgcos θ) decreases as θ increases. Therefore, as θ increases, the maximum force caused by static friction will decrease.

Which of the following does NOT depend on the attraction of the bonding pair towards the nucleus? A. The number of protons in the nucleus B. The repulsion by the electrons in the same valence shell C. The amount of shielding by inner shell electrons D. The distance from the nucleus

B. The repulsion by the electrons in the same valence shell Electronegativity is the tendency for an atom to attract a bonding pair of electrons. Electronegativity unlike other periodic trends is measured within a bond. Effective nuclear charge has a direct effect on each of the periodic trends. The greater the effective nuclear charge the greater the electronegativity. Effective nuclear charge depends on the number of protons in the nucleus, the distance from the nucleus, and the amount of shielding by inner shell electrons. The number of protons increases Zeff, while the distance and amount of shielding decreases Zeff. The repulsion by the electrons in the same valence shell is not measured by Zeff, and the attraction of the bonding pair does not depend on that repulsion.

In an experimental study, a group of people were given information on an individual's performance on Task A (success or failure). The group was also given information regarding the individual's past performances on the same task and other similar tasks. Success rates of other individuals performing the same task were also provided. The group was then asked to judge the impact of internal and external factors in influencing the individual's Task A performance outcomes. Which of the following statement is false? A. The results showed that if Task A performance outcomes were similar to results of others, the group attributed it to the external locus of personal control. This indicates that the group believed that factors such as task difficulty impacted the individual's Task A performance outcome. B. The results showed that if past performance records were inconsistent, then the group attributed the Task A performance outcomes to the individual's external locus of control. This indicates that the group believed that the individual's ability and effort influenced the performance outcome of Task A. C. The results showed that if past performance records were consistent, then the group attributed the Task A performance outcomes to the individual's internal locus of control. This indicates that the group believed that the individual's ability and effort influenced the performance outcome of Task A. D. The results showed that success, compared to failures, was more likely to be attributed to internal factors. This means that the group believed that the general effort and ability of the individual heavily influenced the performance outcome of Task A.

B. The results showed that if past performance records were inconsistent, then the group attributed the Task A performance outcomes to the individual's external locus of control. This indicates that the group believed that the individual's ability and effort influenced the performance outcome of Task A. The internal locus of personal control focuses on internal factors such as ability and effort of an individual. The external locus of personal control focuses on external factors such as task difficulty or luck that impacts the performance of an individual.

An ultrasound examination could show the motion of a fetus. In order to image this motion, the ultrasound examination devices requires what minimal information? A. The speed of the sound and the moving object B. The speed of the sound, and the frequencies of the sound waves emitted and observed. C. The speeds of the sound and of the moving object, and the frequencies of the sound waves emitted and observed. D. The speeds of the sound and of the moving object, and the frequencies and wavelengths of the sound waves emitted and observed.

B. The speed of the sound, and the frequencies of the sound waves emitted and observed. Doppler effect is used with ultrasound waves to provide fetal images. This effect relates the frequency of the ultrasound wave as detected by a moving detector to the frequency of the wave when the source is stationary, the speed of the source, and the speed of the detector. Three of the four quantities involved in the effect are required.

How do the majority of fatty acids enter the outer membrane of the mitochondria? A. Transferase facilitated entry of free fatty acids B. Transferase facilitated entry of carnitine bound fatty acids C. Free diffusion of hydrophobic free fatty acids D. Free diffusion of carnitine bound fatty acids

B. Transferase facilitated entry of carnitine bound fatty acids Coupling the transfer of large molecules to energetically favorable reactions allows this barrier to be overcome. Three reactions allow for FA entry into the mitochondria, in a process known as the carnitine shuttle. First, acyl-CoA synthetase links coenzyme A with the fatty acid, made favorable by the hydrolysis of two bond in ATP (to make AMP). Second, fatty acid acyl-CoA is attached to carnitine via carnitine acyltransferase I, and the acyl-carnitine enters the outer membrane. The majority of fatty acids enter the outer membrane of the mitochondria via transferase facilitated entry of carnitine bound fatty acids.

Imagine someone that was fasting for long period but then satiated his appetite with very fatty and sugary meal. What can be said about the hormonal changes from hungry to satiated. A. After eating he had increased secretion of insulin and ghrelin, and decreased secretion of glucagon and leptin B. While fasting he had increased secretion of glucagon, and ghrelin, and reduced secretion of leptin and insulin C. While fasting he had increased secretion of insulin, and leptin, and decreased secretion of glucagon and ghrelin D. After eating he had increased secretion of glucagon and ghrelin, and decreased secretion of insulin and leptin

B. While fasting he had increased secretion of glucagon, and ghrelin, and reduced secretion of leptin and insulin During fasting insulin secretion is kept low During fasting leptin secretion is kept low Leptin secretion indicates satiation During fasting ghrelin secretion is high, inducing hunger sensation

Anxiety disorders are LEAST characterized by which of the following traits? A. The experience of unwanted fear B. Worries about one's present circumstances C. A physical manifestation of excessive sympathetic nervous system activation D. The frequent experience of excessive responses to stress

B. Worries about one's present circumstances Anxiety is generally orientated toward concerns about the future and hypothetical situations rather than present circumstances

The activity of galactokinase causes a buildup of galactose-1-phosphate when the GALT enzyme is deficient because: A. galactosekinase lowers the energy of activation for the forward reaction in the phosphorylation of galactose and does not affect the energy of activation for the reverse reaction. B. galactokinase lowers the energy of activation for both the forward and reverse reactions in the phosphorylation of galactose. C. galactokinase lowers the energy of activation for the forward reaction in the phosphorylation of galactose and raises the energy of activation for the reverse reaction. D. galactokinase alters the relative amounts of reactant and product at equilibrium, causing the galactose-1-phosphate product to be favored.

B. galactokinase lowers the energy of activation for both the forward and reverse reactions in the phosphorylation of galactose. An enzyme (like any type of catalyst) lowers the energy of activation of both the forward and reverse reactions.

As non-polar protein groups cluster in water, the water experiences a(n): A. decrease in entropy B. increase in entropy C. decrease in hydrogen-bonding D. increase in the solvation layer

B. increase in entropy The organization of the solvation layer causes a decrease in entropy, so the clustering of non-polar groups, by virtue of diminishing the layer, causes a favorable increase in entropy. In fact, this increase in entropy is the predominant thermodynamic influence resulting in the clustering of nonpolar groups in polar solvents like water.

Increasing the volume of air that reaches the alveoli and takes part in gas exchange will cause blood pH to: A. increase, because the neural mechanisms that remove acid from the blood will be activated. B. increase, bc the partial pressure of CO2 in the blood will decrease. C. decrease, bc the affinity of Hb for oxygen will be increased D. decrease, bc the work associated with increased ventilation will come more O2.

B. increase, bc the partial pressure of CO2 in the blood will decrease Increasing the volume of air that reaches the alveoli and takes part in gas exchange will enhance O2 uptake and CO2 removal, thereby increasing blood pH.

One difference between prokaryotic and eukaryotic translation is that: A. eukaryotic ribosomes are larger B. prokaryotic translation may occur simultaneously with transcription, while eukaryotic translation cannot. C. prokaryotes do not contain supra molecular complexes such as ribosomes D. prokaryotic DNA is circular and thus does not require a termination sequence

B. prokaryotic translation may occur simultaneously with transcription, while eukaryotic translation cannot. Only choice B is both true and concerns translation. Choice A is incorrect because eukaryotic cells contain some ribosomes that are larger than those of prokaryotes. Prokaryotes do contain ribosomes, so choice C is wrong. Translation does not concern DNA, so choice Dis incorrect.

When a human female is born, the development of her oocytes is arrested in A. prophase of mitosis B. prophase I of meiosis C. prophase II of meiosis D. interphase

B. prophase I of meiosis It is worth memorizing that in humans, the life cycle of all oocytes is arrested at the primary oocyte stage (prophase I) until puberty. Therefore, choice B is the only correct answer. Even without knowledge of the oocyte life cycle, be aware that oocytes are gametes and therefore produced through meiosis, not mitosis. Choice A can be readily eliminated.

The partial double bond character of a peptide bond has its greatest effect in which level of structure of an enzyme? A. primary B. secondary C. tertiary D. quaternary

B. secondary The amino acid bias and shapes of the secondary structures (alpha helices and beta pleated sheets) are partially explained by the rigid structure of the peptide bond, whose double bond character prevents rotation and makes amide group planar. This rigidity provides steric constraints on hydrogen bond formation for some amino acids that prevents them from participating in some secondary structural elements.

Which of the following is an influence on identity formation that is similar to Freud's theory of how the superego develops? A. self-efficacy B. socialization C. peer pressure D. looking-glass self

B. socialization Theory of superego development involves the internalization of cultural values, imparted by the parents or other figures close to the child, quite similar to the process of socialization.

Excessive amounts of nitrogen are found in the urine of an individual who has experienced a period of extended fasting. This is most likely due to: A. glycogenolysis in the liver B. the breakdown of body proteins C. lipolysis in adipose tissue D. a tumor on the posterior pituitary causing excessive ADH secretion

B. the breakdown of body proteins Proteins are the only major nutrient containing nitrogen. Excess ADH secretion would result in decreased urine production.

Why do uncompetitive inhibitors cause a decrease in Km? A. the enzyme's apparent affinity for the substrate is decreased B. the enzyme's apparent affinity for the substrate is increased C. uncompetitive inhibitors bind to the enzyme only before it binds substrate. D. uncompetitive inhibitors do not actually cause a decrease in Km

B. the enzyme's apparent affinity for the substrate is increased Inhibitor only binds to the enzyme substrate complex but do not bind to the free enzyme. Km is decreased because the binding of the uncompetitive inhibitor prevents the enzyme and substrate from dissociating and therefore it takes a lower concentration of substrate to saturate the enzyme, lower the Km.

A referee at a soccer game notices that another referee made multiple incorrect calls and loudly criticizes the referee for being incompetent. When it is pointed out to the critical referee that she herself has made multiple incorrect calls, she blames each on poor visibility. This situation best exemplifies which of the following principles? I. Causation bias II. The fundamental attribution error III. Self-serving bias A. I only B. II only C. II and III only D. I, II, and III

C is correct. The fundamental attribution error occurs when one attributes others' actions to internal factors, rather than external circumstances. This often occurs in tandem with the self-serving bias, in which one attributes their own actions to external circumstances. In this question, both elements are present, as the referee attributes the other ref's actions to incompetence (an internal factor), while explaining away her own performance due to poor visibility (an external factor), making choice C correct and eliminating choice B. Causation bias refers to the tendency to assume a cause and effect relationship between correlated variables. This is not implied here; choices A and D can be eliminated.

A parent attempts to motivate a child to eat her vegetables by stressing how good it will feel for her to know that she will group up to be strong and healthy. Which theory of motivation is consistent with the parent's strategy? A. Drive reduction theory B. Incentive theory C. A cognitive theory D. A need-based theory

C is correct. The parent's emphasis on "how good it feels to know..." demonstrates an intrinsic motivation (an internal feeling of satisfaction that will be experienced by the child). This is in contrast to an extrinsic motivation, which, in this case, would be some type of tangible award received by the child after eating her vegetables. The use of intrinsic motivation most closely aligns with a cognitive theory in the absence of any further information, making choice C the best answer. Drive reduction theories focus on limiting basic physiological drives, such as hunger and thirst. As the parent does not emphasize reducing the feeling of hunger, choice A can be eliminated. Incentive theories use external rewards to motivate behavior, such as in operant conditioning. Because the reward here is internal, rather than external, choice B can be eliminated. Need-based theories refer to motivation driven by the desire to fulfill unmet needs. Because there is no mention or implication that the child's good feeling of knowing that she will grow up to be strong and healthy satisfies an unmet need, choice D can be eliminated.

What is the net charge on the R-groups of the five amino acids listed at pH 7.4? Ala-Thr-Gly-Asn-Tyr A. -2 B. -1 C. 0 D. 1

C. 0 Basic amino acids: lysine and arginine Acidic amino acids: aspartic and glutamic acid

Approximately how many molecules of ATP can be produced from the oxidation of a 20 carbon fatty acid, including those produced in the Krebs cycle? A. 107 B. 129 C. 136 D. 100

C. 136 Start simple-- a 20 carbon FA will yield 10 molecules of acetyl-CoA (20 carbons available, two carbons per acetyl-CoA). Recall that for each acetyl-CoA, the Krebs cycle produces 10 molecules of ATP. Remember that the process of oxidizing fatty acids to form molecules of acetyl-CoA also generates energy. Each pass of β-oxidation generates one molecule of NADH and one molecule of FADH2 To produce 10 acetyl-CoA, this fatty acid will require 9 passes of oxidation. 2.5 ATP can be generated per NADH, and 1.5 ATP per FADH2 2.5x9= 22.5; 1.5x9= 13.5; 13.5+22.5+100 = 136

How many grams of water are produced in the reaction between 2 moles of propylamine and 1 mole of cycloheanone? propylamine + cyclohexonone --> imine + water yield of imine is 78% A. 5 B. 10 C. 14 D. 18

C. 14 The yield of imine and water is 78% which represents the fractional conversion of the limiting reactant to product. In this case, limiting reactant is 1 mol of cyclohexonone. molecular weight of water is 18 g/mol and 0.78 mol of water has a mass of about 14 g.

The half life of a substance X is 45 years, and it decomposes to substance Y. A sample from a meteorite was taken which contained 1.5% of X and 13.5% of Y by mass. If substance Y is not normally found on a meteorite, what is the approximate age of the meteorite? A. 45 years B. 100 years C. 140 years D. 270 years

C. 140 years Since Y is not normally found in the meteorite, assume that all of the Y came from decomposition of X. The percentage of the sample that was X at the birth of the meteorite must have been the sum of the percentages of Y and X, or 15% (1.5% + 13.5%). The percentage of the sample that is X is now only 10% of that or 1.5%. After the first half life there would have been 50% left; after the second half life, 25% left; after 3rd half life, 12.5% left. Thus a little more than 3 half lives is required to reduce a substance to 10% of its original amount. 45x3 = 135 years

0.44 moles of AgNO3 and 0.20 moles of MgCl2 are mixed together. What is the mass percent composition of Ag in the product? A. 0.8% B. 25.7% C. 75.2% D. 90.9%

C. 75.2% starting number of moles is irrelevant information. The products of the reaction are AgCl and Mg(NO3)2. Product of interest is AgCl. There will always be 107.9 g of Ag per mole of product and 35.5 g of Cl per mole of product, since those are the molar masses respectively. Total mass of AgCl is 143.3 g/mol so the mass percent composition of Ag is 107.9/143.3 or 75.2%

Which of the following would least likely disrupt the Hardy-Weinberg equilibrium? A. Emigration of part of a population B. A predator that selectively targets the old and sick C. A massive flood killing 15% of a large homogenous population D. Exposure of the entire population to intense radiation

C. A massive flood killing 15% of a large homogenous population Catastrophic events will not cause significant genetic drift to a large, homogenous

Hormones are analogous to neurotransmitters as they both mediate communication between cells. Which type of signaling molecules and which class of signaling molecules are similar to a neuron using a 2nd messenger signaling to communicate with the postsynaptic membrane? A. A autocrine hormone because it establishes local communication, and protein because it does not use receptors the plasma membrane B. An endocrine hormone because it establishes distant communication , and steroid because it does not use receptors the plasma membrane C. A paracrine hormone because it establishes regional communication, and protein because it uses receptors in the plasma membrane D. An merocrine hormone because it established cell to cell communication, and tyrosine derived hormone because it uses receptors the plasma membrane

C. A paracrine hormone because it establishes regional communication, and protein because it uses receptors in the plasma membrane Autocrines self stimulate the cells that secreted, neurons communicate with each other. Endocrine hormones use the blood to travel long distances in the body. Merocine is a type of gland not a type of hormone A paracrine creates regional communication just like a neuron does, because it it depends on second messenger it the hormone cannot bypass the plasma membrane and therefore depend on receptors.

Which of the following statements is an example of explicit memory? A. A dog learns that food is associated with a bell. B. A 30 year-old woman recalling how to ride a bicycle after years of not riding. C. A teenager remembers her tenth birthday party. D. A man forms a habit of checking his email every night right before bed.

C. A teenager remembers her tenth birthday party. Implicit memories are nondeclarative and unconscious, while explicit memories are declarative and conscious. All memories formed by conditioning are implicit memories. All habits are procedural memories, a type of implicit memory. Memories that inform unconscious motor skills are procedural memories, a type of implicit memory. Memories that inform unconscious motor skills are procedural memories, a type of implicit memory.A teenager remembers her tenth birthday party, is an episodic memory (a memory about a specific event). Episodic memories are a type of explicit memory.

In E. Coli, the bglF transcript is known to have a short half-life within the cytosol. What mechanism is most likely responsible for transport of this transcript to the cytoplasmic membrane once it is synthesized? A. Diffusion across the cytoplasm B. Transport via attachment to the mitotic spindle C. Active transport along cytoskeletal filaments D. Transport from the endoplasmic reticulum in vesicles

C. Active transport along cytoskeletal filaments This allows for more rapid transport than diffusion A is incorrect because the process would take too long. B is incorrect because E. Coli (prokaryote) does not undergo mitosis D is incorrect bc E Coli cells do not have ER or vesicles

Which of the following statements regarding depression most likely misconstrues correlation as causation? A. One can have many depression-predisposing alleles without expressing depression B. A family history of depression predisposes one to exhibit depression C. Altered neural activation along HPA axis is responsible for depressive symptoms D. Those with depression are more likely to have altered monoamine functionality

C. Altered neural activation along HPA axis is responsible for depressive symptoms Choice C misconstrues correlation as a causation by stating that the alterations in brain activity that are correlated with depression actually cause depressive symptoms. This is not necessarily true: there may be an underlying cause that is responsible for both the depressive symptoms and altered neural activation. It could be the case that depression causes altered brain activity.

A study looking at the relationship between developmental dyslexia and biliteracy in young adults produces a correlation coefficient of -1.39. What does this indicate? A. Strong negative correlation - as developmental dyslexia increases, biliteracy decreases. B. Cause and effect relationship - developmental dyslexia causes biliteracy and vice versa. C. An error when computing the correlation between developmental dyslexia and biliteracy. D. Strong positive correlation - as developmental dyslexia increases, biliteracy also increases.

C. An error when computing the correlation between developmental dyslexia and biliteracy Strength of a correlation is based on how close the correlation coefficient is to 1 or -1. A correlation coefficient that is below -1 is likely the result of an error by the experimenter when computing the correlation between developmental dyslexia and biliteracy.

Which of the following statements correctly describes the relationship between node link strength and association in learning? A. As node link strength increases, the degree of association decreases. This increases the processing time. B. As node link strength decreases, the degree of association increases. This reduces the processing time. C. As node link strength increases, the degree of association increases. This reduces the processing time. D. As node link strength decreases, the degree of association decreases. This increases the processing time.

C. As node link strength increases, the degree of association increases. This reduces the processing time. Node link strength is a function of exposure. Increased exposure increases node link strength. The relative strength of the node links determines the amount of activation emitted to a network or a specific node. Stronger nodal links decrease processing time. Learning reduces processing time.

Which statement correctly identifies a similarity between cartilage and bone? A. Both are avascular. B. Both are innervated. C. Both are produced by specialized mesenchymal cells. D. Both are sites of hematopoiesis

C. Both are produced by specialized mesenchymal cells. The Haversian canals of bones provide pathways for nerves and blood vessels. Cartilage does not contain a structure comparable to Haversian canals. Hematopoiesis occurs within red bone marrow. The cells that produce cartilage (chondrocytes) and the cells that produce bone (osteoblasts) are both specialized mesenchymal cells. Mesenchymal cells are undifferentiated connective tissue cells, able to become osteoblasts, chondrocytes, and adipocytes.

Using knowledge of Michaelis-Menten kinetics, what effect would the addition of chloamphenicol, a competitive inhibitor, have on the kinetics of its target enzyme? A. Vmax decreases, and Km increases. B. Vmax decreases, Km unchanged. C. Vmax unchanged, Km increases D. Vmax remains unchanged, Km decreases

C. Vmax unchanged, Km increases A competitive inhibitor binds to the same site as the substrate. It thereby increases Km while leaving Vmax unchanged because high substrate concentrations displace the inhibitor.

Measurements of a compound reveal that it is 36% carbon, 6% hydrogen, and 48% oxygen, with systematic error accounting for the remaining 10%. Which of the following is the most accurate empirical formula for this compound? A. C6H1O8 B. C6H12O8 C. C1H2O1 D. C1H1O1

C. C1H2O1 Pick an arbitrary initial mass of 100 grams. The given analysis would suggest that this contains 36 g carbon, 6g hydrogen, and 48 g oxygen. Convert these masses into mols by dividing them by the known molar masses of the various elements: 36/12 = 3 mol carbon, 6/1 = 6 mol hydrogen, 48/16 = 3 mol oxygen Divide the three mol counts by their greatest common denominator to obtain the empirical formula: C1H2O1

Which of the following is NOT true regarding nuclear DNA and its associated proteins? A. Nucleosomes provide the structure for DNA to form into solenoids and supercoils. B. Histones contain basic functional groups that result in a net positive charge C. Constitutive heterochromatin is transcribed by cellular machinery D. By mass, chromatin contains more protein than DNA

C. Constitutive heterochromatin is transcribed by cellular machinery Nucleosomes are the structural level that allows the DNA/protein complex to form tightly organized spirals called solenoids and supercoils, eliminating choice A. Choice B is true and incorrect. Histone proteins do contain basic functional groups that give them a net positive charge at normal body pH, which encourages interaction with the negatively charged phosphates groups on DNA. By mass, chromatin is approximately one-third D A and two-thirds protein, eliminating choice D. Choice Cis untrue and correct because heterochromatin is too tightly wound to be accessed by cellular machinery. Before transcription can take place, heterochromatin must first be unwound, but constitutive heterochromatin is permanently wound and, therefore, is not transcribed.

Based on their compositional differences, different lipoproteins serve very different biological roles; what physical quality of lipoproteins most likely determines their function? A. Volume B. Shape C. Density D. pH

C. Density Lipoproteins are the spherical carrier molecular assemblies in which lipids are transported throughout the body. Examples include HDL, LDL, and chylomicrons. The functional properties of lipoproteins are largely determined by which apolipoproteins they express, as apolipoproteins are important signalling molecules for receptor-mediated recognition. Apolipoproteins are simply lipoproteins unbound to lipids. Different apolipoproteins will bind lipids in a different fashion, resulting in a difference in the ratio of lipid content to protein content. A high lipid content relative to a low protein content would be much less dense than a lipoprotein with a high protein content. The physical quality of lipoproteins that most likely determines their function is their density.

Suppose a healthy individual experiences a surge in plasma calcium. Subsequently, where would one expected to find the greatest amount of hydrolyzed PTH? A. Leaving the parathyroid glands B. Entering the liver C. Entering the nephron D. Leaving the haversian canals

C. Entering the nephrons If plasma calcium in a healthy individual is above normal, the body will respond by excreting calcium and PTH by-products. Excretion occurs in the kidneys, making the nephron a good prediction and the correct answer. The parathyroid gland produces PTH one would expect high levels leaving the liver, not entering. While it is likely that there will be some hydrolyzed products in systemic circulation (and therefore entering the bones), the highest concentration would still be found in the nephrons.

Which type of cell produces osteoid, a combination of proteins that become mineralized to produce new bone tissue? A. Osteoclasts B. Osteoprogenitor cells C. Osteoblasts D. Osteocytes

C. Osteoblasts Osteoclasts break down bone tissue by secreting tartrate resistant acid phosphatase. Osteoprogenitor cells are undifferentiated mesenchymal precursors to osteoblasts. They are not yet specialized to produce new bone tissue. Osteocytes are mature forms of osteoblasts that are no longer active. Osteoblasts produce osteoid, a combination of proteins that become mineralized to produce new bone tissue.

Which of the following scenarios represents disruptive selection within a population? A. Oak trees with medium-sized leaves are evolutionarily favored, as large leaves suffer from dehydration and small leaves don't receive sufficient sun. B. As pollution creates turbidity in a lake, brighter fish have greater reproductive success, causing the population to become more brightly colored on average. C. Heterozygous pink flowers are selected against while homozygous red and homozygous white flowers are selected for. D. A population of snakes is split by the construction of a large road, causing speciation of divided populations.

C. Heterozygous pink flowers are selected against while homozygous red and homozygous white flowers are selected for. Oak trees with medium-sized leaves being evolutionarily favored over those with large or small leaves would be an example of stabilizing selection (the opposite of disruptive selection). If a population of snakes is split by the construction of a large road it would likely cause speciation, but this is different than disruptive selection. Pollution creating turbidity in a lake and causing brighter fish to have greater reproductive success would be an example of directional selection.

Which of the following is true regarding chromosomes? A. The number of chromosomes in a human cell doubles during replication B. Diploid cells lack homologous chromosomes. C. Homologous chromosomes contain different alleles D. Chromosomes only form prior to transcription

C. Homologous chromosomes contain different alleles Homologous chromosomes contain information coding for the same traits, but each chromosome may contain different alleles coding for different phenotypes for each trait, making choice C the correct answer. Choice A is false because the number of chromosomes in a human cell prior to replication is 46 (23 homologous pairs) and the number following replication is also 46 (with sister chromatids). Choice B is also false. Diploid cells have homologous chromosomes; haploid chromosomes lack homologues. Choice D is incorrect because the tightly wound structure of chromosomes inhibits the process of transcription which can only take place on relatively unwound genetic material.

When an electron moves from a 2p to a 3s orbital, the atom containing that electron: A. becomes a new isotope B. becomes a new element C. absorbs energy D. releases energy

C. absorbs energy 3s orbitals are at higher energy than 2p orbitals. The atom would release energy if the electron moved to a lower energy orbital (from 3s to 2p)

Which of the following is always true concerning the base composition of DNA? A. In each single strand, the number of adenine residues equals the number of thymine residues. B. In each single strand, the number of adenine residues equals the number of guanine residues. C. In a molecule of double stranded DNA, the ration of adenine residues to thymine residues equals the ratio of cytosine residues to guanine residues D. In a molecule of double stranded DNA, the number of adenine residues plus thymine residues equals the number of cytosine residues plus guanine residues.

C. In a molecule of double stranded DNA, the ratio of adenine residues to thymine residues equals the ratio of cytosine residues to guanine residues

Predict the outcome of a mitochondrial membrane that is more permeable to hydrogen ions than normal. A. Reduced activity of electron transport chain B. Reduced formation of water C. Increased levels of inorganic phosphate in the mitochondrial matrix D. Increased activity of ATP synthase

C. Increased levels of inorganic phosphate in the mitochondrial matrix The electron transport chain pumps protons regardless of the permeability of the membrane. So water continues to be formed ATP synthase depends on proton gradient to form ATP A leaky membrane reduces the gradient of hydrogen, thus less ATP is formed, increasing the levels of inorganic phosphate in the mitochondrial matrix

Which of the following processes would NOT occur in response to reduced blood insulin levels? A. Decreased uptake of blood glucose by muscle cells B. Increased conversion of glycogen into muscle glucose C. Increased utilization of glucose as fuel D. Increased utilization of fatty acid as fuel

C. Increased utilization of glucose as fuel Insulin levels are reduced at times of low blood glucose concentration in order to conserve glucose for the use by the brain. Thus when insulin levels fall, tissues such as muscle and liver decrease glucose uptake and utilization, choice (A), and start using fatty acids instead, (D). When insulin levels are low, glucagon levels rise by default and promote conversion of glycogen into glucose, (B), to maintain blood glucose levels. The only process that doesn't occur in response low insulin levels, (C), increased utilization of glucose as fuel.

Which of the following is true, according to signal detection theory? A. An individual with high sensitivity has a hit rate roughly equal to the false alarm rate. B. A false alarm occurs when an individual fails to differentiate a signal from noise. C. Individuals with the same sensitivity to a signal can be more or less adept at identifying the signal D. Signal detection is determined solely by physical parameters like absolute thresholds

C. Individuals with the same sensitivity to a signal can be more or less adept at identifying the signal The hit rate describes the frequency that an individual is able to distinguish a signal from noise when the signal is actually there. A false alarm occurs when the individual perceives a signal that is really just noise. Choice B is an inaccurate statement and can be eliminated. Sensitivity is the individual's overall ability to discern signal from noise. In simplified terms, sensitivity is the hit rate minus the false alarm rate. Someone with equivalent hit and false alarm rates would have very low sensitivity, making choice A incorrect. Whether or not someone interprets a signal as such does not depend entirely on physical sensitivity levels. The individual has some level of conscious control over whether an input is interpreted as signal or noise, and this control is affected by biases. Bias is the reason that two individuals with the same physical abilities can perceive stimuli at different rates, making choice C correct and choice D incorrect.

Problem Research shows that our behavior is affected by intentions. In addition to intentions, however, it is also found that whether we have the resources or capacity to carry out these intentions also influence our behavior. This additional factor can be referred to as: A. Subjective norms B. Planned behavior C. Perceived behavior control D. Attitudes

C. Perceived behavior control This demonstrates a person's perception of the ease or difficulty in performing a behavior. Whether a person perceives that he or she has the resources or ability to carry out certain intentions influences their behavior. Perceived behavior control refers to a person's ability to carry out intentions to perform a certain behavior.

Most avian skeletons include pneumatic bones, which contain pockets of air that make the bones less dense. A researcher suspects that this design weakens the tensile strength of bone and that the skeletal system of birds must compensate for this loss by adjusting the composition of the bone matrix. If this hypothesis is true, how would the bone matrix of birds differ from that of other vertebrates? A. It would contain a larger number of osteoclasts. B. It would contain a relatively high proportion of hydroxyapatite. C. It would contain a relatively high proportion of collagen. D. It would contain larger lacunae.

C. It would contain a relatively high proportion of collagen. The question asks us to predict how birds might adjust the composition of their bone matrix to increase its tensile strength. Tensile strength is a material's ability to stretch longitudinally (get longer) before breaking. Although pneumatic bones have very large lacunae (empty spaces in the matrix), this characteristic is unlikely to add tensile strength to the bone. Osteoclasts reabsorb calcium be deconstructing the bone matrix. This characteristic is also unlikely to add tensile strength to bone. The bone matrix contains an organic portion composed of collagen and a protein mixture called osteoid as well as an inorganic portion composed of hydroxyapatite, which are basically calcium phosphate crystals. Which of these adds tensile strength to bone?

Which of the following would be expected to cross a cell membrane without the aid of an accessory protein? A. Glucose B. Glycine C. Ketone Bodies D. Triglycerides

C. Ketone Bodies Small, nonpolar molecules are most easily able to cross the cell membrane through simple diffusion because of the hydrophobic interior of the membrane. Oxygen is a small and nonpolar molecule. Therefore, it can diffuse across the cell membrane without the assistance of an accessory protein. Glucose and glycine are too polar to cross unassisted. Triglycerides, while nonpolar, are too large to cross the cell membrane without being broken down first.

Which of the following statements most accurately describes the characteristics of 1H NMR spectroscopy? A. Deshielded nuclei resonate at lower δ values, while shielded nuclei resonate at higher δ values. B. For a spin ½ nuclei, there are two possible energy levels: the lower occupied by spins aligned against and the higher occupied by those aligned with the magnetic field. C. Labile protons such as -OH and -NH have no characteristic chemical shift. D. The energy needed to flip spin energy states of the nuclei correspond to energies in the microwave region.

C. Labile protons such as -OH and -NH have no characteristic chemical shift. NMR spectroscopy uses non-ionizing EM radiation to flip the nuclei into the different energy states. More specifically, the nuclei absorb energies that are in the radio wave region, which is the region of lowest energy and longest wavelength. When there is an applied magnetic field, for a nuclei with ½ spin, there are two possible energy levels, assigned α (lower energy) and β (higher energy). By irradiating with the proper frequency, the α spin state, where spins are aligned with the magnetic field, flips to the β spin state, where spins are aligned against the magnetic field, with absorption of energy or resonance. A nucleus whose chemical shift is increased feels a stronger magnetic field due to the removal of electron density in neighboring atoms or groups. Lower electron density means less diamagnetic shielding, which in turn means a greater overall exposure to B0, a stronger Beff,, and a higher resonance frequency. Note that labile protons (-OH, -NH2, -SH) have no characteristic chemical shift, but their resonances can be definitely identified by the disappearance of a peak when reacted with D2O when deuterium replaces a protium atom.

Which of the following does not necessarily constitute social behavior? A. Nonverbal communication B. Mating behavior C. Learning behavior D. Agressive behavior

C. Learning behavior

All the participants in the study are given information regarding the benefits of a healthy diet. According to the cognitive dissonance theory, which hypothetical finding is most likely? A. Obese participants will change their unhealthy eating behaviors B. Non-obese participants will change their unhealthy eating behaviors C. Obese participants will question the validity of the information provided. D. Non-obese participants will overemphasize the importance of the information provided.

C. Obese participants will question the validity of the information provided. According to the cognitive dissonance theory, when an individual's attitudes are incongruent with his or her behavior, this leads to cognitive dissonance. To eliminate cognitive dissonance, the individual can either change his or her attitudes or his/her behavior. The theory posits that individuals are more likely to adjust their attitudes to align with their behavior than the other way around. Therefore, obese participants are likely to question the importance of the information provided.

Which structures in the male reproductive system will NOT necessarily lead to sterility if they became impaired? A. Degeneration of epididymis B. Prostatic cancer C. Obstruction of bulbourethral glands D. Blockage of the ampulla of vas deferens

C. Obstruction of bulbourethral glands Epididymis is essential for sperm maturation Vas deferens is responsible for conducting the sperm outside during ejaculation Prostatic cancer may lead to inhibition of secretions of prostate that are essential for sperm activation Bulbourethral glands produce pre ejaculatory fluid that aids in lubrication, thus its obstruction does not interfer with sperm production, maturation or activation

A psychiatrist has children complete tasks to assess development. Each child is shown a three dimensional model of a mountain scene, while positioned on one side with the psychiatrist observing from the opposite side of the scene. The child is asked to look at four pictures from different viewpoints including both the child's and the psychiatrist's viewpoints. Nathan, one of the children from the group, looks at the 4 pictures and selects the viewpoint that matches his own. Which of Piaget's stages of cognitive development has Nathan currently attained? A. Sensorimotor B. Concrete operational C. Preoperational D. Formal operational

C. Preoperational The psychiatrist is assessing each child's level of egocentrism utilizing a task developed by Piaget. Sensorimotor stage involves issues such as object permanence and stranger anxiety. The concrete operational stage describes children who are able to grasp concrete (real) events logically, conversion, and reversibility. In the formal operational stage, a child will be able to think logically about abstract ideas, hypothetical situations, and use abstract thinking to solve novel problems. The preoperational stage is associated with an inability to understand the perspective of others.

What is the function of the INFERIOR AND SUPERIOR COLLICULI? A. Vital function (breathing, digestion) B. Emotion and memory C. Sensorimotor reflexes D. Sensory relay station

C. Sensorimotor reflexes Prominent nuclei found in the midbrain where superior colliculus receives visual sensory input and inferior colliculus receives sensory information from auditory system. The inferior colliculus has a role in reflexive reactions to sudden noises.

Which of the following statements most accurately describes the chemical shift in 1H-NMR? A. Hydrogens attached to π-bonded carbons are more shielded than those in alkanes due to magnetic anisotropy B. Aryl, carbonyl, and allylic groups are electron-donating by induction and cause a downfield shift of a hydrogen on an adjacent carbon. C. The applied magnetic field causes the π electrons of aromatic compounds to circulate and produce a secondary magnetic field that causes the deshielding of the aromatic hydrogens. D. Electronegative atoms enhance the shielding of hydrogens and cause downfield shifts, and the extent of this shift is proportional to the electronegativity of the atom and proximity to the hydrogen.

C. The applied magnetic field causes the π electrons of aromatic compounds to circulate and produce a secondary magnetic field that causes the deshielding of the aromatic hydrogens. Chemical shifts are used to record the position of an absorption, where the standard chosen is tetramethylsilane (TMS). Increasing magnetic field is recorded towards the right of the spectrum, as well as a more shielded hydrogen, i.e. greater electron density. Electronegative atoms lessen the shielding of hydrogens by removing the electron density around them and cause them to feel a greater effective magnetic field (higher ppm). This will also result in a downfield shift, which is proportional to electronegativity and proximity. Hydrogens attached to π-bonded carbons are less shielded those in alkanes due to the electron-withdrawing nature. Magnetic anisotropy has different effects on attached protons for alkanes, alkenes, and alkynes. Aryl, carbonyl, and allylic groups are electron-withdrawing, not electron-donating, by induction and cause a downfield shift of a hydrogen on an adjacent carbon. The applied magnetic field causes the π electrons of aromatic compounds to circulate in a ring current. Such a current will produce a secondary magnetic field. When the field lines are aligned with the applied magnetic field, there is a greater magnetic field, which is equivalent to a deshielding effect.

Which of the following endocrine glands produces testosterone? A. anterior pituitary B. pancreas C. adrenal cortex D. adrenal medulla

C. adrenal cortex The adrenal cortex makes many other steroid based hormones, as well as testosterone. Again, the anterior pituitary hormones are FLAT PG. LH does stimulate the production of testosterone, but testosterone is not produced in the anterior pituitary. The pancreas produces insulin and glucagon. The adrenal medulla produces epinephrine and norepinephrine.

All of the following must change the rate of an enzyme catalyzed reaction EXCEPT: A. changing the pH B. lowering the temperature C. decreasing the concentration of substrate D. adding a noncompetitive inhibitor

C. decreasing the concentration of substrate

Decreasing progesterone levels during the luteal phase of the menstrual cycle are associated with: A. thickening of the endometrial lining in preparation for implantation of the zygote. B. increased secretion ofLH, leading to the luteal surge and ovulation. C. degeneration of the corpus luteum in the ovary. D. increased secretion of estrogen in the follicle, leading to the flow phase of the menstrual cycle.

C. degeneration of the corpus luteum in the ovary. Decreased progesterone secretion results from the degeneration of the corpus luteum, which occurs because fertilization of the egg and implantation didn't happen. Choice A is out because thickening of the endometrial lining occurs while estrogen and progesterone levels are high, not while progesterone secretion is decreasing. Choice B can be eliminated because increased estrogen secretion causes the luteal surge, and because the luteal surge occurs earlier in the cycle. Choice Dis out because while the flow phase does follow decreased progesterone secretion, it does not occur as a result of increased estrogen secretion.

The field of attribution theory focuses on the study of: A. how people subconsciously develop beliefs about the factors that motivate actions by others B. how each individual develops unique identity attributes C. how people consciously and unconsciously form ideas about the casual factors behind the behaviors of others D. how people reason about the casual factors behind their own behaviors

C. how people consciously and unconsciously form ideas about the casual factors behind the behaviors of others attribution theory considers both conscious and unconscious influences on the formation of attributions

Say the vaporization of water requires more heat than does the melting of ice. The most likely explanation for this is that: A. ice has a smaller heat capacity than does steam. B. ice has a larger heat capacity than does steam. C. melting requires that only a fraction of intermolecular forces be broken while vaporization requires complete separation of molecules. D. the temperature of ice must be raised less than does the temperature of water to achieve a phase change

C. melting requires that only a fraction of intermolecular forces be broken while vaporization requires complete separation of molecules. More energy is required to vaporize liquid than energy required to melt solid because intermolecular forces must be broken from liquid to gas.

Suppose that an enzyme inhibitor lowers Vmax and has no effect on Km. This inhibitor is likely to be: A. uncompetitive B. proteolytic C. noncompetitive D. mixed

C. noncompetitive

All of the following types of lipids can be found in cell membranes EXCEPT: A. glycolipids B. steroids C. prostaglandins D. sphingolipids

C. prostaglandins Prostaglandins are lipids that act as hormones, and hormones are secreted into the circulatory system and allow for communication with different organs or tissues.

Research has shown that when video and audio recordings of a person speaking are temporally aligned, observers are better able to remember both visual and auditory information about the recordings. By contrast, misaligned recordings cause a severe decline in the ability to remember either visual or auditory information, depending on the individual. The latter finding is an example of: A. sensation B. an absolute threshold C. selective attention D. divided attention

C. selective attention The first part of this question stem is extraneous, as the question only asks about the second half. The important information here is that viewing a recording with video and audio out of sync causes a severe decline in an individual's ability to remember details about either the auditory or visual information from the recording. If the question had described both auditory and visual memory decreasing for each participant, this would be a clear example of divided attention, which causes attention paid to each input to decline. Since the question states that either one or the other declines, the best explanation is that one input, either auditory or visual, is receiving all or most available attention at the expense of the other input. This is an example of selective attention. Thus choice Cis correct and D is incorrect. Since absolute thresholds are largely (but not entirely) determined by the physical parameters of sense organs, it is unlikely that they are being altered in this experiment, making choice B incorrect. Sensation is a relevant term but does not directly address the question; choice A is also incorrect.

Proline is not technically an alpha-amino acid. Due to the ring structure of proline, it cannot conform to the geometry of the alpha-helix and creates a bend in the polypeptide chain. This phenomenon assists in the creation of what level of protein structure? A. primary B. secondary C. tertiary D. quaternary

C. tertiary tertiary structure is defined by interactions between side groups

Cardiac output, which is the product of the heart rate and the stroke volume (the amount of blood pumped per contraction by either the left or the right ventricle), would most likely be: A. greater if measured using the stroke volume of the left ventricle B. greater if measured using the stroke volume of the right ventricle C. the same regardless of which stroke volume is measured D. dependent on the viscosity of the blood

C. the same regardless of which stroke volume is measured Stroke volume must be the same for both ventricles. If it weren't, there would be a never-ending backlog of blood in one or the other circulations, ending with the faster circulation running dry. To keep the whole system running smoothly, both halves of the circulation must pump the same quantity of blood with each stroke.

Which type of molecules is LEAST likely to be found in a eukaryotic cell membrane? A. Phospholipid B. Cholesterol C. Glycoprotein D. Peptidoglycan

D. Peptidoglycan Peptidoglycan is found in the cell walls of some bacteria, but not in the cell membranes of eukaryotes.

Which of the following is NOT a target characteristic that can influence attitude change? A. Knowledge base B. Age C. Willingness to engage with the argument D. Credibility of the speaker

D is correct. Choices A, B, and C all describe characteristics of the target, or the person whose attitude is to be changed. A target with relevant knowledge and a willingness to engage will be more likely to use central route processing, making choices A and C incorrect. Age is also an important factor: children respond more emotionally than rationally, making B incorrect. The credibility of the speaker is a characteristic of the message, not the target, so choice D is correct.

Aldosterone exerts its effects on target cells by: A) binding to a receptor at the cell surface, setting off a second-messenger cascade. B) diffusing into adrenal cortical cells, where it influences transcription of certain DNA sequences. C) flowing across the synapse, where it binds and initiates an action potential. D) entering into target cells, where it increases the rate of production of sodium-potassium pump proteins.

D) entering into target cells, where it increases the rate of production of sodium-potassium pump proteins. Aldosterone, as we can tell by its name, is a steroid (any hormone whose name ends in "sterone" or something similar is a steroid). This eliminates choice A, because steroid hormones do not need cell membrane receptors or second-messenger systems. They simply diffuse across the cell membrane. Choice B can be eliminated because the adrenal cortex is aldosterone's source, not its target tissue. Answer choice C describes how neurotransmitters act at a synapse. For the most part, endocrine hormones act by being released into the blood stream and having effects at target tissues. Epinephrine, also produced in the adrenal gland, is the exception. It has roles as both a hormone and neurotransmitter. Aldosterone exerts its effect as is written in answer choice D, increasing the production of sodium- potassium pump proteins.

An individual who believes which of the following is committing the fundamental attribution error? A. A classmate who did well on a test must have done so because she received tutoring B. A classmate from another culture who did poorly on a test must have done so due to a cultural bias in the test C. A classmate who did poorly on a test must have personal problems that prevented her from studying D. A classmate who did poorly on a test must have done so because she is not intelligent.

D. A classmate who did poorly on a test must have done so because she is not intelligent. Determine which answer choice describes a dispositional attribution--the belief that another person's actions were due to internal factors rather than situational ones. All of the answers except for choice D describe situational attributions, where success or failure is said to occur due to external factors.

Which of these scenarios describes a scenario from the perspective of the Cannon-Bard theory of emotion? A. A man, who is allergic to bees, encounters a bee. Simultaneously, the man interprets that his allergy to bees makes this encounter threatening, his heart beat increases, and he starts sweating. He labels the emotion he is experiencing as fear. B. A man, who is allergic to bees, encounters a bee. He interprets that his allergy to bees makes this encounter threatening. Simultaneously, the man's heart beat increases, he starts sweating, and he labels the emotion he is experiencing as fear. C. A man, who is allergic to bees, encounters a bee. The man's heart beat increases, he starts sweating, and he interprets these physiological changes as the emotion fear. D. A man, who is allergic to bees, encounters a bee. Simultaneously, the man's heart beat increases, he starts sweating, and he labels the emotion he is experiencing as fear.

D. A man, who is allergic to bees, encounters a bee. Simultaneously, the man's heart beat increases, he starts sweating, and he labels the emotion he is experiencing as fear. The James-Lange theory of emotion describes an event (encounter with bee) followed by a physiological response (fight or flight) which is interpreted as fear and fear is perceived. Lazarus theory requires that interpretation must happen before arousal or emotion, which happen simultaneously. The event (encounter with bee) was interpreted as being threatening before a physiological response (fight or flight) and emotion (fear) were to be perceived simultaneously. Since the event was interpreted as threatening a physiological response (fight or flight) and the emotion (fear) were elicited. Schachter-Singer theory describes an event (encounter with bee) which simultaneously elicits a physiological response (fight or flight) and an interpretation of the event. If these agree the emotion (fear) is experienced. Cannon-Bard theory describes an event (encounter with bee) which elicits simultaneous physiological response (fight or flight) and perception of an emotion (fear).

A therapist treats a patient with agoraphobia. If the therapist subscribes strongly to the tenets of cognitive-behavioral therapy, which of the following treatments would be most representative of the therapist's view of the causes of the disorder? A. Showing the patient pictures of crowded situations while playing soothing music B. Preparing the patient with relaxation techniques before having her spend a day in a busy airport. C. Prescribing a serotonin reuptake inhibitor to treat the symptoms of anxiety generally. D. Accompanying the patient to the mall and challenging her beliefs that she will become trapped with no escape.

D. Accompanying the patient to the mall and challenging her beliefs that she will become trapped with no escape. A CBT therapist would see the causes of disorder as stemming from the interactions between thoughts and behaviors. While a CBT therapist may use associative learning, Choice A, B, or C as a part of therapy, only choice D mentions correcting patterns of conscious thought and it is therefore more representative and the correct answer.

In humans chromosomal abnormalities are correlated with age of the mother during conception. How is this linked with meiosis? A. Because telophase II may not lead to formation of the first polar body B. Because eggs do not finish anaphase I until fertilization occurs C. Because metaphase II does not finish until ovulation occurs D. Because eggs are locked in prophase I from birth to ovulation

D. Because eggs are locked in prophase I from birth to ovulation Telopase II may lead to formation of second polar body Metaphase II finishes at fertilization, so after ovulation occurs Anaphase I occurs monthly during each ovulation cycle. Because crossing over occurs in prophase I and eggs are locked in prophase I from birth to their ovulation the longer they are in prophase I the more likely it is that chromosomal abnormality will occur

After electrons are passed from one protein complex to another, the final electron acceptor of the electron transport chain is: A. ATP B. H20 C. NADH D. O2

D. O2 Oxygen accepts the electrons (along with protons) to form water. ATP and NADH are not electron acceptors. H20 is the product of the electron transport chain.

All of the following may be true concerning catalysts and the reactions which they catalyze EXCEPT: A. Catalysts are not used up by the reaction B. Catalysts lower the energy of activation C. Catalysts increase the rate of reverse reaction D. Catalysts shift the reaction equilibrium to the right

D. Catalysts shift the reaction equilibrium to the right

What is the function of the CEREBRAL CORTEX? A. Sensorimotor reflexes B. Emotion and memory C. Vital function D. Complex, perceptual, cognitive, and behavioral

D. Complex, perceptual, cognitive, and behavioral The outer covering of the cerebral hemispheres; it's associated with everything from language processing to problem-solving, and from impulse control to long-term planning.

Acid bromides are the bromine analogs of acid chlorides. One would expect the reactivity of acid bromides to be: A. less than the reactivity of esters B. less than the reactivity of acid chlorides C. equal to the reactivity of acid chlorides D. greater than the reactivity of acid chlorides

D. Greater than the reactivity of acid chlorides Br is a better leaving group than Cl (HBr is more acidic) so acid bromides are predicted to be more reactive than acid chlorides.

Several friends embark on a road trip and the driver starts playing country music. She asks the other passengers if they are happy with the music selection and they all agree, even though the majority of them dislike country music. This situation best demonstrates which of the following principles? A. Diffusion of responsibility B. Group polarization C. Group dynamics D. Groupthink

D. Groupthink

Hyperhidrosis is characterized by excessive sweating, especially at the palms of the hands and soles of the feet. Which of the following could cause this disorder? A. Higher density of holocrine glands B. Apocrine gland activation with puberty C. Increased sweat at the base of hair follicles D. Increased stimulation of merocrine glands

D. Increased stimulation of merocrine glands Sweat glands can be distinguished by both location and substance secreted. While apocrine glands are activated at puberty, they are mostly found in the armpits and groin. Increased stimulation of the merocrine glands, which produce what most of think of as sweat, can lead to hyperhidrosis.

Which of the following best explains why the boiling point of water decreases with increasing altitude? A. Increasing altitude is accompanied by an increase in pressure B. Increasing altitude is accompanied by an increase in escape velocity C. Increasing altitude is accompanied by a decrease in temperature D. Increasing altitude is accompanied by a decrease in pressure.

D. Increasing altitude is accompanied by a decrease in pressure. Boiling happens when vapor pressure of a liquid is equal to the atmospheric pressure

Glucose intake by pancreas beta cells, leads to depolarization of the cell voltage gated calcium channels allow calcium inflow and exocytosis of insulin. A point mutation can lead to an increase in the efficiency of these voltage gated calcium channels. Predict a probable outcome effect of such mutation for patients with diabetes type 2? A. Insulin secretion would be brought to a halt B. Increased levels of blood sugar C. Increased secretion of glucagon D. Insulin resistance would develop faster

D. Insulin resistance would develop faster The mutation creates excessive insulin secretion not increased glucagon secretion More insulin would lower blood sugar levels Patients with diabetes type 2 develop insulin resistance do increased secretion of insulin, thus insulin resistance would develop faster in this scenario

Which of these traits is most heritable? A. Personality (extraversion/introversion) B. Religiosity C. Information processing speed D. Intelligence

D. Intelligence Religiosity in monozygotic twins has a coefficient of correlation value of 0.49 based on the Minnesota twin studies reared apart data. rocessing speed Personality in monozygotic twins has a coefficient of correlation value of 0.50 based on the Minnesota twin studies reared apart data. Information processing speed in monozygotic twins has a coefficient of correlation value of 0.56 based on the Minnesota twin studies reared apart data. Intelligence in monozygotic twins has a coefficient of correlation value of 0.69 based on the Minnesota twin studies reared apart data.

How does starvation influence secretion of hormones that regulate blood sugar levels and glucose metabolism? A. It increases glucagon levels, and inhibit glucogenesis B. It decreases glucagon levels, and stimulate gluconeogenesis C. It increases insulin levels, and inhibit glycolysis D. It decreases insulin levels, and stimulate gluconeogenesis

D. It decreases insulin levels, and stimulate gluconeogenesis Starvation promotes the breakdown of glucose reserves, glucogenesis Starvation leads to hypoglycemia and therefore promote the secretion of glucagon stimulating the breakdown of glycogen via glucogenesis Insulin promotes the use of glucose in the blood stream During starvation insulin levels are decreased, and gluconeogeneis, breakdown fats and protein to generate glucose, is stimulated

A researcher wants to replicate the Creaky Bridge experiment. Participants (all males) crossed either a fear-arousing bridge or a non-fear-arousing bridge. Midway across, the participants were asked to complete a survey by a woman (a confederate). All participants were informed by the confederate of the availability of a phone number so they could discuss any concerns regarding the survey with her personally. A significant number of participants in the fear-arousing bridge group contacted the confederate compared to the non-fear-arousing condition. What is this researcher investigating? A. Emotional intelligence B. Flynn effect C. Availability heuristics D. Misattribution of arousal

D. Misattribution of arousal An availability heuristic occurs when a judgement is made based on the perceived probability, the extent that one can remember or imagine similar occurrences, or whatever information that is available. The Flynn effect is an observation regarding the growth of IQ from one generation to the next. Emotional intelligence is the ability to understand emotions present in oneself and how those emotions motivate oneself and others. The men in this study are misattributing their physiological arousal from crossing the fear-arousing bridge as arousal toward the woman giving the survey and are, therefore, more likely to contact her to follow-up than the control participants.

Which of the following statements about Korsakoff syndrome is not true? A. Many individuals with Korsakoff syndrome experience symptoms like aphasia. B. Korsakoff syndrome related to severe thiamine deficiency. C. Korsakoff syndrome produces anterograde amnesia and retrograde amnesia. D. Most cases of Korsakoff syndrome are caused by traumatic brain injuries.

D. Most cases of korsakoff syndrome are caused by traumatic brain injuries. Aphasia is a communication disorder that causes problems with language, like speaking, listening reading, and writing. Korsakoff syndrome is caused by chronic thiamine deficiency. Individuals with Korsakoff syndrome have problem forming new memories and recalling old memories. Most cases of Korsakoff syndrome are caused by alcohol abuse, not traumatic brain injuries.

Epigenetic changes are on way the environment influences gene expression. Which of the following is NOT an example of an epigenetic change? A. A methyl group is added to a cytosine residue in the DNA sequence B. A methyl group is added to a lysine within a histone protein C. ncRNAs influence chromatin structure D. Mutagens in the environment cause the creation of a new single nucleotide polymorphism

D. Mutagens in the environment cause the creation of a new single nucleotide polymorphism Epigenetic changes help control gene expression by regulating the access of cellular machinery to certain areas of the genome through their effects on chromatin structure. Choices A, B, and Call affect chromatin structure and are examples of epigenetic changes. Epigenetic changes do NOT change the sequence of the genome's nucleotides (only the way the genome is read), making choice D the correct answer.

One theory of language development is that individuals are born with a Language Acquisition Device (LAD), an organ that enables humans to understand and develop language. Once young children are exposed to language, this organ activates and allows the child to understand the language they are being exposed to. What perspective does the LAD theory follow? A. Bilingualist B. Interactionist C. Behaviorist D. Nativist

D. Nativist This theory is based on innate ability. Behaviorist theories focused on abilities that are learned. This theory was developed by Noam Chomsky, a key figure in support of the nativist perspective.

Which statement is NOT compatible with the hypothesis that the self-serving bias can account for participants' explanations of their body weights? A. Obese patients view their unhealthy weight as a result of having too many fast food restaurants near home. B. Non-obese participants view their healthy weight as a result of having strong willpower. C. Obese participants view their unhealthy weight as a result of not having time to exercise regularly. D. Non-obese participants view their healthy weight as a result of not having any fast food restaurants near their home.

D. Non-obese participants view their healthy weight as a result of not having any fast food restaurants near their home. Self-serving bias suggests that when explaining their own behavior, individuals attribute positive behaviors to internal, stable sources, but attribute negative behaviors to external sources. A non-obese individual would attribute his or her healthy weight to an internal, stable source, such as strong willpower. However, a non-obese individual would not attribute his or her healthy weight to an external source, such as not having any fast food restaurants near home. Therefore, D is incompatible with the self-serving bias.

Which of the following lists of values, obtained from bomb calorimetry, constitutes sufficient information to determine the bond energy of activated glucosamine? A. Mass of reactants, heat of formation of reactants, temperature of products B. Temperature of reactants, mass of calorimeter and specific heat of calorimeter. C. Mass of water in calorimeter and specific heat of water. D. None of the above is sufficient to determine the bond energy.

D. None of the above is sufficient to determine the bond energy. Bomb calorimetry is a very inefficient way to determine the bond enthalpies within a compound when not starting from a standard state. In order to do so, we would need all of the information provided above, plus the heat of formation of the products and background information on bond energies contained within either the reactants or products.

A researcher studying student intentions to engage in safer sex practices found that individual attitudes toward wearing condoms, and their friend's attitudes toward wearing condoms explained the variations in the intent of an individual to use condoms in the short-term. However, the study further showed that the perception of one's ability to use condoms or of condom availability significantly explained the variation between condom usage and non-usage. This "perception" is an example of: A. Theory of perception B. Cognitive dissonance C. Social cognition D. Perceived behavior control

D. Perceived behavior control This study shows that a person's behavior is influenced by his or her perception of the ease or difficulty in performing a behavior. This study shows that a person's behavior is influenced by the perceptions of their own ability or access to resources in performing a behavior.

Lysine and amino acids with similar chemical characteristics in histones most likely promote the interaction of histones with which DNA components? A. Purines B. Pyrimidines C. Deoxyribose D. Phosphate groups

D. Phosphate groups Lysine is a basic, + charged amino acid that interacts with negatively-charged phosphate groups on DNA. A and B are wrong bc histones do not interact with DNA in a highly sequence-specific manner. C is wrong bc deoxyribose is not negatively charged.

Which hormone is consistently positively correlated with thickening of the endometrium? A. Estrogen B. Luteinizing hormone C. Follicle-stimulating hormone D. Progesterone

D. Progesterone Estrogen levels peak twice during the uterine cycle, and changes do not consistently correlate with changes in the endometrium. Luteinizing hormone levels peak prior to thickening of the endometrium. Follicle-stimulating hormone levels peak prior to thickening of the endometrium. Progesterone is a progestational hormone whose peak is correlated with thickening of the endometrium.

A person who memorizes large segments of speeches by repeating them out loud many times is practicing which memory encoding technique? A. Hypnosis B. Feature recognition C. Chunking D. Rehearsal

D. Rehearsal A person repeating a piece of information many times to facilitate memory formation is utilizing rehearsal, whereas chunking involves reorganizing large pieces of information into smaller segments to bypass the limits of working memory. Since the question stem involves the use of repetition to facilitate encoding, choice D is correct and choice C is incorrect. Feature recognition and hypnosis are unrelated to the question stem, and can both be eliminated.

Which of the following is an example of substrate-level phosphorylation? A. The phosphorylation of glucose by glucokinase B. The reduction of NAD+ by triosphosphate dehydrogenase C. The removal of a phosphate group from glucose 6-phosphate by glucose-6-phosphatase D. The addition of an inorganic phosphate to ADP by pyruvate kinase

D. The addition of an inorganic phosphate to ADP by pyruvate kinase Substrate-level phosphorylation reactions are characterized by the direct transfer of a phosphate group from a reactive intermediate onto an ADP or GOP molecule.

A researcher adds 30 J of energy to a sample of water of 0 deg C and detects no change in temperature. Which of the following best explains this observation? A. The energy is dissipated by friction between the water and the walls of the container. B. The energy is used to decrease the entropy of the water. C. The energy is used to raise the Gibbs free energy associated with water D. The energy is used to break intermolecular forces between water molecules

D. The energy is used to break intermolecular forces between water molecules Energy is added to either increase temperature or to break intermolecular forces. Since the researcher noticed no change in temperature, D is correct.

How does the free energy (G) of a system containing one mole of ADP compare to the free energy of a system containing one mole of AMP under standard physiological conditions? A. The free energy of the ADP system is negative and less than the free energy of the AMP system, which is also negative B. The free energy of the AMP system is positive and greater than the free energy of the ADP system, which is also positive C. The free energy of the ADP system is positive; the free energy of the AMP system is negative D. The free energy of the ADP system is positive and greater than the free energy of the AMP system, which is also positive

D. The free energy of the ADP system is positive and greater than the free energy of the AMP system, which is also positive The free energy of a system can never be negative. Remember, free energy (G) is not the same as free energy change (∆G). ADP contains one more phosphoanhydride bond than AMP.

If gluconeogenesis is occurring in the liver, which metabolic pathway will be happening in a red blood cell? A. Gluconeogenesis B. Glycogenolysis C. The citric acid cycle D. The pentose phosphate pathway

D. The pentose phosphate pathway The pentose phosphate pathway is used to generate NADPH, which will be used to reduce glutathione in red blood cells. In the fasting state, red blood cells rely upon liver gluconeogenesis for their glucose supply. They do not have the enzymes necessary to produce glucose themselves. Red blood cells do not store glycogen, so they cannot have glyocogenolytic pathways active. The citric acid cycle occurs in mitochondria, which red blood cells do not possess.

A researcher studying plasma membrane proteins uses a detergent to disrupt cellular membranes. Why might the researcher choose erythrocytes for this study? A. The erythrocytic plasma membrane is static in vivo. B. Typical membrane proteins are not soluble in aqueous solution. C. Membrane proteins are usually present at low levels in native environments. D. The plasma membrane is only a fraction of a typical cell's membranes.

D. The plasma membrane is only a fraction of a typical cell's membranes. Erythrocytes famously lack organelles. Organelles have membranes and membrane protein. Thus, in order to study plasma membrane proteins and not organelle membrane proteins, the researcher chose RBC.

Which of the following does not necessarily constitute an agent of socialization? A. The media that one consumes B. One's religion C. The school that one attends D. The works of an influential philosopher

D. The works of an influential philosopher

Additional gas is pumped inside a rigid container that stores compressed gas. Which of the following is a true statement about this system? A. The molar concentration of gas is decreasing. B. The volume of the container is decreasing. C. Pressure is constant throughout the compression. D. There is no work done on the container

D. There is no work done on the container. The molar concentration of gas is increasing as more moles of gas is added into the rigid container. A rigid container does not change in volume, it is an isochoric system. Pressure change is unknown in this system. Using PV=nRTPV=nRTP, V, equals, n, R, T for ideal gases, we expect nnn, moles of gas to increase and VVV, volume to stay constant. However, we do not know how pressure and temperature are being affected in this system. W=PΔV No work is done on or by the container since the container is isochoric.

The following may or may not be steps included in the endochondral ossification of long bones: I. Ossification of the epiphyses II. Ossification of the diaphysis III. Intramembranous ossification IV. Formation of the epiphyseal plate V. Formation of a hyaline cartilage model What is the correct sequence of events in ossification of a long bone? A. V → I → II → IV B. III → II → I → IV C. V → III → I → II → IV D. V → II → I → IV

D. V → II → I → IV Endochondral ossification is the process by which long bones develop through the ossification of cartilage. The first step in this process formation and growth of a hyaline cartilage model. Intramembranous ossification is the process by which flat bones like the skull form from a membrane of mesenchymal tissue. This is not a step in endochondral ossification but rather a separate process of ossification. The first place where bone tissue develops on a long bone is at the primary ossification center, which can be found near the center of the shaft of a long bone. After primary ossification begins in the shaft, or diaphysis, of a long bone, secondary ossification begins on both ends, in the epiphyses. The epiphyseal plate is an area of hyaline cartilage that remains between the diaphysis and each epiphysis through adolescence; in adults the epiphyseal plate becomes the epiphyseal line. The correct sequence of events in ossification of a long bone are: V → II → I → IV.

Problem Researchers studying memory utilized a common test called the delayed response task, where a participant must respond to a stimulus that occurred in the past. In this test, the researchers had each participant listen to a sequence of words from a specific category, such as types of fruit, shapes, or colors, and prompted participants to repeat the previous word. After the participant repeated 10 words, the category was changed and the participant was asked to repeat the word that occurred two words previously in the list (researcher: "green, blue, red" - participant: "blue"). After every 10 words repeated by the participant, the delay is increased until the participant can no longer correctly identify the correct word. What type of memory is being tested in this study? A. The primacy effect B. Procedural memory C. Implicit memory D. Working memory

D. Working Memory Implicit memories are formed unconsciously, the participants are attempting to form these memories. Procedural memory is long-term memory for actions or habits such as how to kick a ball or washing hands before eating. The primacy effect occurs when a person remembers the items at the beginning of a list better than items at other locations in the list. Working memory is memory that is stored while it is held in attention. Working memory is being tested in this study.

Which of the following would NOT be predicted to be paramagnetic based on the electronic configurations given? A. Sc2+ -- [Ar]3d1 B. Mn2+ -- [Ar]3d5 C. Co3+ -- [Ar]3d6 D. Zn2+ -- [Ar]3d10

D. Zn2+ -- [Ar]3d10 An atom/ion is paramagnetic if at least one electron is unpaired. Eliminates A and B. There are five d orbitals, and when filing them, 1 electron goes into each orbital before they start to pair up. This, the 6 d-orbital electrons in Co3+ are placed as follows: the first 5 electrons go one at a time into the five available d orbitals, and the sixth pairs up with the electron in the first d orbital, leaving the other four d orbitals with one (unpaired electron).

Osteoporosis is an absolute decrease in bone tissue mass, especially trabecular bone. All of the following might be contributory factors to the disease EXCEPT: A. increased sensitivity to endogenous parathyroid hormone B. defective intestinal calcium absorption C. menopause D. abnormally high blood levels of calcitonin

D. abnormally high blood levels of calcitonin Calcitonin builds bone mass -it "tones the bone". If there were excess levels of calcitonin this would increase bone tissue mass. Parathyroid hormone increases blood calcium levels in part by breaking down bone. Bring more sensitive to parathyroid hormone would certainly lead to a decrease in bone tissue mass. Defective intestinal calcium absorption would lead to low blood calcium levels and activation of parathyroid hormone in an attempt to restore blood calcium. Parathyroid hormone would break down bone to release the calcium into the blood. Menopause contributes to osteoporosis by reducing estrogen levels leading to diminished osteoblastic activity. Choice D has the opposite effect of any of the other choices, so it can also be picked in this way. The answer that is different from the rest will often be correct.

The purpose of a twin study is usually to A. compare monozygotic and dizygotic twins to their non-twin siblings B. randomly assign twins to certain environmental conditions to separate the effect of genetics and environment C. examine whether genetic or environmental factors are responsible for the presence or absence of a trait D. determine the degree to which genetic inheritance influences a trait

D. determine the degree to which genetic inheritance influences a trait Set up of twin study is the comparison of monozygotic and dizygotic twins in order to separate the effects of the environment and genetics on some aspect of psychological function, usually personality. By comparing twin pairs that have different levels of genetic relatedness but similar environments, a twin study allows the determination of how much hereditary influences the expression of a trait Not A= twin study compares two types of twins, not twins and non-twins Not B = poses ethical and practical problems Not C= suggests that only genetic OR environmental factors are responsible for a trait but both types of factors influence the development of traits.

Vygotsky's theory of development is best described as a theory of how children: A. learn to follow social norms in accordance with their in-groups B. achieve each potential developmental level of identity formation C. master stages of social learning sequentially D. learn new abilities and social roles with guidance of others

D. learn new abilities and social roles with guidance of others Recall that Vygotsky's theory describes a gradual process of sociocultural learning, in contrast to the theories that describe stages that must be achieved in a set order.

What is sublimation? A. gas to liquid B. liquid to gas C. solid to liquid D. solid to gas

D. solid to gas


संबंधित स्टडी सेट्स

AP Biology Classification Review

View Set

Madness of Crowds by Douglas Murray quotes

View Set

1.3.5 Quiz - Week Three: Inductive & Deductive Reasoning

View Set

Apologia Biology Module 10 Study Guide for Emma

View Set

Rasgos heredados y comportamientos aprendidos

View Set

Anatomy - Unit 2 Movement Quiz Review

View Set

Fundamental Concepts and Skills for Nursing Ch.8

View Set

Essential Interviewing Midterm 2

View Set

California Certified Veterinary Assistant Final Review - Breed Identification - Unit 4

View Set